You are on page 1of 176

Chapter 2

2.1

Nominal rate(%)(NR) 5 10 20 60
Inflation rate(%) ( IR) 2 4 10 40
Real rate by the rule of thumb(%) 3 6 10 20
= NR - IR
Correct real rate (%) 2.9 5.7 9.0 14.29
=(1+NR)/(1+IR) -1 4 7 9
Error from using the rule of 0.0 0.2 0.9 5.71
thumb(%) 6 3 1

Chapter 3
FINANCIAL STATEMENTS, TAXES AND CASH FLOW

3.1.

(a)
Classified cash flow statement
For the Period 01.04.20 X 0 to 31.03.20 X 1 (Rs. in million)
------------------------------------------------------------------------------------------------------------
A. Cash flow from operating activities
- Net profit before tax and extraordinary items 150
- Adjustments for
Interest paid 30
Depreciation 30
- Operating profit before working capital changes 210
- Adjustments for
Inventories (20)
Debtors (20)
Trade creditors 20
- Cash generated from operations 190
Income tax paid (30)
- Cash flow before extraordinary items 160
Extraordinary item (60)
- Net cash flow from operating activities 100
B. Cash flow from investing activities
- Purchase of fixed assets (50)
- Net cash flow from investing activities (50)
C. Cash flow from financing activities\
- Additional share capital 20
- Proceeds loans 10
- Interest paid (30)
- Dividends paid (40)
Net cash flow from financing activities (40)

D. Net increase in cash and cash equivalents 10


- Cash and cash equivalents as on 31.03.2000 20
- Cash and cash equivalents as on 31.03.2001 30

(b)

A. Cash flow from assets


- Operating cash flow 120
- Net capital spending (50)
- Change in net working capital (30)
- Cash flow from assets 40

B. Cash flow to creditors


- Interest paid 30
- Net new borrowing (10)
- Cash flow to creditors 20

C. Cash flow to shareholders


- Dividends paid 40
- Net new equity raised (20)
- Cash flow to shareholders 20

We find that
(A) = (B) + ( C)

i.e., Cash flow from assets = Cash flow to creditors + Cash flow to shareholders

3.2.
(a)
Classified cash flow statement
For the Period 01.04.20 X 0 to 31.03.20 X 1 (Rs. in million)
------------------------------------------------------------------------------------------------------------
A. Cash flow from operating activities
- Net profit before tax and extraordinary items 100
- Adjustments for
Interest paid 30
Depreciation 20
- Operating profit before working capital changes 150
- Adjustments for
Inventories 10
Debtors (10)
Trade creditors 10
Provisions (5)
Increase in other assets (5)
- Cash generated from operations 160
- Income tax paid (20)
- Cash flow before extraordinary items 140
Extraordinary item (50)
- Net cash flow from operating activities 90
B. Cash flow from investing activities
- Purchase of fixed assets (30)
- Net cash flow from investing activities (30)

C. Cash flow from financing activities


- Repayment of term loans (15)
- Interest paid (30)
- Dividends paid (20)
Net cash flow from financing activities (65)

D. Net increase in cash and cash equivalents (5)


- Cash and cash equivalents as on 31.03.2000 20
- Cash and cash equivalents as on 31.03.2001 15

Note It has been assumed that “other assets” represent “other current assets”.

(b)

B. Cash flow from assets


- Operating cash flow 80
- Net capital spending (30)
- Change in net working capital (5)
- Cash flow from assets 45

B. Cash flow to creditors


- Interest paid 30
- Repayment of debt (5)
- Cash flow to creditors 25
C. Cash flow to shareholders
- Dividends paid 20
- Net new equity raised 0
- Cash flow to shareholders 20

We find that
(A) = (B) + ( C)

i.e., Cash flow from assets = Cash flow to creditors + Cash flow to shareholders

Chapter 4
ANALYSING FINANCIAL PERFORMANCE

Net profit
1. Return on equity =
Equity

= Net profit Net sales Total assets


x x
Net sales Total assets Equity

1
= 0.05 x 1.5 x = 0.25 or 25 per cent
0.3

Debt Equity
Note : = 0.7 So = 1-0.7 = 0.3
Total assets Total assets

Hence Total assets/Equity = 1/0.3

2. PBT = Rs.40 million


PBIT
Times interest covered = = 6
Interest

So PBIT = 6 x Interest
PBIT – Interest = PBT = Rs.40 million
6 x Interest = Rs.40 million
Hence Interest = Rs.8 million

3. Sales = Rs.7,000,000
Net profit margin = 6 per cent
Net profit = Rs.7000000 x 0.06 = 420,000
Tax rate = 60 per cent
420,000
So, Profit before tax = = Rs.1,050,000
(1-.6)
Interest charge = Rs.150,000

So Profit before interest and taxes = Rs.1,200,000


Hence
1,200,000
Times interest covered ratio = = 8
150,000

4. CA = 1500 CL = 600
Let BB stand for bank borrowing
CA+BB
= 1.5
CL+BB

1500+BB
= 1.5
600+BB

BB = 120

1,000,000
5. Average daily credit sales = = 2740
365
160000
ACP = = 58.4
2740

If the accounts receivable has to be reduced to 120,000 the ACP must be:
120,000
x 58.4 = 43.8days
160,000

Current assets
6. Current ratio = = 1.5
Current liabilities

Current assets - Inventories


Acid-test ratio = = 1.2
Current liabilities
Current liabilities = 800,000
Sales
Inventory turnover ratio = = 5
Inventories
Current assets - Inventories
Acid-test ratio = = 1.2
Current liabilities

Current assets Inventories


This means - = 1.2
Current liabilities Current liabilities

Inventories
1.5 - = 1.2
800,000

Inventories
= 0.3
800,000

Inventories = 240,000

Sales
=5 So Sales = 1,200,000
2,40,000

7. Debt/equity = 0.60
Equity = 50,000 + 60,000 = 110,000
So Debt = Short term bank borrowing = 0.6 x 110,000 = 66,000
Hence Total assets = 110,000+66,000 = 176,000
Total assets turnover ratio = 1.5
So Sales = 1.5 x 176,000 = 264,000
Gross profit margin = 20 per cent
So Cost of goods sold = 0.8 x 264,000 = 211,200
Day’s sales outstanding in accounts receivable = 40 days
Sales
So Accounts receivable = x 40
360

264,000
= x 40 = 29,333
360
Cost of goods sold 211,200
Inventory turnover ratio = = = 5
Inventory Inventory

So Inventory = 42,240

As short-term bank borrowing is a current liability as well,

Cash + Accounts receivable


Acid-test ratio =
Current liabilities

Cash + 29,333
= = 1.2
66,000
So Cash = 49867

Plant and equipment = Total assets - Inventories – Accounts receivable – Cash


= 176,000 - 42240 - 29333 – 49867
= 54560

Pricing together everything we get

Balance Sheet
Equity capital 50,000 Plant & equipment 54,560
Retained earnings 60,000 Inventories 42,240
Short-term bank borrowing 66,000 Accounts receivable 29,333
Cash 49,867

176,000 176,000

Sales 264,000
Cost of goods sold 211,200

8. For purposes of ratio analysis, we may recast the balance sheet into report form as under.
Let assume that ‘Others’ in the balance sheet represents other current assets.

Liabilities and Equity


Equity capital 10,000,000
Reserves and surplus 22,500,000
Long-term debt 12,500,000
Short-term bank borrowing 15,000,000
Total 60,000,000
Assets
Fixed assets (net) 30,000,000
Current assets
Cash and bank 5,000,000
Receivables 15,000,000
Inventories 20,000,000
Pre-paid exp 2,500,000
Others 2,500,000 45,000,000
Less: Current liablilities
Trade creditors 10,000,000
Provisions 5,000,000 15,000,000
Net current assets 30,000,000
Total 60,000,000

(i) Current ratio


= Current assets/ Current liabilities

45,000,000
= = 1.5
30,000,000
Note: Please note that for the purpose of calculation of current ratio and acid –test ratio, we have
to include short-term bank borrowings in current liabilities.

Current assets – Inventories 25,000,000


(ii) Acid-test ratio = = = 0.8
Current liabilities 30,000,000

Long-term debt + Short-term bank borrowings


(iii) Debt-equity ratio =
Equity capital + Reserves & surplus

12,500,000 + 15,000,000
= = 0.8
10,000,000 + 22,500,000
Profit before interest and tax
(iv) Times interest coverage ratio =
Interest

15,100,000
= = 3.02
5,000,000

Cost of goods sold 72,000,000


(v) Inventory turnover period = = = 3.6
Inventory 20,000,000
365
(vi) Average collection period =
Net sales/Accounts receivable
365
= = 57.6 days
95,000,000/15,000,000

(vii) Total assets =Equity + Total debt =( 10,000,000 + 22,500,000 ) +(12,500,000+15,000,000)


= 60,000,000

Net sales 95,000,000


Total assets turnover ratio = = = 1.6
Total assets 60 ,000,000

Profit after tax 5,100,000


(ix) Net profit margin = = = 5.4%
Net sales 95,000,000

PBIT 15,100,000
(x) Earning power = = = 25.2%
Total assets 60,000,000

Equity earning 5,100,000


(xi) Return on equity = = = 15.7%
Net worth 32,500,000
The comparison of the Omex’s ratios with the standard is given below

Omex Standard
Current ratio 1.5 1.5
Acid-test ratio 0.8 0.8
Debt-equity ratio 0.8 1.5
Times interest covered ratio 3.02 3.5
Inventory turnover ratio 3.6 4.0
Average collection period 57.6 days 60 days
Total assets turnover ratio 1.6 1.0
Net profit margin ratio 5.4% 6%
Earning power 25.2% 18%
Return on equity 15.7% 15%

9. We may rearrange the balance sheet figures in the report form as under, for purposes of
ratio analysis. It is assumed that ‘Other assets’ are other current assets.

20X 20X 20X 20X


1 2 3 4 20X5
Share capital 2.4 2.4 3 3 3.2
Reserves and surplus 0.6 1 1.5 2 2.5
Long-term debt 1.2 1.3 2 2.3 2.6
Short-term bank
borrowing 1.2 1.4 2.1 2.5 2.6

Total 5.4 6.1 8.6 9.8 10.9


Assets
Net fixed assets 2.5 3.2 4.4 4.7 4.8
Current assets
Cash and bank 0.5 0.6 0.7 0.8 0.7
Receivables 1.5 1.6 2.3 2.6 3.2
Inventories 2 2.2 3 3.7 4.2
Other assets 0.2 4.2 0.3 4.7 0.3 6.3 0.4 7.5 0.6 8.7
Less: Current
liabilities 1.3 1.3 1.8 1.8 2.1 2.1 2.4 2.4 2.6 2.6
Net current assets 2.9 2.9 4.2 5.1 6.1
Total 5.4 6.1 8.6 9.8 10.9
20X1 20X2 20X3 20X4
20X5
Current ratio 1.68 1.47 1.50 1.53 1.67
Debt-equity ratio 0.80 0.79 0.91 0.96 0.91
Total assets turnover
ratio 0.74 1.00 0.91 0.93 1.03
Net profit margin(%) 5.00 6.56 3.85 5.49 6.25
Earning power (%) 9.26 18.03 11.63 13.27 18.35

Return on equity (%) 6.67 11.76 6.67 10.00 12.28

MINICASE

Solution:

(a) Key ratios for 20 X 5


12.4
Current ratio = ---------- = 0.67
6.7+11.7

3.8 + 11.7
Debt-equity ratio = = 0.98
6.5 + 9.3

57.4
Total assets turnover ratio = = 1.96
[(34 – 6.6) + (38 – 6.7)] / 2

3.0
Net profit margin = = 5.2 percent
57.4

5
Earning power = = 17.0 percent
[(34 – 6.6) + (38 – 6.7)] / 2

3.0
Return on equity = = 20.2 percent
(13.9 + 15.8) / 2
(b) Dupont Chart for 20 x 5

Net sales +/-


Non-op. surplus
deficit 57.8

Net profit
3.0 –

Net profit Total costs


margin ÷ 54.8
5.2%

Net sales
57.4
Return on
total assets
10.2%

Net sales
57.4

Total asset Average


turnover fixed assets
1.96 ÷ 21.4

+
Average total
assets Average
29.35 net current
assets 54.0

Average
other assets
2.55
(c) Common size and common base financial statements

Common Size Financial Statements


Profit and Loss Account

Regular (in million) Common Size (%)


20 X 4 20 X 5 20 X 4 20 X 5
• Net sales 39.0 57.4 100 100
• Cost of goods sold 30.5 45.8 78 80
• Gross profit 8.5 11.6 22 20
• Operating expenses 4.9 7.0 13 12
• Operating profit 3.6 4.6 9 8
• Non-operating surplus / 0.5 0.4 1 1
deficit
• PBIT 4.1 5.0 11 9
• Interest 1.5 2.0 4 3
• PBT 2.6 3.0 7 5
• Tax - - - -
• Profit after tax 2.6 3.0 7 5

Balance Sheet

Regular (in million) Common Size (%)


20 X 4 20 X 5 20 X 4 20 X 5
• Shareholders’ funds 13.9 15.8 51 50
• Loan funds 13.5 15.5 49 50
Total 27.4 31.3 100 100
• Net fixed assets 19.6 23.2 72 74
• Net current assets 5.1 5.7 19 18
• Other assets 2.7 2.4 10 8
Total 27.4 31.3 100 100
Common Base Year Financial Statements
Profit and Loss Account

Regular (in million) Common Base Year(%)


20 X 4 20 X 5 20 X 4 20 X 5
• Net sales 39.0 57.4 100 147
• Cost of goods sold 30.5 45.8 100 150
• Gross profit 8.5 11.6 100 136
• Operating expenses 4.9 7.0 100 43
• Operating profit 3.6 4.6 100 128
• Non-operating surplus / 0.5 0.4 100 80
deficit
• PBIT 4.1 5.0 100 122
• Interest 1.5 2.0 100 133
• PBT 2.6 3.0 100 115
• Tax - - 100 100
• Profit after tax 2.6 3.0 100 115

Balance Sheet

Regular (in million) Common Base Year(%)


20 X 4 20 X 5 20 X 4 20 X 5
• Shareholders’ funds 13.9 15.8 100 114
• Loan funds 13.5 15.5 100 115
Total 27.4 31.3 100 114
• Net fixed assets 19.6 23.2 100 118
• Net current assets 5.1 5.7 100 112
• Other assets 2.7 2.4 100 89
Total 27.4 31.3 100 114

(d) The financial strengths of the company are:

• Asset productivity appears to be good.


• Earning power and return on equity are quite satisfactory
• Revenues have grown impressively over 20 x 4 – 20 x 5

The financial weaknesses of the company are:

• Current ratio is unusually low


• While revenues grew impressively, costs rose even faster: As a result profit margins
declined
• The company did not have any tax liability in the last two years. If the company has to
bear the burden of regular taxes, its return on equity will be adversely impacted

(e) The following are the problems in financial statement analysis

• There is no underlying theory


• It is difficult to find suitable benchmarks for conglomerate firms
• Firms may resort to window dressing
• Financial statements do not reflect price level changes
• Diversity of accounting policies may vitiate financial statement analysis
• It is somewhat difficult to judge whether a certain ratio is ‘good’ or ‘bad’

(f) The qualitative factors relevant for evaluating the performance and prospects of a company are
as follows:

• Are the company’s revenues tied to one key customer?


• To what extent are the company’s revenues tied to one key product?
• To what extent does the company rely on a single supplier?
• What percentage of the company’s business is generated overseas?
• How will competition impact the company?
• What are the future prospects of the firm?
• What could be the effect of the changes in the legal and regulatory environment?
Chapter 5
FINANCIAL PLANNING AND FORECASTING

1. The proforma income statement of Modern Electronics Ltd for year 3 based on the per cent
of sales method is given below

Average per cent Proforma income statement


of sales for year 3 assuming sales of
1020

Net sales 100.0 1020.0


Cost of goods sold 76.33 778.57
Gross profit 23.67 241.43
Selling expenses 7.40 75.48
General & administration expenses 6.63 67.63
Depreciation 6.75 68.85
Operating profit 2.90 29.58
Non-operating surplus/deficit 1.07 10.91
Earnings before interest and taxes 3.96 40.39
Interest 1.24 12.65
Earnings before tax 2.72 27.74
Tax 1.00 10.20
Earnings after tax 1.72 17.54
Dividends (given) 8.00
Retained earnings 9.54
2. The proforma income statement of Modern Electronics for year 3 using the combination
method is given below:
Average per cent Proforma income statement
of sales for year 3

Net sales 100.0 1020.0


Cost of goods sold 76.33 778.57
Gross profit 23.67 241.43
Selling expenses 7.40 75.48
General & administration expenses Budgeted 55.00
Depreciation Budgeted 60.00
Operating profit 50.95
Non-operating surplus/deficit 1.07 10.91
Earnings before interest and taxes 61.86
Interest Budgeted 12.0
Earnings before tax 49.86
Tax 1.00 10.20
Earnings after tax 39.66
Dividends (given) Budgeted 8.00
Retained earnings 31.66

3. The proforma balance sheet of Modern Electronics Ltd for year 3 is given below

Average of percent Projections for year 3


of sales or some based on a forecast
other basis sales of 1400

Net sales 100.0 1020.0

ASSETS
Fixed assets (net) 40.23 410.35
Investments No change 20.00

Current assets, loans & advances :


Cash and bank 1.54 15.71
Receivables 22.49 229.40
Inventories 21.60 220.32

Prepaid expenses 5.09 51.92


Miscellaneous expenditure & losses No change 14.00

961.70
LIABILITIES:

Share capital:
Equity No change 150.00
Reserves & surplus Proforma income 160.66
statement

Secured loans:
Term loans No change 175.00
Bank borrowings No change 199.00

Current liabilities:
Trade creditors 17.33 176.77
Provisions 5.03 51.31

External funds requirement Balancing figure 48.96

961.7

A L
4. EFR = - ∆ S – m S1 (1-d)
S S

800 190
= - 300 – 0.06 x 1,300 (1-0.5)
1000 1000

= (0.61 x 300) – (0.06) x 1,300 x (0.5)

= 183 – 39 = Rs.144.
Projected Income Statement for Year Ending 31st December , 2001

Sales 1,300
Profits before tax 195
Taxes 117
Profit after tax (6% on sales) 78
Dividends 39
Retained earnings 39

Projected Balance Sheet as at 31.12 2001

Liabilities Assets

Share capital 150 Fixed assets 520


Retained earnings 219 Inventories 260
Term loans (80+72) 152 Receivables 195
Short-term bank borrowings 272 Cash 65
(200 + 72)
Accounts payable 182
Provisions 65

1,040 1,040

A L
5. (a) EFR = - ∆ S – m S1 (1 –d)
S S

150 30
= - x 80 – (0.0625) x 240 x (0.5)
160 160

= (60 – 7.5) = 52.5


(b) Projected Balance Sheet as on 31.12.20X1

Liabilities Assets

Share capital 56.25 Net fixed assets 90


Retained earnings 47.50 Inventories 75
(40 + 7.5)
Term loans 46.25 Debtors 45
Short-term bank 30.00 Cash 15
borrowings
Trade creditors 37.50
Provisions 7.50

225.00 225.00

(c) 20X0 20X1


i) Current ratio 1.50 1.80
ii) Debt to total assets ratio 0.53 0.54
iii) Return on equity 14.3% 14.5%

(d)
A L
EFR 20X1= - ∆ S – mS1 (1 – d)
S S

150 30
= - 20 – 0.0625 x 180 x 0.5
160 160

= 9.38

150 x (1.125) 30 x 1.125


EFR 20X2 = - x 20 – 0.0625 x 200 x 0.5
180 180

168.75 33.75
= - x 20 –0.0625 x 220 x 0.5
180 180

= 8.75
168.75 x (1.11) 33.75 x (1.11)
EFR 20X3 = - 20 – 0.0625 x 220 x 0.5
200 200

187.31 37.46
= - x 20 – 6.88
200 200

= 8.11

187.31 x (1.1) 37.46 x (1.1)


EFR 20X4 = - x 20 – 0.0625 x 240 x 0.5
220 220

= 7.49

Balance Sheet as on 31st December, 20X4

Liabilities Rs. Assets Rs.

Share capital 46.87 Net fixed assets 90.00


(30+16.87) (60 x 240/160)
Retained earnings Inventories
(40.00+5.63+6.25+6.88+7.50)66.26 (50x240/160) 75.00
Term loans(20+16.87) 36.87 Debtors (30x240/160) 45.00
Short-term bank borrowings 30.00 Cash (10x240/160) 15.00
Trade creditors 37.50
Provisions 7.50

225.00 225.00

6. EFR A L m (1+g) (1-d)


= - -
∆S S S g
Given A/S= 0.8 , L/S= 0.5 , m= 0.05 , d= 0.6 and EFR = 0 we have,

(0.05)(1+g)(0.4)
(0.8-0.5) - =0
g
(0.05)(1+g)(0.4)
i.e. 0.3 - =0
g
Solving the above equation we get g = 7.14%

A L
7. (a) EFR = - ∆ S – mS1 (1-d)
S S

320 70
= - x 100 – (0.05) (500) (0.5)
400 400

= Rs.50

(b) Let CA = denote Current assets


CL = Current liabilities
SCL = Spontaneous current liabilities
STL = Short-term bank borrowings
FA = Fixed assets
and LTL = Long-term loans

i. Current ratio ≥ 1.25


CA
i.e greater than or equal to 1.25 or
CL

CA
≥ 1.25
STL +SCL

As at the end of 20X1, CA = 20x0 x 1.25 = 237.50


SCL = 70 x 1.25 = 87.50
Substituting these values, we get
1.25 (STL + 87.5) ≤ 237.50
or 1.25 STL ≤ 237.50 − (8.50 x 1.25)

1 285.125
or STL =
1.25
i.e STL ≤ Rs.102.50
ii. Ratio of fixed assets to long term loans ≥ 1.25
FA
≥ 1.25
LTL
At the end of 20X1 FA = 130 x 1.25 = 162.5
162.5
∴ LTL ≤ or LTL = Rs.130
1.25
If ∆ STL and ∆ LTL denote the maximum increase in ST borrowings & LT
borrowings , we have :
∆ STL = STL (20X1) – STL (20X0) = 102.50 – 60.00 = 42.50
∆ LTL = LTL (20X1)- LTL (20X0) = 130.00 – 80.00 = 50.00
Hence, the suggested mix for raising external funds will be :
Short-term borrowings 42.50
Long-term loans 7.50
Additional equity issue --

50.00

A L
8. EFR = - ∆ S – m S1 (1-d)
S S
A S
Therefore, mS1(1-d) – - ∆ S represents surplus funds
S S
Given m= 0.06, S1 =11,000, d= 0.6 , L= 3,000 S= 10,000 and
surplus funds = 150 we have
A 3,000
(0.06) 11,000 (1-0.6) - - 1,000 = 150
10,000 10,000

A – 3,000
= (0.06) (0.4) (11,000) – 150 = 114
10

or A = (1,140 + 3,000) = 4,140

∴ The total assets of Videosonics must be 4,140

9. m= .05 , d = 0.6 , A/E = 2.5 , A/S = 1.4


m (1-d)A/E .05 (1-0.6) 2.5
(a) g= = = 3.70 per cent
A/S –m(1-d)A/E 1.4 -.05 (1-0.6) 2.5

.05 (1-0.6) x A/E


(b) 0.5 = A/E = 3.33
2.4 - .05 (1-0.6) A/E

d = 0.466
The dividend payout ratio must be reduced from 60 per cent to 46.6 per cent

.05 (1-0.6) x A/E


(c) .05 = A/E = 3.33
1.4 -.05 (1-0.6) A/E

The A/E ratio must increase from 2.5 to 3.33

m (1-0.6) 2.5
(d) .06 = m = 7.92 per cent
1.4 – m (1-0.6) x 2.5

The net profit margin must increase from 5 per cent to 7.92 per cent

.05 (1-0.6) 2.5


(e) .06 = A/S = .883
A/S - .05 (1-0.6) 2.5

The asset to sales ratio must decrease from 1.4 to 0.883

10 m= .06 , b = 0.8 , S0/A= 1/0.9 =1.11

m ( S0 /A0 ) b 0.06 x 1.11 x 0.8


g = ---------------------------- = ----- ------------------- = 0.0563
1 – m ( S0 / A0 ) b 1 – 0.06 x 1.11 x 0.8
or 5.63 percent
Chapter 6
TIME VALUE OF MONEY

1. Value five years hence of a deposit of Rs.1,000 at various interest rates is as follows:

r = 8% FV5 = 1000 x FVIF (8%, 5 years)


= 1000 x 1.469 = Rs.1469

r = 10% FV5 = 1000 x FVIF (10%, 5 years)


= 1000 x 1.611 = Rs.1611

r = 12% FV5 = 1000 x FVIF (12%, 5 years)


= 1000 x 1.762 = Rs.1762

r = 15% FV5 = 1000 x FVIF (15%, 5 years)


= 1000 x 2.011 = Rs.2011

2. Rs.160,000 / Rs. 5,000 = 32 = 25

According to the Rule of 72 at 12 percent interest rate doubling takes place dsx
approximately in 72 / 12 = 6 years

So Rs.5000 will grow to Rs.160,000 in approximately 5 x 6 years = 30 years

3. In 12 years Rs.1000 grows to Rs.8000 or 8 times. This is 23 times the initial deposit. Hence
doubling takes place in 12 / 3 = 4 years.

According to the Rule of 69, the doubling period is:

0.35 + 69 / Interest rate

Equating this to 4 and solving for interest rate, we get

Interest rate = 18.9%.

4. Saving Rs.2000 a year for 5 years and Rs.3000 a year for 10 years thereafter is equivalent
to saving Rs.2000 a year for 15 years and Rs.1000 a year for the years 6 through 15.
Hence the savings will cumulate to:
2000 x FVIFA (10%, 15 years) + 1000 x FVIFA (10%, 10 years)
= 2000 x 31.772 + 1000 x 15.937 = Rs.79481.

5. Let A be the annual savings.

A x FVIFA (12%, 10 years) = 1,000,000


A x 17.549 = 1,000,000

So, A = 1,000,000 / 17.549 = Rs.56,983.

6. 1,000 x FVIFA (r, 6 years) = 10,000

FVIFA (r, 6 years) = 10,000 / 1000 = 10

From the tables we find that

FVIFA (20%, 6 years) = 9.930


FVIFA (24%, 6 years) = 10.980

Using linear interpolation in the interval, we get:

20% + (10.000 – 9.930)


r= x 4% = 20.3%
(10.980 – 9.930)

7. 1,000 x FVIF (r, 10 years) = 5,000


FVIF (r,10 years) = 5,000 / 1000 = 5

From the tables we find that

FVIF (16%, 10 years) = 4.411


FVIF (18%, 10 years) = 5.234

Using linear interpolation in the interval, we get:

(5.000 – 4.411) x 2%
r = 16% + = 17.4%
(5.234 – 4.411)

8. The present value of Rs.10,000 receivable after 8 years for various discount rates (r ) are:
r = 10% PV = 10,000 x PVIF(r = 10%, 8 years)
= 10,000 x 0.467 = Rs.4,670

r = 12% PV = 10,000 x PVIF (r = 12%, 8 years)


= 10,000 x 0.404 = Rs.4,040

r = 15% PV = 10,000 x PVIF (r = 15%, 8 years)


= 10,000 x 0.327 = Rs.3,270

9. Assuming that it is an ordinary annuity, the present value is:


2,000 x PVIFA (10%, 5years)
= 2,000 x 3.791 = Rs.7,582

10. The present value of an annual pension of Rs.10,000 for 15 years when r = 15% is:
10,000 x PVIFA (15%, 15 years)
= 10,000 x 5.847 = Rs.58,470

The alternative is to receive a lumpsum of Rs.50,000.

Obviously, Mr. Jingo will be better off with the annual pension amount of Rs.10,000.

11. The amount that can be withdrawn annually is:


100,000 100,000
A = ------------------ ------------ = ----------- = Rs.10,608
PVIFA (10%, 30 years) 9.427

12. The present value of the income stream is:


1,000 x PVIF (12%, 1 year) + 2,500 x PVIF (12%, 2 years)
+ 5,000 x PVIFA (12%, 8 years) x PVIF(12%, 2 years)

= 1,000 x 0.893 + 2,500 x 0.797 + 5,000 x 4.968 x 0.797 = Rs.22,683.

13. The present value of the income stream is:


2,000 x PVIFA (10%, 5 years) + 3000/0.10 x PVIF (10%, 5 years)
= 2,000 x 3.791 + 3000/0.10 x 0.621
= Rs.26,212

14. To earn an annual income of Rs.5,000 beginning from the end of 15 years from now, if the
deposit earns 10% per year a sum of
Rs.5,000 / 0.10 = Rs.50,000
is required at the end of 14 years. The amount that must be deposited to get this sum is:
Rs.50,000 / FVIF (10%, 14 years) = Rs.50,000 / 3.797 = Rs.13,165

15. Rs.20,000 =- Rs.4,000 x PVIFA (r, 10 years)


PVIFA (r,10 years) = Rs.20,000 / Rs.4,000 = 5.00

From the tables we find that:


PVIFA (15%, 10 years) = 5.019
PVIFA (18%, 10 years) = 4.494
Using linear interpolation we get:
5.019 – 5.00
r = 15% + ---------------- x 3%
5.019 – 4.494

= 15.1%

16. PV (Stream A) = Rs.100 x PVIF (12%, 1 year) + Rs.200 x


PVIF (12%, 2 years) + Rs.300 x PVIF(12%, 3 years) + Rs.400 x
PVIF (12%, 4 years) + Rs.500 x PVIF (12%, 5 years) +
Rs.600 x PVIF (12%, 6 years) + Rs.700 x PVIF (12%, 7 years) +
Rs.800 x PVIF (12%, 8 years) + Rs.900 x PVIF (12%, 9 years) +
Rs.1,000 x PVIF (12%, 10 years)

= Rs.100 x 0.893 + Rs.200 x 0.797 + Rs.300 x 0.712


+ Rs.400 x 0.636 + Rs.500 x 0.567 + Rs.600 x 0.507
+ Rs.700 x 0.452 + Rs.800 x 0.404 + Rs.900 x 0.361
+ Rs.1,000 x 0.322

= Rs.2590.9

Similarly,
PV (Stream B) = Rs.3,625.2
PV (Stream C) = Rs.2,851.1

17. FV5 = Rs.10,000 [1 + (0.16 / 4)]5x4


= Rs.10,000 (1.04)20
= Rs.10,000 x 2.191
= Rs.21,910

18. FV5 = Rs.5,000 [1+( 0.12/4)] 5x4


= Rs.5,000 (1.03)20
= Rs.5,000 x 1.806
= Rs.9,030
19 A B C

Stated rate (%) 12 24 24

Frequency of compounding 6 times 4 times 12 times

Effective rate (%) (1 + 0.12/6)6- 1 (1+0.24/4)4 –1 (1 + 0.24/12)12-1

= 12.6 = 26.2 = 26.8

Difference between the


effective rate and stated
rate (%) 0.6 2.2 2.8

20. Investment required at the end of 8th year to yield an income of Rs.12,000 per year from
the end of 9th year (beginning of 10th year) for ever:

Rs.12,000 x PVIFA(12%, ∞ )
= Rs.12,000 / 0.12 = Rs.100,000

To have a sum of Rs.100,000 at the end of 8th year , the amount to be deposited now is:
Rs.100,000 Rs.100,000
= = Rs.40,388
PVIF(12%, 8 years) 2.476

21. The interest rate implicit in the offer of Rs.20,000 after 10 years in lieu of Rs.5,000 now
is:
Rs.5,000 x FVIF (r,10 years) = Rs.20,000

Rs.20,000
FVIF (r,10 years) = = 4.000
Rs.5,000

From the tables we find that


FVIF (15%, 10 years) = 4.046

This means that the implied interest rate is nearly 15%.


I would choose Rs.20,000 after 10 years from now because I find a return of 15%
quite acceptable.
22. FV10 = Rs.10,000 [1 + (0.10 / 2)]10x2
= Rs.10,000 (1.05)20
= Rs.10,000 x 2.653
= Rs.26,530

If the inflation rate is 8% per year, the value of Rs.26,530 10 years from now, in terms of
the current rupees is:
Rs.26,530 x PVIF (8%,10 years)
= Rs.26,530 x 0.463 = Rs.12,283

23. A constant deposit at the beginning of each year represents an annuity due.
PVIFA of an annuity due is equal to : PVIFA of an ordinary annuity x (1 + r)
To provide a sum of Rs.50,000 at the end of 10 years the annual deposit should
be

Rs.50,000
A = FVIFA(12%, 10 years) x (1.12)

Rs.50,000
= = Rs.2544
17.549 x 1.12

24. The discounted value of Rs.20,000 receivable at the beginning of each year from 2005 to
2009, evaluated as at the beginning of 2004 (or end of 2003) is:
Rs.20,000 x PVIFA (12%, 5 years)
= Rs.20,000 x 3.605 = Rs.72,100.

The discounted value of Rs.72,100 evaluated at the end of 2000 is


Rs.72,100 x PVIF (12%, 3 years)
= Rs.72,100 x 0.712 = Rs.51,335

If A is the amount deposited at the end of each year from 1995 to 2000 then
A x FVIFA (12%, 6 years) = Rs.51,335
A x 8.115 = Rs.51,335
A = Rs.51,335 / 8.115 = Rs.6326

25. The discounted value of the annuity of Rs.2000 receivable for 30 years, evaluated as at the
end of 9th year is:
Rs.2,000 x PVIFA (10%, 30 years) = Rs.2,000 x 9.427 = Rs.18,854

The present value of Rs.18,854 is:


Rs.18,854 x PVIF (10%, 9 years)
= Rs.18,854 x 0.424
= Rs.7,994
26. 30 per cent of the pension amount is
0.30 x Rs.600 = Rs.180

Assuming that the monthly interest rate corresponding to an annual interest rate of 12% is
1%, the discounted value of an annuity of Rs.180 receivable at the end of each month for 180
months (15 years) is:
Rs.180 x PVIFA (1%, 180)

(1.01)180 - 1
Rs.180 x ---------------- = Rs.14,998
.01 (1.01)180
If Mr. Ramesh borrows Rs.P today on which the monthly interest rate is 1%

P x (1.01)60 = Rs.14,998
P x 1.817 = Rs.14,998

Rs.14,998
P = ------------ = Rs.8254
1.817

27. Rs.300 x PVIFA(r, 24 months) = Rs.6,000


PVIFA (r,24) = Rs.6000 / Rs.300 = 20

From the tables we find that:


PVIFA(1%,24) = 21.244
PVIFA (2%, 24) = 18.914

Using a linear interpolation


21.244 – 20.000
r = 1% + ---------------------- x 1%
21.244 – 18,914

= 1.53%

Thus, the bank charges an interest rate of 1.53% per month.


The corresponding effective rate of interest per annum is
[ (1.0153)12 – 1 ] x 100 = 20%

28. The discounted value of the debentures to be redeemed between 8 to 10 years evaluated at
the end of the 5th year is:
Rs.10 million x PVIF (8%, 3 years)
+ Rs.10 million x PVIF (8%, 4 years)
+ Rs.10 million x PVIF (8%, 5 years)
= Rs.10 million (0.794 + 0.735 + 0.681)
= Rs.2.21 million

If A is the annual deposit to be made in the sinking fund for the years 1 to 5,
then
A x FVIFA (8%, 5 years) = Rs.2.21 million
A x 5.867 = Rs.2.21 million
A = 5.867 = Rs.2.21 million
A = Rs.2.21 million / 5.867 = Rs.0.377 million

29. Let `n’ be the number of years for which a sum of Rs.20,000 can be withdrawn annually.

Rs.20,000 x PVIFA (10%, n) = Rs.100,000


PVIFA (10 %, n) = Rs.100,000 / Rs.20,000 = 5.000

From the tables we find that


PVIFA (10%, 7 years) = 4.868
PVIFA (10%, 8 years) = 5.335

Thus n is between 7 and 8. Using a linear interpolation we get

5.000 – 4.868
n=7+ ----------------- x 1 = 7.3 years
5.335 – 4.868

30. Equated annual installment = 500000 / PVIFA(14%,4)


= 500000 / 2.914
= Rs.171,585

Loan Amortisation Schedule

Beginning Annual Principal Remaining


Year amount installment Interest repaid balance
------ ------------- --------------- ----------- ------------- -------------
1 500000 171585 70000 101585 398415
2 398415 171585 55778 115807 282608
3 282608 171585 39565 132020 150588
4 150588 171585 21082 150503 85*

(*) rounding off error

31. Define n as the maturity period of the loan. The value of n can be obtained from the
equation.
200,000 x PVIFA(13%, n) = 1,500,000
PVIFA (13%, n) = 7.500

From the tables or otherwise it can be verified that PVIFA(13,30) = 7.500


Hence the maturity period of the loan is 30 years.

32. Expected value of iron ore mined during year 1 = Rs.300 million

Expected present value of the iron ore that can be mined over the next 15 years
assuming a price escalation of 6% per annum in the price per tonne of iron

1 – (1 + g)n / (1 + i)n
= Rs.300 million x ------------------------
i-g

= Rs.300 million x 1 – (1.06)15 / (1.16)15


0.16 – 0.06

= Rs.300 million x (0.74135 / 0.10)


= Rs.2224 million
33 (a) PV = Rs.500,000
(b) PV = 1,000,000PVIF10%,6yrs = 1,000,000 x 0.564 = Rs.564,000
(c ) PV = 60,000/r = 60,000/0.10 = Rs.600,000
(d) PV = 100,000 PVIFA10%,10yrs = 100,000 x 6.145 = Rs.614,500
(e) PV = C/(r-g) = 35,000/(0.10-0.05) = Rs.700,000
Option e has the highest present value viz. Rs.700,000

34. (a) PV = c/(r – g) = 12/[0.12 – (-0.03)] = Rs.80 million

1+g n
1 - -------
(b) 1+r
PV = A(1+g) ----------------- = 12 x 0.97 x 0.9725 / 0.15 = Rs.75.466 million
r- g

35. It may be noted that if g1 is the growth rate in the no. of units and g2 the growth rate in price
per unit, then the growth rate of their product, g = (1+g1)(1+g2) - 1
In this problem the growth rate in the value of oil produced, g = (1- 0.05)(1 +0.03) - 1 = -
0.0215
Present value of the well’s production =

1+g n
1 - -------
1+r
PV = A(1+g) -----------------
r- g

= (50,000 x 50) x ( 1-0.0215)x 1 – (0.9785 / 1.10)15


0.10 + 0.0215

= $ 16,654,633
36.
The growth rate in the value of the oil production g = (1- 0.06)(1 +0.04) - 1
= - 0.0224

Present value of the well’s production =

1+g n
1 - -------
1+r
PV = A(1+g) -----------------
r- g

= (80,000 x 60) x ( 1-0.0224)x 1 – (0.9776 / 1.12)20


0.12 + 0.0224

= $ 30,781,328.93

37. Future Value Interest Factor for Growing Annuity,


( 1+ i )n – ( 1 + g)n
FVIFGA =
i-g
(1. 09)20 – ( 1.08)20
So the value of the savings at the end of 20 years = 100,000 x
0.09 – 0.08

= Rs. 9,434,536

38
Assuming 52 weeks in an year, the effective interest rate is
52
0.08
1 + - 1 = 1.0832 - 1 = 8.32 percent
52

MINICASE

Solution:

1. How much money would Ramesh need 15 years from now?

500,000 x PVIFA (10%, 15years)


+ 1,000,000 x PVIF (10%, 15years)
= 500,000 x 7.606 + 1,000,000 x 0.239
= 3,803,000 x 239,000
= Rs.4,042,000

2. How much money should Ramesh save each year for the next 15 years to be able to meet
his investment objective?

Ramesh’s current capital of Rs.600,000 will grow to :

600,000 (1.10)15 = 600,000 x 4.177 = Rs 2,506,200

This means that his savings in the next 15 years must grow to :

4,042,000 – 2,506,200 = Rs 1,535,800

So, the annual savings must be :


1,535,800 1,535,800
= = Rs.48,338
FVIFA (10%, 15 years) 31.772

3. How much money would Ramesh need when he reaches the age of 60 to meet his donation
objective?

200,000 x PVIFA (10% , 3yrs) x PVIF (10%, 11yrs)

= 200,000 x 2.487 x 0.317 = 157,676

4. What is the present value of Ramesh’s life time earnings?


400,000 400,000(1.12) 400,000(1.12)14

46
1 2 15

15
1.12
1–
1.08
= 400,000
0.08 – 0.12

= Rs.7,254,962
Chapter 7

VALUATION OF BONDS AND STOCKS

1. 5 11 100
P = ∑ +
t=1 (1.15)t (1.15)5

= Rs.11 x PVIFA(15%, 5 years) + Rs.100 x PVIF (15%, 5 years)


= Rs.11 x 3.352 + Rs.100 x 0.497
= Rs.86.7

2.(i) When the discount rate is 14%


7 12 100
P = ∑ +
t
t=1 (1.14) (1.14)7

= Rs.12 x PVIFA (14%, 7 years) + Rs.100 x PVIF (14%, 7 years)


= Rs.12 x 4.288 + Rs.100 x 0.4
= Rs.91.46

(ii) When the discount rate is 12%


7 12 100
P = ∑ + = Rs.100
t=1 (1.12) t (1.12)7

Note that when the discount rate and the coupon rate are the same the value is equal to
par value.

3. The yield to maturity is the value of r that satisfies the following equality.
7 120 1,000
Rs.750 = ∑ +
t=1 (1+r) t (1+r)7

Try r = 18%. The right hand side (RHS) of the above equation is:
Rs.120 x PVIFA (18%, 7 years) + Rs.1,000 x PVIF (18%, 7 years)
= Rs.120 x 3.812 + Rs.1,000 x 0.314
= Rs.771.44

Try r = 20%. The right hand side (RHS) of the above equation is:
Rs.120 x PVIFA (20%, 7 years) + Rs.1,000 x PVIF (20%, 7 years)
= Rs.120 x 3.605 + Rs.1,000 x 0.279
= Rs.711.60
Thus the value of r at which the RHS becomes equal to Rs.750 lies between 18% and 20%.

Using linear interpolation in this range, we get

771.44 – 750.00
Yield to maturity = 18% + 771.44 – 711.60 x 2%

= 18.7%

4.
10 14 100
80 = ∑ +
t=1 (1+r) t (1+r)10

Try r = 18%. The RHS of the above equation is

Rs.14 x PVIFA (18%, 10 years) + Rs.100 x PVIF (18%, 10 years)


= Rs.14 x 4.494 + Rs.100 x 0.191 = Rs.82

Try r = 20%. The RHS of the above equation is


Rs.14 x PVIFA(20%, 10 years) + Rs.100 x PVIF (20%, 10 years)
= Rs.14 x 4.193 + Rs.100 x 0.162
= Rs.74.9

Using interpolation in the range 18% and 20% we get:

82 - 80
Yield to maturity = 18% + ----------- x 2%
82 – 74.9

= 18.56%

5.
12 6 100
P = ∑ +
t=1 (1.08) t (1.08)12

= Rs.6 x PVIFA (8%, 12 years) + Rs.100 x PVIF (8%, 12 years)


= Rs.6 x 7.536 + Rs.100 x 0.397
= Rs.84.92
6. The post-tax interest and maturity value are calculated below:

Bond A Bond B

* Post-tax interest (C ) 12(1 – 0.3) 10 (1 – 0.3)


=Rs.8.4 =Rs.7

* Post-tax maturity value (M) 100 - 100 -


[ (100-70)x 0.1] [ (100 – 60)x 0.1]
=Rs.97 =Rs.96

The post-tax YTM, using the approximate YTM formula is calculated below

8.4 + (97-70)/10
Bond A : Post-tax YTM = --------------------
0.6 x 70 + 0.4 x 97

= 13.73%

7 + (96 – 60)/6
Bond B : Post-tax YTM = ----------------------
0.6x 60 + 0.4 x 96

= 17. 47%

7.
14 6 100
P = ∑ +
t=1 (1.08) t (1.08)14

= Rs.6 x PVIFA(8%, 14) + Rs.100 x PVIF (8%, 14)


= Rs.6 x 8.244 + Rs.100 x 0.341
= Rs.83.56

8. Do = Rs.2.00, g = 0.06, r = 0.12

Po = D1 / (r – g) = Do (1 + g) / (r – g)

= Rs.2.00 (1.06) / (0.12 - 0.06)


= Rs.35.33

Since the growth rate of 6% applies to dividends as well as market price, the market
price at the end of the 2nd year will be:
P2 = Po x (1 + g)2 = Rs.35.33 (1.06)2
= Rs.39.70

9. Po = D1 / (r – g) = Do (1 + g) / (r – g)
= Rs.12.00 (1.10) / (0.15 – 0.10) = Rs.264

10. Po = D1 / (r – g)

Rs.32 = Rs.2 / (0.12 – g)


g = 0.0575 or 5.75%

11. Po = D1/ (r – g) = Do(1+g) / (r – g)


Do = Rs.1.50, g = -0.04, Po = Rs.8
So
8 = 1.50 (1- .04) / (r-(-.04)) = 1.44 / (r + .04)

Hence r = 0.14 or 14 per cent

12. The market price per share of Commonwealth Corporation will be the sum of three
components:

A: Present value of the dividend stream for the first 4 years


B: Present value of the dividend stream for the next 4 years
C: Present value of the market price expected at the end of 8 years.

A= 1.50 (1.12) / (1.14) + 1.50 (1.12)2 / (1.14)2 + 1.50(1.12)3 / (1.14)3 +


+ 1.50 (1.12)4 / (1.14)4

= 1.68/(1.14) + 1.88 / (1.14)2 + 2.11 / (1.14)3 + 2.36 / (1.14)4


= Rs.5.74

B= 2.36(1.08) / (1.14)5 + 2.36 (1.08)2 / (1.14)6 + 2.36 (1.08)3 / (1.14)7 +


+ 2.36 (1.08)4 / (1.14)8

= 2.55 / (1.14)5 + 2.75 / (1.14)6 + 2.97 / (1.14)7 + 3.21 / (1.14)8


= Rs.4.89

C = P8 / (1.14)8

P8 = D9 / (r – g) = 3.21 (1.05)/ (0.14 – 0.05) = Rs.37.45


So

C = Rs.37.45 / (1.14)8 = Rs.13.14


Thus,
Po = A + B + C = 5.74 + 4.89 + 13.14
= Rs.23.77

13. Let us assume a required rate of return of 12 percent. The intrinsic value of the equity share
will be the sum of three components:

A: Present value of the dividend stream for the first 5 years when the
growth rate expected is 15%.

B: Present value of the dividend stream for the next 5 years when the
growth rate is expected to be 10%.

C: Present value of the market price expected at the end of 10 years.

2.00 (1.15) 2.00 (1.15)2 2.00 (1.15)3 2.00(1.15)4 2.00 (1.15)5


A= ------------- + ------------- +-------------- + ------------- + -------------
(1.12) (1.12)2 (1.1.2)3 (1.1.2)4 (1.12)5

= 2.30 / (1.12) + 2.65 / (1.12)2 + 3.04 / (1.12)3 + 3.50 / (1.12)4 + 4.02/(1.12)5


= Rs.10.84

4.02(1.10) 4.02 (1.10)2 4.02(1.10)3 4.02(1.10)4 4.02 (1.10)5


B= ------------ + ---------------- + ------------- + --------------- + ---------------
(1.12)6 (1.12)7 (1.12)8 (1..12)9 (1.12)10
4.42 4.86 5.35 5.89 6.48
= --------- + -------------- + --------------- + ------------- + -------------
(1.12)6 (1.12)7 (1.12)8 (1.1.2)9 (1.12)10

= Rs.10.81

D11 1 6.48 (1.05)


C= -------- x --------------- = ------------------- x 1/(1.12)10
r–g (1 +r)10 0.12 – 0.05

= Rs.97.20

The intrinsic value of the share = A + B + C


= 10.84 + 10.81 + 97.20 = Rs.118.85

14. Terminal value of the interest proceeds


= 140 x FVIFA (16%,4)
= 140 x 5.066
= 709.24
Redemption value = 1,000

Terminal value of the proceeds from the bond = 1709.24

Define r as the yield to maturity. The value of r can be obtained from the equation

900 (1 + r)4 = 1709.24


r = 0.1739 or 17.39%

15. Intrinsic value of the equity share (using the 2-stage growth model)

(1.18)6
2.36 x 1 - ----------- 2.36 x (1.18)5 x (1.12)
6
(1.16)
= --------------------------------- + -----------------------------------
0.16 – 0.18 (0.16 – 0.12) x (1.16)6

- 0.10801
= 2.36 x ----------- + 62.05
- 0.02

= Rs.74.80

16. Intrinsic value of the equity share (using the H model)

4.00 (1.20) 4.00 x 4 x (0.10)


= -------------- + ---------------------
0.18 – 0.10 0.18 – 0.10

= 60 + 20
= Rs.80
17.

Po = D1
r–g

Po
Rs. 8 = Rs. 266.7
=
0.15-0.12

Po = E1 + PVGO
r
Po = Rs. 20 + PVGO
0.15
Rs. 266.7 = Rs. 133.3 + PVGO

So, PVGO = Rs. 133.4

MINICASE

(a) The value of a bond is calculated using the formula


n C M
P = ∑ +
t=1 (1+r)t (1+r)n
where P is the value (in rupees), n is the number of years to maturity, C is the annual
coupon payment (in rupees), r is the periodic required return, M is the maturity value, and t is
the time when the payment is received

(b) Value of the bond = 100 PVIFA8% , 5years + 1000 PVIF8% , 5years
= 100 x 3.993 + 1000 x 0.681 = Rs.1080.30

100 + ( 1000 – 1060)/8


(c) Approximate YTM = = 8.93%
0.4 x 1000 + 0.6 x 1060

(d) 100 + ( 1050 – 1060)/2


Approximate YTC = = 9%
0.4 x 1050 + 0.6 x 1060

(e) The general formula for valuing any stock is :


∞ Dt
P0 = ∑

t=1 (1+r)t

(f) A constant growth stock is valued using the formula

D1
P0 =
r-g

where D1 is the dividend expected a year hence, r is the required rate of return and g is the
constant growth rate
(g)
(i) The expected value of the stock a year from now
D2 6 x (1+0.12)2
P1 = = = Rs.250.88
r- g 0.15 – 0.12
6 x 1.12
(ii) Price of the stock at present, P0 = = Rs.224
0.15 – 0.12

Expected dividend in the first year = 6 x1.12 = Rs.6.72


6.72
Dividend yield = x 100 = 3 %
224
Expected capital gains in the first year = P1 –P0 = 250.88 – 224 = Rs.26.88
26.88
Capital gains yield = x 100 = 12 %
224

(h) Present value of the stock is :

n
1+g1
1 -
1+r D1 (1+g1)n-1 (1+g2) 1
P0 = D1 +
r - g1 r - g2 (1+r)n

4
1.25
1 - 1.16 (10 x 1.25) x (1.25)3 x 1.10 1
= (10 x 1.25) + x
0.16 – 0.10 (1.16)4
0.16 – 0.25

= 48.38 + 447.59 /1.81 = Rs. 295.67


(i) Intrinsic value per share:

D0 [(1+gn) + H (ga-gn)]
P0 =
r - gn

8[ 1.10 + 1.5 x (0.20 – 0.10]


= = Rs. 250
0.14 – 0.10

-----------------------------------------------------------------------------------------------------------------------
Chapter 8
RISK AND RETURN

1 (a) Expected price per share a year hence will be:

= 0.4 x Rs.10 + 0.4 x Rs.11 + 0.2 x Rs.12 = Rs.10.80

(b) Probability distribution of the rate of return is

Rate of return (Ri) 10% 20% 30%

Probability (pi) 0.4 0.4 0.2

Note that the rate of return is defined as:

Dividend + Terminal price


-------------------------------- - 1
Initial price

2 (a) For Rs.1,000, 20 shares of Alpha’s stock can be acquired. The probability distribution of
the return on 20 shares is

Economic Condition Return (Rs) Probability


High Growth 20 x 55 = 1,100 0.3
Low Growth 20 x 50 = 1,000 0.3
Stagnation 20 x 60 = 1,200 0.2
Recession 20 x 70 = 1,400 0.2

Expected return = (1,100 x 0.3) + (1,000 x 0.3) + (1,200 x 0.2) + (1,400 x 0.2)

= 330 + 300 + 240 + 280


= Rs.1,150

Standard deviation of the return = [(1,100 – 1,150)2 x 0.3 + (1,000 – 1,150)2 x

0.3 + (1,200 – 1,150)2 x 0.2 + (1,400 – 1,150)2 x 0.2]1/2


= Rs.143.18

(b) For Rs.1,000, 20 shares of Beta’s stock can be acquired. The probability distribution of the
return on 20 shares is:
Economic condition Return (Rs) Probability

High growth 20 x 75 = 1,500 0.3


Low growth 20 x 65 = 1,300 0.3
Stagnation 20 x 50 = 1,000 0.2
Recession 20 x 40 = 800 0.2

Expected return = (1,500 x 0.3) + (1,300 x 0.3) + (1,000 x 0.2) + (800 x 0.2)
= Rs.1,200

Standard deviation of the return = [(1,500 – 1,200)2 x .3 + (1,300 – 1,200)2 x .3


+ (1,000 – 1,200)2 x .2 + (800 – 1,200)2 x .2]1/2 = Rs.264.58

(c ) For Rs.500, 10 shares of Alpha’s stock can be acquired; likewise for Rs.500, 10
shares of Beta’s stock can be acquired. The probability distribution of this option is:
Return (Rs) Probability
(10 x 55) + (10 x 75) = 1,300 0.3
(10 x 50) + (10 x 65) = 1,150 0.3
(10 x 60) + (10 x 50) = 1,100 0.2
(10 x 70) + (10 x 40) = 1,100 0.2

Expected return = (1,300 x 0.3) + (1,150 x 0.3) + (1,100 x 0.2) +


(1,100 x 0.2)
= Rs.1,175
Standard deviation = [(1,300 –1,175)2 x 0.3 + (1,150 – 1,175)2 x 0.3 +

(1,100 – 1,175)2 x 0.2 + (1,100 – 1,175)2 x 0.2 ]1/2


= Rs.84.41
d. For Rs.700, 14 shares of Alpha’s stock can be acquired; likewise for Rs.300, 6
shares of Beta’s stock can be acquired. The probability distribution of this
option is:

Return (Rs) Probability

(14 x 55) + (6 x 75) = 1,220 0.3


(14 x 50) + (6 x 65) = 1,090 0.3
(14 x 60) + (6 x 50) = 1,140 0.2
(14 x 70) + (6 x 40) = 1,220 0.2

Expected return = (1,220 x 0.3) + (1,090 x 0.3) + (1,140 x 0.2) + (1,220 x 0.2)
= Rs.1,165
Standard deviation = [(1,220 – 1,165)2 x 0.3 + (1,090 – 1,165)2 x 0.3 +
(1,140 – 1,165)2 x 0.2 + (1,220 – 1,165)2 x 0.2]1/2
= Rs.57.66

The expected return to standard deviation of various options are as follows :


Expected return Standard deviation Expected / Standard
Option (Rs) (Rs) return deviation
a 1,150 143 8.04
b 1,200 265 4.53
c 1,175 84 13.99
d 1,165 58 20.09

Option `d’ is the most preferred option because it has the highest return to risk ratio.

3.(a) Define RA and RM as the returns on the equity stock of Auto Electricals Limited a and
Market portfolio respectively. The calculations relevant for calculating the beta of the
stock are shown below:

Year RA RM RA-RA RM-RM (RA-RA) (RM-RM) RA-RA/RM-RM


1 15 12 -0.09 -3.18 0.01 10.11 0.29
2 -6 1 -21.09 -14.18 444.79 201.07 299.06
3 18 14 2.91 -1.18 8.47 1.39 -3.43
4 30 24 14.91 8.82 222.31 77.79 131.51
5 12 16 0-3.09 0.82 9.55 0.67 -2.53
6 25 30 9.91 14.82 98.21 219.63 146.87
7 2 -3 -13.09 -18.18 171.35 330.51 237.98
8 20 24 4.91 8.82 24.11 77.79 43.31
9 18 15 2.91 -0.18 8.47 0.03 -0.52
10 24 22 8.91 6.82 79.39 46.51 60.77
11 8. 12 -7.09 -3.18 50.27 10.11 22.55

RA = 15.09 RM = 15.18

∑ (RA – RA)2 = 1116.93 ∑ (RM – RM) 2 = 975.61 ∑ (RA – RA) (RM – RM) = 935.86

Beta of the equity stock of Auto Electricals


∑ (RA – RA) (RM – RM)

∑ (RM – RM) 2
= 935.86 = 0.96
975.61
(b)
Alpha = R A – βA R M

= 15.09 – (0.96 x 15.18)= 0.52

Equation of the characteristic line is

RA = 0.52 + 0.96 RM

4 The required rate of return on stock A is:

RA = RF + βA (RM – RF)
= 0.10 + 1.5 (0.15 – 0.10)
= 0.175

Intrinsic value of share = D1 / (r- g) = Do (1+g) / ( r – g)

Given Do = Rs.2.00, g = 0.08, r = 0.175


2.00 (1.08)
Intrinsic value per share of stock A =
0.175 – 0.08

= Rs.22.74

5. The SML equation is RA = RF + βA (RM – RF)

Given RA = 15%. RF = 8%, RM = 12%, we have

0.15 = .08 + βA (0.12 – 0.08)

0.07
i.e.βA = = 1.75
0.04

Beta of stock A = 1.75

6. The SML equation is: RX = RF + βX (RM – RF)

We are given 0.15 = 0.09 + 1.5 (RM – 0.09) i.e., 1.5 RM = 0.195
or RM = 0.13%

Therefore return on market portfolio = 13%


7. RM = 12% βX = 2.0 RX =18% g = 5% Po = Rs.30

Po = D1 / (r - g)

Rs.30 = D1 / (0.18 - .05)

So D1 = Rs.39 and Do = D1 / (1+g) = 3.9 /(1.05) = Rs.3.71

Rx = Rf + βx (RM – Rf)

0.18 = Rf + 2.0 (0.12 – Rf)

So Rf = 0.06 or 6%.

Original Revised

Rf 6% 8%
RM – Rf 6% 4%
g 5% 4%
βx 2.0 1.8

Revised Rx = 8% + 1.8 (4%) = 15.2%

Price per share of stock X, given the above changes is

3.71 (1.04)
= Rs.34.45
0.152 – 0.04
8

We know that:

Debt (1-tc)
β equity = β assets 1 +
Equity
i.e
β equity 1.1
β assets = = = 0.71
Debt(1-tc) 4
1 + ----------- 1 + --- ( 1 – 0.30)
Equity 5
9

Period RB(%) RM(%) RB-R B RM-R M (R B-R B)(R M-R M) (R M-R M)2
1 15 12 1.2 0.2 0.24 0.04
2 25 20 11.2 8.2 91.84 67.24
3 -10 -5 -23.8 -16.8 399.84 282.24
4 20 24 6.2 12.2 75.64 148.84
5 15 10 1.2 -1.8 -2.16 3.24
6 30 25 16.2 13.2 213.84 174.24
7 -5 -10 -18.8 -21.8 409.84 475.24
8 20 15 6.2 3.2 19.84 10.24
9 16 17 2.2 5.2 11.44 27.04
10 12 10 -1.8 -1.8 3.24 3.24

∑RB=138 ∑RM=118 ∑(RB-RB)(RM-RM)=1223.6 ∑(RM-RM)2=1191.6


RB=13.8 RM= 11.8

2
∑( RM-RM) ∑ (RB-RB)(RM-RM)
2
σ Μ = = 132.4 CovB,M = = 1 36
n-1 n-1

CovB,M 136
Beta :β Β = =
= 1 .03
2
σ Μ 1 32.4
Alpha = α B = RB - β Β RM = 13.8 – 1.03 x 11.8 = 1.65%
The characteristic line for stock B is : RB = 1.65 + 1.03 RM
Chapter 9
RISK AND RETURN: PORTFOLIO THEORY AND ASSET PRICING MODELS
1. (a)
E (R1) = 0.2(-5%) + 0.3(15%) + 0.4(18%) + .10(22%)
= 12%
E (R2) = 0.2(10%) + 0.3(12%) + 0.4(14%) + .10(18%)
= 13%
σ(R1) = [.2(-5 –12)2 + 0.3 (15 –12)2 + 0.4 (18 –12)2 + 0.1 (22 – 12)2]½
= [57.8 + 2.7 + 14.4 + 10]½ = 9.21%
σ(R2) = [.2(10 –13)2 + 0.3(12 – 13)2 + 0.4 (14 – 13)2 + 0.1 (18 – 13)2] ½
= [1.8 + 0.09 + 0.16 + 2.5] ½ = 2.13%

(b)The covariance between the returns on assets 1 and 2 is calculated below


State of Probability Return Deviation Return on Deviation Product of
nature on asset of return asset 2 of the deviation
1 on asset 1 return on times
from its asset 2 probability
mean from its
mean
(1) (2) (3) (4) (5) (6) (2)x(4)x(6)
1 0.2 -5% -17% 10% -3% 10.2
2 0.3 15% 3% 12% -1% -0.9%
3 0.4 18% 6% 14% 1% 2.4
4 0.1 22% 10% 18% 5% 5
Sum = 16.7

Thus the covariance between the returns of the two assets is 16.7.

(c) The coefficient of correlation between the returns on assets 1 and 2 is:
Covariance12 16.7
= = 0.85
σ1 x σ2 9.21 x 2.13
2. Expected rates of returns on equity stock A, B, C and D can be computed as follows:

A: 0.10 + 0.12 + (-0.08) + 0.15 + (-0.02) + 0.20 = 0.0783 = 7.83%


6

B: 0.08 + 0.04 + 0.15 +.12 + 0.10 + 0.06 = 0.0917 = 9.17%


6

C: 0.07 + 0.08 + 0.12 + 0.09 + 0.06 + 0.12 = 0.0900 = 9.00%

D: 0.09 + 0.09 + 0.11 + 0.04 + 0.08 + 0.16 = 0.095 = 9.50%


6

(a) Return on portfolio consisting of stock A = 7.83%

(b) Return on portfolio consisting of stock A and B in equal


proportions = 0.5 (0.0783) + 0.5 (0.0917)
= 0.085 = 8.5%

(c ) Return on portfolio consisting of stocks A, B and C in equal


proportions = 1/3(0.0783 ) + 1/3(0.0917) + 1/3 (0.090)
= 0.0867 = 8.67%

(d) Return on portfolio consisting of stocks A, B, C and D in equal


proportions = 0.25(0.0783) + 0.25(0.0917) + 0.25(0.0900) +
0.25(0.095)
= 0.08875 = 8.88%

3. The standard deviation of portfolio return is:

σ p= [w12σ 12 + w22σ 22 + w32σ 32 + σ 42σ 42 + 2 w1 w2 ρ 12 σ 1 σ 2 + 2 w1 w3 ρ 13 σ 1 σ 3 + 2 w1 w4


ρ 14 σ 1σ 4 + 2 w2 w3 ρ 23 σ 2 σ 3 + 2 w2 w4 ρ 24 σ 2 σ 4 + 2 w3 w4 ρ 34 σ 3 σ 4 ]1/2
= [0.22 x 42 + 0.32 x 82 + 0.42 x 202 + 0.12 x 102 + 2 x 0.2 x 0.3 x 0.3 x 4 x 8
+ 2 x 0.2 x 0.4 x 0.5 x 4 x 20 + 2 x 0.2 x 0.1 x 0.2 x 4 x 10
+ 2 x 0.3 x 0.4 x 0.6 x 8 x 20 + 2 x 0.3 x 0.1 x 0.8 x 8 x 10
+ 2 x 0.4 x 0.1 x 0.4 x 20 x 10]1/2
= 10.6%

MINICASE
a. For stock A:

Expected return = (0.2 x -15) + (0.5 x 20) + (0.3 x 40) = 19

Standard deviation = [ 0.2 ( -15 -19)2 + 0.5 (20-19)2 + 0.3 (40 – 19)2 ] 1/2
= [231.2 + 0.5 + 132.3]1/2 = 19.08

For stock B:

Expected return = (0.2 x 30) + (0.5 x 5) + [ 0.3 x (-) 15] = 4


Standard deviation = [0.2 ( 30 – 4)2 + 0.5 (5 -4)2 + 0.3 (-15–4)2]1/2

= (135.2 + 0.5 + 108.3) ½ = 15.62


For stock C:

Expected return = [0.2 x (-5)] + (0.5 x 15) + (0.3 x 25)] = 14


Standard deviation = [0.2 (-5 – 14)2 + 0.5 (15 -14)2 + 0.3 (25-14)2] ½
= [72.2 + 0.5 + 36.3] ½ = 10.44

For market portfolio:

Expected return = [0.2 x (-)10] + (0.5 x 16) + (0.3 x 30) = -2 + 8 + 9 = 15

Standard deviation = [0.2 (-10-15)2 + 0.5(16-15)2 + 0.3 (30 – 15)2] ½

= ( 125 + 0.5 + 67.5 ) ½ = 13.89

b.

State of the Prob- Return on Return RA-E(RA) RB-E(RB) p


Economy ability (p) A (%) (RA) B (%) (RB) x [RA-E(RA)]
x[RB-E(RB)]

Recession 0.2 -15 30 -34 26 -176.8


Normal 0.5 20 5 1 1 0.5
Boom 0.3 40 -15 21 -19 - 119.7
total = - 296.00

Covariance between the returns of A and B is (-) 296

State of the Prob- Return on A Return C RA-E(RA) RC-E(RC) p


Economy ability (p) (%) (RA) (%) (RC) x [RA-E(RA)]
x[RC-E(RC)]

Recession 0.2 -15 - 5.0 -34 -19 129.2


Normal 0.5 20 15.0 1. 1 0.5
Boom 0.3 40 25.0 21 11 69.3
total = 199.0

Covariance between the returns of A and C is 199


(-) 296
c. Coefficient of correlation between the returns of A and B = = (-) 1
19.08 x 15.62

199
Coefficient of correlaton between the returns of A and C = = 1
19.08 x 10.44

d. Portfolio in which stocks A and B are equally weighted:

Economic condition Probability Overall expected return


Recession 0.2 0.5 x (-) 15 + 0.5 x 30 = 7.5
Normal 0.5 0.5 x 20 + 0.5 x 5 = 12.5
Boom 0.3 0.5 x 40 + 0.5 x (-)15 = 12.5

Expected return of the portfolio = (0.2 x 7.5) + (0.5 x 12.5) + (0.3 x 12.5)
= 0.7 + 6.25 + 4.5 = 11.5

Standard deviation of the portfolio


= [ 0.2 (7.5 – 11.5)2 + 0.5 (12.5 – 11.5)2 + 0.3 (12.5 – 11.5)2]1/2

= [ 3.2 + 0.5 + 0.3] ½ = 2

Portfolio in which weights assigned to stocks A, B and C are 0.4, 0.4 and 0.2 respectively.

Expected return of the portfolio = (0.4 x 19.0) + (0.4 x 4) + (0.2 x 14))


= 12

For calculating the standard deviation of the portfolio we also need covariance between B and
C, which is calculated as under:

State of the Prob- Return on Return on RB-E(RB) RC-E(RC) p


Economy ability (p) B (%) (RB) C (%) x[RB-E(RB)]
(RC) x[RC-E(RC)]

Recession 0.2 30 - 5.0 26 -19 (-) 98.8


Normal 0.5 5 15.0 1 1 0.50
Boom 0.3 (-)15 25.0 (-)19 11 (-) 62.7
total = (-)161

Covariance between the returns of B and C is (-)161


We have the following values:
WA = 0.4 WB = 0.4 WC = 0.2
σA = 19.08 σB = 15.62 σC = 10.44
σAB = (-)296 σAC = 199 σBC = (-) 161

Standard deviation

= [ (0.4 x 19.08)2 + (0.4 x 15.62)2 + (0.2 x 10.44)2 + [ 2 x 0.4 x 0.4 x (-) 296 ] +
+ [2 x 0.4 x 0.2 x 199] + [2 x 0.4 x 0.2 x (-) 161]1/2

= (58.25 + 39.04 + 4.36– 94.72 + 31.84 – 25.76)1/2 = 3.61


e.

(i) Risk-free rate is 6% and market risk premium is 15 – 6 = 9%


The SML relationship is
Required return = 6% + β x 9%

(ii) For stock A:


Required return = 6 % + 1.2 x 9 % = 16.8 %; Expected return = 19 %
Alpha = 19 – 16.8 = 2.2 %

For stock B:
Required return = 6 % - 0.70 x 9 % = - 0.3 %; Expected return = 4 %
Alpha = 4 + 0.3 = 4.3 %

For stock C:
Required return = 6% + 0.9 x 9 % = 14.1 %; Expected return = 14%
Alpha = 14 – 14.1 = (-) 0.1 %

f.

2
Period RD (%) RM (%) RD-RD RM-RM (RM-RM ) (RD-RD) (RM-RM)
1 -12 -5 -18.4 -11.2 125.44 206.08
2 6 4 -0.4 -2.2 4.84 0.88
3 12 8 5.6 1.8 3.24 10.08
4 20 15 13.6 8.8 77.44 119.68
5 6 9 -0.4 2.8 7.84 -1.12
Mean= 6.4 6.2 SUM= 218.8 335.6

σ2m = 218.8/4 = 54.7 Cov (D,M) = 335.6/4 = 83.9 ß = 83.9 / 54.7 = 1.53
Interpretation: The change in return of D is expected to be 1.53 times the expected change in
return on the market portfolio.

h.

CAPM assumes that return on a stock/portfolio is solely influenced by the market factor
whereas the APT assumes that the return is influenced by a set of factors called risk factors.

Chapter 10
OPTIONS AND THEIR VALUATION

1. S = 100 u = 1.5 d = 0.8

E = 105 r = 0.12 R = 1.12

The values of ∆ (hedge ratio) and B (amount borrowed) can be obtained as follows:

Cu – Cd
∆ =
(u – d) S

Cu = Max (150 – 105, 0) = 45

Cd = Max (80 – 105, 0) = 0

45 – 0 45 9
∆ = = = = 0.6429
0.7 x 100 70 14

u.Cd – d.Cu
B =
(u-d) R

(1.5 x 0) – (0.8 x 45)


=
0.7 x 1.12

-36
= = - 45.92
0.784

C = ∆S+B
= 0.6429 x 100 – 45.92
= Rs.18.37

Value of the call option = Rs.18.37

2. S = 40 u=? d = 0.8
R = 1.10 E = 45 C=8

We will assume that the current market price of the call is equal to the pair value of the call
as per the Binomial model.

Given the above data

Cd = Max (32 – 45, 0) = 0

∆ Cu – Cd R
= x
B u Cd – d Cu S

∆ Cu – 0 1.10
= x
B -0.8Cu 40

= (-) 0.034375

∆ = - 0.34375 B (1)
C = ∆S+B
8 = ∆ x 40 + B (2)

Substituting (1) in (2) we get

8 = (-0.034365 x 40) B + B
8 = -0.375 B
or B = - 21.33

∆ = - 0.034375 (-21.33) = 0.7332

The portfolio consists of 0.7332 of a share plus a borrowing of Rs.21.33 (entailing a


repayment of Rs.21.33 (1.10) = Rs.23.46 after one year). It follows that when u occurs either u x
40 x 0.7332 – 23.46 = u x 40 – 45
-10.672 u = -21.54
u = 2.02

or
u x 40 x 0.7332 – 23.46 = 0
u = 0.8

Since u > d, it follows that u = 2.02.


Put differently the stock price is expected to rise by 1.02 x 100 = 102%.

3. Using the standard notations of the Black-Scholes model we get the following results:
ln (S/E) + rt + σ2 t/2
d1 =
σ √ t

= ln (120 / 110) + 0.14 + 0.42/2


0.4

= 0.08701 + 0.14 + 0.08


0.4

= 0.7675

d2 = d1 - σ √ t
= 0.7675 – 0.4
= 0.3675

N(d1) = N (0.7675) ~ N (0.77) = 0.80785


N (d2) = N (0.3675) ~ N (0.37) = 0.64431

C = So N(d1) – E. e-rt. N(d2)


= 120 x 0.80785 – 110 x e-0.14 x 0.64431
= (120 x 0.80785) – (110 x 0.86936 x 0.64431)
= 35.33

Value of the call as per the Black and Scholes model is Rs.35.33.
4
l (S/E) + (r + σ2 /2) t
d1 =
σ √ t

= ln (80 / 82) + [0.1503 + (0.2)2/2]


0.2
= -0.0247 + 0.1703
0.2

= 0.7280

d2 = d1 - σ √ t
= 0.7280 – 0.2
= 0.5280

N(d1) = N (0.7280).
From the tables we have N(0.70) = 1- 0.2420 = 0.7580
and N(0.75)= 1- 0.2264 = 0.7736
By linear extrapolation, we get
N(0.7280) = 0.7580 + (0.7280 – 0.7000)(0.7736-0.7580)/0.05
= 0.7580 + 0.008736 = 0.7667
N(d2) = N(0.5280)
From the tables we have N(0.50) = 1- 0.3085 = 0.6915
N(0.55) = 1- 0.2912 = 0.7088
By linear extrapolation, we get
N(0.5280) = 0.6915 + (0.5280 – 0.5000)(0.7088 – 0.6915)/0.05
= 0.6915 + 0.009688 = 0.7012
E/ert = 82/1.1622 = 70.5558
C = So N(d1) – E. e-rt. N(d2)
= 80 x 0.7667 -70.5558 x 0.7012 = 11.86

l (S/E) + (r + σ2 /2) t
d1 =
σ √ t

= ln (80 / 85) + [0.1503 + (0.2)2/2]


0.2

= -0.060625 + 0.1703
0.2
= 0.5484

d2 = d1 - σ √ t
= 0.5484 – 0.2
= 0.3484

N(d1) = N (0.5484).
From the tables we have N(0.50) = 1- 0.3085 = 0.6915
and N(0.55)= 1- 0.2912 = 0.7088
By linear extrapolation, we get
N(0.5484) = 0.6915 + (0.5484 – 0.5000)(0.7088-0.6915)/0.05
= 0.6915 + 0.0167 = 0.7082
N(d2) = N(0.3484)
From the tables we have N(0.30) = 1- 0.3821 = 0.6179
N(0.35) = 1- 0.3632 = 0.6368
By linear extrapolation, we get
N(0.3484) = 0.6179 + (0.3484 – 0.3000)(0.6368 – 0.6179)/0.05
= 0.6179 + 0.0183= 0.6362
rt
E/e = 85/1.1622 = 73.1372
C = So N(d1) – E. e-rt. N(d2)
= 80 x 0.7082 -73.1372 x 0.6362 = 10.13
P = C –S + E/ert
= 10.13 – 80 + 73.1372 = 3.27

Value of the put option = Rs.3.27

6. So = Vo N(d1) – B1 e –rt N (d2)

= 6000 N (d1) – 5000 e – 0.1 N(d2)

ln (6000 / 5000) + (0.1 x 1) + (0.18/2)


d1 = ----------------------------------------------
√ 0.18 x √ 1

ln (1.2) + 0.19
=
0.4243

= 0.8775 = 0.88

N(d1) = N (0.88) = 0.81057


d2 = d1 - t
= 0.8775 - 0.18
= 0.4532 = 0.45

N (d2) = N (0.45) = 0.67364


So = 6000 x 0.81057 – (5000 x 0.9048 x 0.67364)
= 1816

B0 = V0 – S0
= 6000 – 1816

= 4184

MINICASE
b)
Call options with strike prices 280, 300 and 320 and put options with
strike prices 340and 360 are in - the – money.
Call options with strike prices 340 and 360 and put options with strike
prices 280, 300 and 320 are out of – the – money.

c) (i) If Pradeep Sharma sells Jan/340 call on 1000 shares, he will earn a
call premium of Rs.5000 now. However, he will forfeit the gains
that he would have enjoyed if the price of Newage Hospitals rises
above Rs.340.
(ii) If Pradeep Sharma sells Mar/300 call on 1000 shares, he will earn
a call premium of Rs.41,000 now. However, he will forfeit the gains
he would have enjoyed if the price of Newage Hospital remains
above Rs.300.
d) Let s be the stock price, p1 and p2 the call premia for March/ 340 and
March/ 360 calls respectively. When s is greater than 360, both the calls
will be exercised and the profit will be { s-340-p1} – { s-360-p2 } =Rs. 11
The maximum loss will be the initial investment , i.e. p1-p2 =Rs. 9
The break even will occur when the gain on purchased call equals the
net premium paid
i.e. s-340 = p1 – p2 =9 Therefore s= Rs. 349

e) If the stock price goes below Rs.300, Mr. Sharma can execute the put option and ensure
that his portfolio value does not go below Rs. 300 per share. However , if stock price goes
above Rs. 340, the call will be exercised and the stocks in the portfolio will have to be
delivered/ sold to meet the obligation, thus limiting the upper value of the portfolio to Rs.
340 per share. So long as the share price hovers between R. 300 and Rs. 340, Mr. Sharma
will be gainer by Rs. 8 ( net premium received).
Pay off
Profit

0
Stock price
305 340 375
·

f). Other things remaining constant, value of a call option


- increases when the current price of the stock increases.
- decreases when the exercise price increases.
- increases when option term to maturity increases.
- increases when the risk-free interest rate increases.
- increases when the variability of the stock price increases.

g). The three equations are


E
C0 = S0 N(d1) - ------ N (d2)
ert

S0 σ2
ln ------ + r + -----
E 2

d1 =
σ t

d2 = d1 - σ √ t

S0 = 325 E =320 t =0.25 r = 0.06 σ =0.30

325 (0.30)2
ln + 0.06 + x 0.25
320 2
d1 =
0.30 x √ 0.25
= ( 0.0155 + 0.02625) / 0.15 = 0. 2783

d2 = 0.2783 -0.30 √0.25 = 0.2783 – 0.15 = 0.1283


Using normal distribution table

N (d1) = 1 – [ 0.3821 + ( 0.4013- 0. 3821) ( 0.30 – 0.2783 ) /( 0.30 – 0.25) ]


=1- [ 0.3821 + 0. 0192 x 0.0217 / 0.05 ] = 0.6096

N ( d2 ) = 1- [ 0. 4404 + ( 0. 4602- 0.4404) ( 0. 15 – 0. 1283 /( 0. 15- 0.10 ) ]


= 1- [ 0.4404 + 0.0198 x 0.0217 / 0.05 ] = 0. 5510
E / e = 320 / e0.06 x 0. 25 = 320 / 1. 0151 = 315. 24
rt

C0 = 325 x 0.6096 – 315.24 x 0. 5510 = 198.12 – 173. 70 = Rs. 24.42


Chapter 11
TECHNIQUES OF CAPITAL BUDGETING

1.(a) NPV of the project at a discount rate of 14%.

= - 1,000,000 + 100,000 + 200,000


---------- ------------
(1.14) (1.14)2
+ 300,000 + 600,000 + 300,000
----------- ---------- ----------
3 4
(1.14) (1.14) (1.14)5

= - 44837

(b) NPV of the project at time varying discount rates

= - 1,000,000

+ 100,000

(1.12)

+ 200,000

(1.12) (1.13)

+ 300,000

(1.12) (1.13) (1.14)

+ 600,000

(1.12) (1.13) (1.14) (1.15)

+ 300,000

(1.12) (1.13) (1.14)(1.15)(1.16)

= - 1,000,000 + 89286 + 158028 + 207931 + 361620 + 155871


= - 27264
2. IRR (r) can be calculated by solving the following equations for the value of r.
60000 x PVIFA (r,7) = 300,000

i.e., PVIFA (r,7) = 5.000


Through a process of trial and error it can be verified that r = 9.20% pa.

3. The IRR (r) for the given cashflow stream can be obtained by solving the following equation
for the value of r.

-3000 + 9000 / (1+r) – 3000 / (1+r) = 0

Simplifying the above equation we get

r = 1.61, -0.61; (or) 161%, (-)61%

NOTE: Given two changes in the signs of cashflow, we get two values for the
IRR of the cashflow stream. In such cases, the IRR rule breaks down.

4. Define NCF as the minimum constant annual net cashflow that justifies the purchase of the
given equipment. The value of NCF can be obtained from the equation

NCF x PVIFA (10,8) = 500000


NCF = 500000 / 5.335
= 93271

5. Define I as the initial investment that is justified in relation to a net annual cash
inflow of 25000 for 10 years at a discount rate of 12% per annum. The value
of I can be obtained from the following equation

25000 x PVIFA (12,10) = I


i.e., I = 141256

6. Let us assume a discount rate of 15 %.

PV of benefits (PVB) = 25000 x PVIF (15,1)


+ 40000 x PVIF (15,2)
+ 50000 x PVIF (15,3)
+ 40000 x PVIF (15,4)
+ 30000 x PVIF (15,5)
= 122646 (A)

Investment = 100,000 (B)

Benefit cost ratio = 1.23 [= (A) / (B)]


7. The NPV’s of the three projects are as follows:

Project
P Q R

Discount rate

0% 400 500 600


5% 223 251 312

10% 69 40 70
15% - 66 - 142 - 135

25% - 291 - 435 - 461


30% - 386 - 555 - 591

8 NPV profiles for Projects P and Q for selected discount rates are as follows:
(a)
Project
P Q
Discount rate (%)
0 2950 500
5 1876 208
10 1075 - 28
15 471 - 222
20 11 - 382

b) (i) The IRR (r ) of project P can be obtained by solving the following


equation for `r’.

-1000 -1200 x PVIF (r,1) – 600 x PVIF (r,2) – 250 x PVIF (r,3)
+ 2000 x PVIF (r,4) + 4000 x PVIF (r,5) = 0

Through a process of trial and error we find that r = 20.13%

(ii) The IRR (r') of project Q can be obtained by solving the following equation for r'

-1600 + 200 x PVIF (r',1) + 400 x PVIF (r',2) + 600 x PVIF (r',3)
+ 800 x PVIF (r',4) + 100 x PVIF (r',5) = 0
Through a process of trial and error we find that r' = 9.34%.

c) From (a) we find that at a cost of capital of 10%

NPV (P) = 1075


NPV (Q) = - 28

Given that NPV (P) . NPV (Q); and NPV (P) > 0, I would choose project P.

From (a) we find that at a cost of capital of 20%

NPV (P) = 11

NPV (Q) = - 382

Again NPV (P) > NPV (Q); and NPV (P) > 0. I would choose project P.

d) Project P

PV of investment-related costs

= 1000 x PVIF (12,0)


+ 1200 x PVIF (12,1) + 600 x PVIF (12,2)
+ 250 x PVIF (12,3)
= 2728

TV of cash inflows = 2000 x (1.12) + 4000 = 6240

The MIRR of the project P is given by the equation:

2728 = 6240 x PVIF (MIRR,5)

(1 + MIRR)5 = 2.2874

MIRR = 18%
Project Q

PV of investment-related costs = 1600

TV of cash inflows @ 15% p.a. = 2772

The MIRR of project Q is given by the equation:

16000 (1 + MIRR)5 = 2772

MIRR = 11.62%

9.
(a) Project A

NPV at a cost of capital of 12%


= - 100 + 25 x PVIFA (12,6)
= Rs.2.79 million

IRR (r ) can be obtained by solving the following equation for r.


25 x PVIFA (r,6) = 100
i.e., r = 12,98%

Project B

NPV at a cost of capital of 12%


= - 50 + 13 x PVIFA (12,6)
= Rs.3.45 million

IRR (r') can be obtained by solving the equation


13 x PVIFA (r',6) = 50
i.e., r' = 14.40% [determined through a process of trial and error]

(b) Difference in capital outlays between projects A and B is Rs.50 million


Difference in net annual cash flow between projects A and B is Rs.12 million.
NPV of the differential project at 12%
= -50 + 12 x PVIFA (12,6)
= Rs.3.15 million

IRR (r'') of the differential project can be obtained from the equation
12 x PVIFA (r'', 6) = 50
i.e., r'' = 11.53%
10.
(a) Project M
The pay back period of the project lies between 2 and 3 years. Interpolating in
this range we get an approximate pay back period of 2.63 years/

Project N
The pay back period lies between 1 and 2 years. Interpolating in this range we
get an approximate pay back period of 1.55 years.

(b) Project M
Cost of capital = 12% p.a
PV of cash flows up to the end of year 1 = 9.82
PV of cash flows up to the end of year 2 = 24.97
PV of cash flows up to the end of year 3 = 47.75
PV of cash flows up to the end of year 4 = 71.26

Discounted pay back period (DPB) lies between 3 and 4 years. Interpolating in this range
we get an approximate DPB of 3.1 years.

Project N
Cost of capital = 12% per annum
PV of cash flows up to the end of year 1 = 33.93
PV of cash flows up to the end of year 2 = 51.47

DPB lies between 1 and 2 years. Interpolating in this range we get an approximate
DPB of 1.92 years.

(c ) Project M
Cost of capital = 12% per annum
NPV = - 50 + 11 x PVIF (12,1)
+ 19 x PVIF (12,2) + 32 x PVIF (12,3)
+ 37 x PVIF (12,4)
= Rs.21.26 million

Project N
Cost of capital = 12% per annum
NPV = Rs.20.63 million

Since the two projects are independent and the NPV of each project is (+) ve,
both the projects can be accepted. This assumes that there is no capital constraint.

(d) Project M
Cost of capital = 10% per annum
NPV = Rs.25.02 million
Project N
Cost of capital = 10% per annum
NPV = Rs.23.08 million

Since the two projects are mutually exclusive, we need to choose the project with the
higher NPV i.e., choose project M.

NOTE: The MIRR can also be used as a criterion of merit for choosing between the two
projects because their initial outlays are equal.

(e) Project M
Cost of capital = 15% per annum
NPV = 16.13 million

Project N
Cost of capital: 15% per annum
NPV = Rs.17.23 million

Again the two projects are mutually exclusive. So we choose the project with the
higher NPV, i.e., choose project N.

(f) Project M
Terminal value of the cash inflows: 114.47
MIRR of the project is given by the equation
50 (1 + MIRR)4 = 114.47
i.e., MIRR = 23.01%

Project N
Terminal value of the cash inflows: 115.41
MIRR of the project is given by the equation
50 ( 1+ MIRR)4 = 115.41
i.e., MIRR = 23.26%
MINICASE
(a) Project A

Cumulative Discounting Cumulative net


Cash net cash factor Present cash flow after
Year flow inflow @12% value discounting
0 (15,000) (15,000) 1.000 (15,000) (15,000)
1 11,000 (4,000) 0.893 9,823 (5,177)
2 7,000 3,000 0.797 5,579 402
3 4,800 0.712 3,418

Payback period is between 1 and 2 years. By linear interpolation we get the payback
period = 1 + 4,000 /(4,000 + 3,000) = 1.57 years.
Discounted payback period = 1 + 5,177 / ( 5,177 + 402) = 1.93 years
Project B
Cumulative Discounting Cumulative net
Cash net cash factor Present cash flow after
Year flow inflow @12% value discounting
0 (15,000) (15,000) 1.000 (15,000) (15,000)
1 3,500 (11,500) 0.893 3,126 (11,875)
2 8,000 (3,500) 0.797 6,376 (5,499)
3 13,000 9,500 0.712 9,256 3,757

Payback period is between 2 and 3 years. By linear interpolation we get the payback period = 2 +
3,500 /(3,500 + 9,500) = 2.27 years.
Discounted payback period = 2 + 5,499 / ( 5,499 + 3,757) = 2.59 years
(b)Project A
Discounting
Cash factor Present
Year flow @12% value
0 (15,000) 1.000 (15,000)
1 11,000 0.893 9,823
2 7,000 0.797 5,579
3 4,800 0.712 3,418
Net present value= 3,820

Project B

Discounting
Cash factor Present
Year flow @12% value
0 (15,000) 1.000 (15,000)
1 3,500 0.893 3,126
2 8,000 0.797 6,376
3 13,000 0.712 9,256
Net present value= 3,758
Project C
Discounting
Cash factor Present
Year flow @12% value
0 (15,000) 1.000 (15,000)
1 42,000 0.893 37,506
2 (4,000) 0.797 (3,188)
3
Net present value= 19,318

(c)
Project A

IRR is the value of r in the following equation.


11,000 / (1+r) + 7,000 / (1+r)2 + 4,800 / (1+r)3 = 15,000
Trying r = 28 %, the LHS = 11,000 / (1.28) + 7,000 / (1.28)2 + 4,800 / (1.28)3
= 15,155
As this value is slightly higher than 15,000, we try a higher discount rate of 29%
for r to get 11,000 / (1.29) + 7,000 / (1.29)2 + 4,800 / (1.29)3
= 14,970
By linear interpolation we get r = 28 + (15,155 – 15,000) / (15,155 – 14,970) =
28.84 %

Project B
IRR is the value of r in the following equation.
3,500 / (1+r) + 8,000 / (1+r)2 + 13,000 / (1+r)3 = 15,000
Trying r = 23 %, the LHS = 3,500 / (1.23) + 8,000 / (1.23)2 + 13,000 / (1.23)3
= 15,119

As this value is slightly higher than 15,000, we try a higher discount rate of 24%
for r to get 3,500 / (1.24) + 8,000 / (1.24)2 + 13,000 / (1.24)3
= 14,844

By linear interpolation we get r = 23 + (15,119 – 15,000) / (15,119 – 14,844) =


23. 43 %

Project C

IRR rule breaks down as the cash flows are non conventional.

(d) Calculation of MIRR for the three projects.


Project A
Terminal value of cash flows if reinvested at the cost of capital of 12% is
= 11,000 x (1.12)2 + 7,000 x 1.12 + 4,800 = 26,438
MIRR is the value of r in the equation: 26,438 / (1+r)3 =15,000
r = (26,438 / 15,000)1/3 -1 = 20.8%
Therefore MIRR = 20.8%

Project B
Terminal value of cash flows if reinvested at the cost of capital of 12% is
= 3,500 x (1.12)2 + 8,000 x 1.12 + 13,000 = 26,350
MIRR is the value of r in the equation: 26,350 / (1+r)3 =15,000
r = (26,350 / 15,000)1/3 -1 = 20.7 %
Therefore MIRR = 20.7 %

Project C
Terminal value of cash flow if reinvested at the cost of capital of 12% is
= 42,000 x 1.12 = 47,040
Present value of the costs = 15,000 + 4,000 / (1.12)2 = 18,189
MIRR is the value of r in the equation: 47,040 / (1+r)2 =18,189
r = (47,040 / 18,189)1/2 -1 = 60.8 %
Therefore MIRR = 60.8 %
Chapter 12
ESTIMATION OF PROJECT CASH FLOWS
1.
(a) Project Cash Flows (Rs. in million)

Year 0 1 2 3 4 5 6 7

1. Plant & machinery (150)

2. Working capital (50)

3. Revenues 250 250 250 250 250 250 250

4. Costs (excluding de-


preciation & interest) 100 100 100 100 100 100 100

5. Depreciation 37.5 28.13 21.09 15.82 11.87 8.90 6.67

6. Profit before tax 112.5 121.87 128.91 134.18 138.13 141.1143.33

7. Tax 33.75 36.56 38.67 40.25 41.44 42.33 43.0

8. Profit after tax 78.75 85.31 90.24 93.93 96.69 98.77100.33

9. Net salvage value of


plant & machinery 48

10. Recovery of working 50


capital

11. Initial outlay (=1+2) (200)

12. Operating CF (= 8 + 5) 116.25 113.44 111.33 109.75 108.56 107.6 107.00

13. Terminal CF ( = 9 +10) 98

14. NCF (200) 116.25 113.44 111.33 109.75 108.56 107.67 205
IRR (r) of the project can be obtained by solving the following equation for r
-200 + 116.25 x PVIF (r,1) + 113.44 x PVIF (r,2)

+ 111.33 x PVIF (r,3) + 109.75 x PVIF (r,4) + 108.56 x PVIF (r,5)


+107.67 x PVIF (r,6) + 205 x PVIF (r,7) = 0

Through a process of trial and error, we get r = 55.17%. The IRR of the project is
55.17%.
2. Post-tax Incremental Cash Flows (Rs. in million)

Year 0 1 2 3 4 5 6 7

1. Capital equipment (120)


2. Level of working capital 20 30 40 50 40 30 20
(ending)
3. Revenues 80 120 160 200 160 120 80
4. Raw material cost 24 36 48 60 48 36 24
5. Variable mfg cost. 8 12 16 20 16 12 8
6. Fixed operating & maint. 10 10 10 10 10 10 10
cost
7. Variable selling expenses 8 12 16 20 16 12 8
8. Incremental overheads 4 6 8 10 8 6 4
9. Loss of contribution 10 10 10 10 10 10 10
10.Bad debt loss 4
11. Depreciation 30 22.5 16.88 12.66 9.49 7.12 5.34
12. Profit before tax -14 11.5 35.12 57.34 42.51 26.88 6.66
13. Tax -4.2 3.45 10.54 17.20 12.75 8.06 2.00
14. Profit after tax -9.8 8.05 24.58 40.14 29.76 18.82 4.66
15. Net salvage value of
capital equipments 25
16. Recovery of working 16
capital
17. Initial investment (120)
18. Operating cash flow 20.2 30.55 41.46 52.80 39.25 25.94 14.00
(14 + 10+ 11)
19. ∆ Working capital 20 10 10 10 (10) (10) (10)
20. Terminal cash flow 41

21. Net cash flow (140) 10.20 20.55 31.46 62.80 49.25 35.94 55.00
(17+18-19+20)

(b) NPV of the net cash flow stream @ 15% per discount rate

= -140 + 10.20 x PVIF(15,1) + 20.55 x PVIF (15,2)


+ 31.46 x PVIF (15,3) + 62.80 x PVIF (15,4) + 49.25 x PVIF (15,5)
+ 35.94 x PVIF (15,6) + 55 x PVIF (15,7)
= Rs.1.70 million

3.
(a) A. Initial outlay (Time 0)
i. Cost of new machine Rs. 3,000,000
ii. Salvage value of old machine 900,000
iii Incremental working capital requirement 500,000
iv. Total net investment (=i – ii + iii) 2,600,000

B. Operating cash flow (years 1 through 5)

Year 1 2 3 4 5

i. Post-tax savings in
manufacturing costs 455,000 455,000 455,000 455,000 455,000

ii. Incremental
depreciation 550,000 412,500 309,375 232,031 174,023

iii. Tax shield on


incremental dep. 165,000 123,750 92,813 69,609 52,207

iv. Operating cash


flow ( i + iii) 620,000 578,750 547,813 524,609 507,207

C. Terminal cash flow (year 5)

i. Salvage value of new machine Rs. 1,500,000


ii. Salvage value of old machine 200,000
iii. Recovery of incremental working capital 500,000
iv. Terminal cash flow ( i – ii + iii) 1,800,000

D. Net cash flows associated with the replacement project (in Rs)

Year 0 1 2 3 4 5

NCF (2,600,000) 620000 578750 547813 524609 2307207

(b) NPV of the replacement project


= - 2600000 + 620000 x PVIF (14,1)
+ 578750 x PVIF (14,2)
+ 547813 x PVIF (14,3)
+ 524609 x PVIF (14,4)
+ 2307207 x PVIF (14,5)
= Rs.267849
4. Tax shield (savings) on depreciation (in Rs)
Depreciation Tax shield PV of tax shield
Year charge (DC) =0.4 x DC @ 15% p.a.

1 25000 10000 8696

2 18750 7500 5671

3 14063 5625 3699

4 10547 4219 2412

5 7910 3164 1573


----------
22051
----------

Present value of the tax savings on account of depreciation = Rs.22051

5. A. Initial outlay (at time 0)


i. Cost of new machine Rs. 400,000
ii. Salvage value of the old machine 90,000
iii. Net investment 310,000

B. Operating cash flow (years 1 through 5)

Year 1 2 3 4 5
i. Depreciation
of old machine 18000 14400 11520 9216 7373

ii. Depreciation
of new machine 100000 75000 56250 42188 31641

iii. Incremental
depreciation
( ii – i) 82000 60600 44730 32972 24268

iv. Tax savings on


incremental
depreciation
( 0.35 x (iii)) 28700 21210 15656 11540 8494

v. Operating cash
flow 28700 21210 15656 11540 8494
C. Terminal cash flow (year 5)
i. Salvage value of new machine Rs. 25000
ii. Salvage value of old machine 10000
iii. Incremental salvage value of new
machine = Terminal cash flow 15000

D. Net cash flows associated with the replacement proposal.

Year 0 1 2 3 4 5

NCF (310000) 28700 21210 15656 11540 23494

MINICASE
Solution:

a. Cash flows from the point of all investors (which is also called the explicit cost funds point of
view)
Rs.in million

Item 0 1 2 3 4 5

1. Fixed assets (15)


2. Net working
capital (8)
3. Revenues 30 30 30 30 30
4. Costs (other than
depreciation and
interest) 20 20 20 20 20
5. Loss of rental 1 1 1 1 1
6. Depreciation 3.750 2.813 2.109 1.582 1.187
7. Profit before tax 5.250 6.187 6.891 7.418 7.813
8. Tax 1.575 1.856 2.067 2.225 2.344
9. Profit after tax 3.675 4.331 4.824 5.193 5.469
10. Salvage value of
fixed assets 5.000
11. Net recovery of
working capital 8.000

12. Initial outlay (23)


13. Operating cash
inflow 7.425 7.144 6.933 6.775 6.656
14. Terminal cash
flow 13.000
15. Net cash flow (23) 7.425 7.144 6.933 6.775 19.656

b. Cash flows form the point of equity investors

Rs.in million

Item 0 1 2 3 4 5

1. Equity funds (10)


2. Revenues 30 30 30 30 30
3. Costs (other than
depreciation and
interest) 20 20 20 20 20
4. Loss of rental 1 1 1 1 1
5. Depreciation 3.75 2.813 2.109 1.582 1.187
6. Interest on working
capital advance 0.70 0.70 0.70 0.70 0.70
7. Interest on term
loans 1.20 1.125 0.825 0.525 0.225
8. Profit before tax 3.35 4.362 5.366 6.193 6.888
9. Tax 1.005 1.309 1.610 1.858 2.066
10. Profit after tax 2.345 3.053 3.756 4.335 4.822
11. Net salvage value
of fixed assets 5.000
12. Net salvage value
of current assets 10.000
13. Repayment of term
term loans 2.000 2.000 2.000 2.000
14. Repayment of bank
advance 5.000
15. Retirement of trade
creditors 2.000
16. Initial investment (10)
17. Operating cash
inflow 6.095 5.866 5.865 5.917 6.009
18. Liquidation and
retirement cash
flows (2.0) (2.0) (2.0) 6.00
19. Net cash flow (10) 6.095 3.866 3.865 3.917 12.009

Chapter 13
RISK ANALYSIS IN CAPITAL BUDGETING
1.
NPV of the project = -250 + 50 x PVIFA (13,10)
= Rs.21.31 million

(a) NPVs under alternative scenarios:


(Rs. in million)
Pessimistic Expected Optimistic

Investment 300 250 200


Sales 150 200 275
Variable costs 97.5 120 154
Fixed costs 30 20 15
Depreciation 30 25 20
Pretax profit - 7.5 35 86
Tax @ 28.57% - 2.14 10 24.57
Profit after tax - 5.36 25 61.43
Net cash flow 24.64 50 81.43
Cost of capital 14% 13% 12%

NPV - 171.47 21.31 260.10

Assumptions: (1) The useful life is assumed to be 10 years under all three
scenarios. It is also assumed that the salvage value of the
investment after ten years is zero.

(2) The investment is assumed to be depreciated at 10% per annum; and


it is also assumed that this method and rate of depreciation are
acceptable to the IT (income tax) authorities.

(3) The tax rate has been calculated from the given table i.e. 10 / 35 x
100 = 28.57%.

(4) It is assumed that only loss on this project can be offset against the
taxable profit on other projects of the company; and thus the
company can claim a tax shield on the loss in the same year.

(b) Accounting break even point (under ‘expected’ scenario)


Fixed costs + depreciation = Rs. 45 million
Contribution margin ratio = 80 / 200 = 0.4
Break even level of sales = 45 / 0.4 = Rs.112.5 million

Financial break even point (under ‘xpected’ scenario)


i. Annual net cash flow = 0.7143 [ 0.3 x sales – 45 ] + 25
= 0.2143 sales – 7.14

ii. PV (net cash flows) = [0.2143 sales – 7.14 ] x PVIFA (13,10)


= 1.1628 sales – 38.74

iii. Initial investment = 200

iv. Financial break even level


of sales = 238.74 / 1.1628 = Rs.205.31 million

2.
(a) (i) Sensitivity of NPV with respect to quantity manufactured and sold:
(in Rs)
Pessimistic Expected Optimistic

Initial investment 30000 30000 30000


Sale revenue 24000 42000 54000
Variable costs 16000 28000 36000
Fixed costs 3000 3000 3000
Depreciation 2000 2000 2000
Profit before tax 3000 9000 13000
Tax 1500 4500 6500
Profit after tax 1500 4500 6500
Net cash flow 3500 6500 8500
NPV at a cost of
capital of 10% p.a
and useful life of
5 years -16732 - 5360 2222

(ii) Sensitivity of NPV with respect to variations in unit price.

Pessimistic Expected Optimistic

Initial investment 30000 30000 30000


Sale revenue 28000 42000 70000
Variable costs 28000 28000 28000
Fixed costs 3000 3000 3000
Depreciation 2000 2000 2000
Profit before tax -5000 9000 37000
Tax -2500 4500 18500
Profit after tax -2500 4500 18500
Net cash flow - 500 6500 20500
NPV - 31895 (-) 5360 47711

(iii) Sensitivity of NPV with respect to variations in unit variable cost.

Pessimistic Expected Optimistic

Initial investment 30000 30000 30000


Sale revenue 42000 42000 42000
Variable costs 56000 28000 21000
Fixed costs 3000 3000 3000
Depreciation 2000 2000 2000
Profit before tax -11000 9000 16000
Tax -5500 4500 8000
Profit after tax -5500 4500 8000
Net cash flow -3500 6500 10000
NPV -43268 - 5360 7908

(b) Accounting break-even point

i. Fixed costs + depreciation = Rs.5000


ii. Contribution margin ratio = 10 / 30 = 0.3333
iii. Break-even level of sales = 5000 / 0.3333
= Rs.15000
Financial break-even point

i. Annual cash flow = 0.5 x (0.3333 Sales – 5000) = 2000


ii. PV of annual cash flow = (i) x PVIFA (10,5)
= 0.6318 sales – 1896
iii. Initial investment = 30000
iv. Break-even level of sales = 31896 / 0.6318 = Rs.50484

3. Define At as the random variable denoting net cash flow in year t.

A1 = 4 x 0.4 + 5 x 0.5 + 6 x 0.1


= 4.7

A2 = 5 x 0.4 + 6 x 0.4 + 7 x 0.2


= 5.8

A3 = 3 x 0.3 + 4 x 0.5 + 5 x 0.2


= 3.9

NPV = 4.7 / 1.1 +5.8 / (1.1)2 + 3.9 / (1.1)3 – 10


= Rs.2.00 million
σ 1
2
= 0.41

σ 2
2
= 0.56

σ 3
2
= 0.49

σ 1
2
σ 2
2
σ 3
2

σ 2
NPV = + +
2 4
(1.1) (1.1) (1.1)6

= 1.00
σ (NPV) = Rs.1.00 million

4. Expected NPV
4 At
= ∑ - 25,000
t=1 (1.08)t

= 12,000/(1.08) + 10,000 / (1.08)2 + 9,000 / (1.08)3


+ 8,000 / (1.08)4 – 25,000

= [ 12,000 x .926 + 10,000 x .857 + 9,000 x .794 + 8,000 x .735]


- 25,000
= 7,708

Standard deviation of NPV


4 σt

t=1 (1.08)t

= 5,000 / (1.08) + 6,000 / (1.08)2 + 5,000 / (1,08)3 + 6,000 / (1.08)4


= 5,000 x .926 + 6,000 x .857 + 5000 x .794 + 6,000 x .735
= 18,152

5. (a) Expected NPV


4 At
= ∑ - 10,000 …. (1)
t
t=1 (1.06)
A1 = 2,000 x 0.2 + 3,000 x 0.5 + 4,000 x 0.3
= 3,100

A2 = 3,000 x 0.4 + 4,000 x 0.3 + 5,000 x 0.3


= 3,900

A3 = 4,000 x 0.3 + 5,000 x 0.5 + 6,000 x 0.2


= 4,900

A4 = 2,000 x 0.2 + 3,000 x 0.4 + 4,000 x 0.4


= 3,200

Substituting these values in (1) we get

Expected NPV = NPV

= 3,100 / (1.06)+ 3,900 / 1.06)2 + 4,900 / (1.06)3 + 3,200 / (1,06)4


- 10,000 = Rs.3,044
(b)
The variance of NPV is givenby the expression
4 σ 2t
σ (NPV) = ∑
2
…….. (2)
t=1 (1.06)2t

σ 1
2
= [(2,000 – 3,100)2 x 0.2 + (3,000 – 3,100)2 x 0.5
+ (4,000 – 3,100)2 x 0.3]
= 490,000

σ 2
2
= [(3,000 – 3,900)2 x 0.4 + (4,000 – 3,900)2 x 0.3
+ (5,000 – 3900)2 x 0.3]
= 690,000

σ 3
2
= [(4,000 – 4,900)2 x 0.3 + (5,000 – 4,900)2 x 0.5
+ (6,000 – 4,900)2 x 0.2]
= 490,000

σ 4
2
= [(2,000 – 3,200)2 x 0.2 + (3,000 – 3,200)2 x 0.4
+ (4,000 – 3200)2 x 0.4]
= 560,000
Substituting these values in (2) we get
490,000 / (1.06)2 + 690,000 / (1.06)4
+ 490,000 / (1.06)6 + 560,000 / (1.06)8
[ 490,000 x 0.890 + 690,000 x 0.792
+ 490,000 x 0.705 + 560,000 x 0.627 ]
= 1,679,150
σ NPV = 1,679,150 = Rs.1,296

NPV – NPV 0 - NPV


Prob (NPV < 0) = Prob. <
σ NPV σ NPV
0 – 3044
= Prob Z<
1296

= Prob (Z < -2.35)

The required probability is given by the shaded area in the following normal curve.

P (Z < - 2.35) = 0.5 – P (-2.35 < Z < 0)


= 0.5 – P (0 < Z < 2.35)
= 0.5 – 0.4906
= 0.0094
(c)
So the probability of NPV being negative is 0.0094

Prob (P1 > 1.2) Prob (PV / I > 1.2)


Prob (NPV / I > 0.2)
Prob. (NPV > 0.2 x 10,000)
Prob (NPV > 2,000)

Prob (NPV >2,000)= Prob (Z > 2,000- 3,044 / 1,296)


Prob (Z > - 0.81)

The required probability is given by the shaded area of the following normal
curve:
P(Z > - 0.81) = 0.5 + P(-0.81 < Z < 0)
= 0.5 + P(0 < Z < 0.81)
= 0.5 + 0.2910
= 0.7910

So the probability of P1 > 1.2 as 0.7910


6
Certainty
Equivalent Certainty Discount
Yea Cash Factor: αt Equivalent Factor Present
r Flow =1 - 0.06t value at 8% Value
(30,000
0 ) 1 (30,000) 1 (30,000.00)
1 7,000 0.94 6,580 0.9259 6,092.59
2 8,000 0.88 7,040 0.8573 6,035.67
3 9,000 0.82 7,380 0.7938 5,858.48
4 10,000 0.76 7,600 0.7350 5,586.23
5 8,000 0.7 5,600 0.6806 3,811.27
NPV
= (2,615.77)

MINICASE

Solution:
1. The expected NPV of the turboprop aircraft

0.65 (5500) + 0.35 (500)


NPV = - 11000 +
(1.12)

0.65 [0.8 (17500) + 0.2 (3000)] + 0.35 [0.4 (17500) + 0.6 (3000)]
+
(1.12)2
= 2369

2. If Southern Airways buys the piston engine aircraft and the demand in year 1 turns out to
be high, a further decision has to be made with respect to capacity expansion. To evaluate
the piston engine aircraft, proceed as follows:

First, calculate the NPV of the two options viz., ‘expand’ and ‘do not expand’ at decision
point D2:

0.8 (15000) + 0.2 (1600)


Expand : NPV = - 4400 +
1.12
= 6600
0.8 (6500) + 0.2 (2400)
Do not expand : NPV =
1.12
= 5071

Second, truncate the ‘do not expand’ option as it is inferior to the ‘expand’ option. This
means that the NPV at decision point D2 will be 6600

Third, calculate the NPV of the piston engine aircraft option.

0.65 (2500+6600) + 0.35 (800)


NPV = – 5500 +
1.12

0.35 [0.2 (6500) + 0.8 (2400)]


+
(1.12)2

= – 5500 + 5531 + 898 = 929

3. The value of the option to expand in the case of piston engine aircraft
If Southern Airways does not have the option of expanding capacity at the end of year 1, the
NPV of the piston engine aircraft would be:

0.65 (2500) + 0.35 (800)


NPV = – 5500 +
1.12

0.65 [0.8 (6500) + 0.2 (2400)] + 0.35 [0.2 (6500) + 0.8 (2400)]
+
(1.12)2

= - 5500 + 1701 + 3842 = 43

Thus the option to expand has a value of 929 – 43 = 886

4. Value of the option to abandon if the turboprop aircraft can be sold for 8000 at the end of
year 1

If the demand in year 1 turns out to be low, the payoffs for the ‘continuation’ and
‘abandonment’ options as of year 1 are as follows.
0.4 (17500) + 0.6 (3000)
Continuation: = 7857
1.12

Abandonment : 8000

Thus it makes sense to sell off the aircraft after year 1, if the demand in year 1 turns out to
be low.

The NPV of the turboprop aircraft with abandonment possibility is

0.65 [5500 +{0.8 (17500) + 0.2 (3000)}/ (1.12)] + 0.35 (500 +8000)
NPV = - 11,000 +
(1.12)

12048 + 2975
= - 11,000 + = 2413
1.12

Since the turboprop aircraft without the abandonment option has a value of 2369,
the value of the abandonment option is : 2413 – 2369 = 44

5 The value of the option to abandon if the piston engine aircraft can be sold for 4400 at the
end of year 1:

If the demand in year 1 turns out to be low, the payoffs for the ‘continuation’ and
‘abandonment’ options as of year 1 are as follows:

0.2 (6500) + 0.8 (2400)


Continuation : = 2875
1.12

Abandonment : 4400

Thus, it makes sense to sell off the aircraft after year 1, if the demand in year 1 turns out to
be low.

The NPV of the piston engine aircraft with abandonment possibility is:

0.65 [2500 + 6600] + 0.35 [800 + 4400]


NPV = - 5500 +
1.12
5915 + 1820
= - 5500 + = 1406
1.12

For the piston engine aircraft the possibility of abandonment increases the NPV from 929
to 1406. Hence the value of the abandonment option is 477.
Chapter 14
THE COST OF CAPITAL

1(a) Define rD as the pre-tax cost of debt. Using the approximate yield formula, rD can be
calculated as follows:

14 + (100 – 108)/10
rD = ------------------------ x 100 = 12.60%
0.4 x 100 + 0.6x108

(b) After tax cost = 12.60 x (1 – 0.35) = 8.19%

2. Define rp as the cost of preference capital. Using the approximate yield formula rp can be
calculated as follows:

9 + (100 – 92)/6
rp = --------------------
0.4 x100 + 0.6x92

= 0.1085 (or) 10.85%

3. WACC = 0.4 x 13% x (1 – 0.35)


+ 0.6 x 18%
= 14.18%

4. Cost of equity = 10% + 1.2 x 7% = 18.4%


(using SML equation)

Pre-tax cost of debt = 14%

After-tax cost of debt = 14% x (1 – 0.35) = 9.1%

Debt equity ratio = 2:3

WACC = 2/5 x 9.1% + 3/5 x 18.4%

= 14.68%

5. Given
0.5 x 14% x (1 – 0.35) + 0.5 x rE = 12%

where rE is the cost of equity capital.

Therefore rE – 14.9%
Using the SML equation we get

11% + 8% x β = 14.9%

where β denotes the beta of Azeez’s equity.

Solving this equation we get β = 0.4875.

6(a) The cost of debt of 12% represents the historical interest rate at the time the debt was
originally issued. But we need to calculate the marginal cost of debt (cost of raising new
debt); and for this purpose we need to calculate the yield to maturity of the debt as on the
balance sheet date. The yield to maturity will not be equal to12% unless the book value of
debt is equal to the market value of debt on the balance sheet date.

(b) The cost of equity has been taken as D1/P0 ( = 6/100) whereas the cost of equity is
(D1/P0) + g where g represents the expected constant growth rate in dividend per share.

7. (a) The book value and market values of the different sources of finance are
provided in the following table. The book value weights and the market value
weights are provided within parenthesis in the table.

(Rs. in million)
Source Book value Market value
Equity 800 (0.54) 2400 (0.78)
Debentures – first series 300 (0.20) 270 (0.09)
Debentures – second series 200 (0.13) 204 (0.06)
Bank loan 200 (0.13) 200 (0.07)
Total 1500 (1.00) 3074 (1.00)

(b) I would use weights based on the market value because to justify its valuation Samanta
must earn competitive returns for investors on its market value

8.
(a) Given
rD x (1 – 0.3) x 4/9 + 20% x 5/9 = 15%
rD = 12.5%,where rD represents the pre-tax cost of debt.

(b) Given
13% x (1 – 0.3) x 4/9 + rE x 5/9 = 15%
rE = 19.72%, where rE represents the cost of equity.
9. Cost of equity = D1/P0 + g
= 3.00 / 30.00 + 0.05
= 15%
(a) The first chunk of financing will comprise of Rs.5 million of retained earnings costing
15 percent and Rs.2.5 million of debt costing 14 (1-.3) = 9.8 per cent
The second chunk of financing will comprise of Rs.5 million of additional equity
costing 15 per cent and Rs.2.5 million of debt costing 15 (1-.3) = 10.5 per cent

(b) The marginal cost of capital in the first chunk will be :


5/7.5 x 15% + 2.5/7.5 x 9.8% = 13.27%

The marginal cost of capital in the second chunk will be:


5/7.5 x 15% + 2.5/7.5 x 10.5% = 13.50%

Note : We have assumed that


(i) The net realisation per share will be Rs.25, after floatation costs, and
(ii) The planned investment of Rs.15 million is inclusive of floatation costs

9.1 (a) (i) The cost of equity and retained earnings


rE = D1/PO + g
= 1.50 / 20.00 + 0.07 = 14.5%
The cost of preference capital, using the approximate formula, is :

11 + (100-75)/10
rP = = 15.9%
0.6 x 75 + 0.4 x 100
The pre-tax cost of debentures, using the approximate formula, is :

13.5 + (100-80)/6
rD = = 19.1%
0.6x80 + 0.4x100

The post-tax cost of debentures is


19.1 (1-tax rate) = 19.1 (1 – 0.5)
= 9.6%

The post-tax cost of term loans is


12 (1-tax rate) = 12 (1 – 0.5)
= 6.0%
The average cost of capital using book value proportions is calculated below:

Source of capital Component Book value Book value Product of


Cost Rs. in million proportion (1) & (3)
(1) (2) (3)
Equity capital 14.5% 100 0.28 4.06
Preference capital 15.9% 10 0.03 0.48
Retained earnings 14.5% 120 0.33 4.79
Debentures 9.6% 50 0.14 1.34
Term loans 6.0% 80 0.22 1.32
360 Average cost 11.99%
capital

(ii) The average cost of capital using market value proportions is calculated below :

Source of capital Component Market value Market value Product of


cost Rs. in million proportion
(1) (2) (3) (1) & (3)

Equity capital
and retained earnings 14.5% 200 0.62 8.99
Preference capital 15.9% 7.5 0.02 0.32
Debentures 9.6% 40 0.12 1.15
Term loans 6.0% 80 0.24 1.44

327.5 Average cost 11.90%


capital

(b)
The Rs.100 million to be raised will consist of the following:
Retained earnings Rs.15 million
Additional equity Rs. 35 million
Debt Rs. 50 million
The first batch will consist of Rs. 15 million each of retained earnings
and debt costing 14.5 percent and 14(1-0.5)= 7 percent respectively. The
second batch will consist of Rs. 10 million each of additional equity and
debt at 14.5 percent and 7percent respectively. The third chunk will
consist of Rs.25 million each of additional equity and debt costing 14.5
percent and 15(1-0.5) = 7.5 percent respectively.
The marginal cost of capital in the chunks will be as under
First batch: (0.5x14.5 ) + (0.5 x 7) = 10.75 %
Second batch: (0.5x14.5 ) + (0.5 x 7) = 10.75 %
Third batch : (0.5x14.5 ) + (0.5 x 7.5) = 11 %
The marginal cost of capital schedule for the firm will be as under.
Range of total financing Weighted marginal cost of
( Rs. in million) capital ( %)
0 - 50 10.75
50-100 11.00
Here it is assumed that the Rs.100 million to be raised is inclusive of floatation costs.

10
(a) WACC = 1/3 x 13% x (1 – 0.3)
+ 2/3 x 20%
= 16.37%

(b) Weighted average floatation cost


= 1/3 x 3% + 2/3 x 12%
= 9%

(c) NPV of the proposal after taking into account the floatation costs

= 130 x PVIFA (16.37, 8) – 500 / (1 - 0.09)


= Rs.8.51 million

11. Required return


based on SML Expected
Project Beta equation (%) return (%)

P 0.6 14.8 13
Q 0.9 17.2 14
R 1.5 22.0 16
S 1.5 22.0 20

Given a hurdle rate of 18% (the firm’s cost of capital), projects P, Q and R would have been
rejected because the expected returns on these projects are below 18%. Project S would be
accepted because the expected return on this project exceeds 18%. An appropriate basis for
accepting or rejecting the projects would be to compare the expected rate of return and the required
rate of return for each project. Based on this comparison, we find that all the four projects need to
be rejected.
MINICASE

Solution:

a. All sources other than non-interest bearing liabilities

b. Pre-tax cost of debt & post-tax cost of debt

10 + (100 – 112) / 8 8.5


rd = = = 7.93
0.6 x 112 + 0.4 x 100 107.2

rd (1 – 0.3) = 5.55

c. Post-tax cost of preference

9 + (100 – 106) / 5 7.8


= = 7.53%
0.6 x 106 + 0.4 x 100 103.6

d. Cost of equity using the DDM

2.80 (1.10)
+ 0.10 = 0.385 + 0.10
80
= 0.1385 = 13.85%

e. Cost of equity using the CAPM

7 + 1.1(7) = 14.70%

f. WACC
0.50 x 14.70 + 0.10 x 7.53 + 0.40 x 5.55
= 7.35 + 0.75 + 2.22
= 10.32%

g. Cost of capital for the new business

0.5 [7 + 1.5 (7)] + 0.5 [ 11 (1 – 0.3)]


8.75 + 3.85
= 12.60%
Chapter 15
CAPITAL BUDGETING: EXTENSIONS

1. Let us assume that the cost of capital is 12 percent.


EAC
(Plastic Emulsion) = 300000 / PVIFA (12,7)
= 300000 / 4.564
= Rs.65732

EAC
(Distemper Painting) = 180000 / PVIFA (12,3)
= 180000 / 2.402
= Rs.74938

Since EAC of plastic emulsion is less than that of distemper painting, it is the preferred
alternative.

2. PV of the net costs associated with the internal transportation system

= 1 500 000 + 300 000 x PVIF (13,1) + 360 000 x PVIF (13,2)
+ 400 000 x PVIF (13,3) + 450 000 x PVIF (13,4)
+ 500 000 x PVIF (13,5) - 300 000 x PVIF (13,5)
= 2709185

EAC of the internal transportation system

= 2709185 / PVIFA (13,5)


= 2709185 / 3.517
= Rs.770 311

3. EAC [ Standard overhaul]

= 500 000 / PVIFA (14,6)


= 500 000 / 3.889
= Rs.128568 ……… (A)

EAC [Less costly overhaul]

= 200 000 / PVIFA (14,2)


= 200 000 / 1.647
= Rs.121433 ……… (B)

Since (B) < (A), the less costly overhaul is preferred alternative.
4.
(a) Base case NPV

= -12,000,000 + 3,000,000 x PVIFA (20,6)


= -12,000,000 + 997,8000
= (-) Rs.2,022,000

(b) Issue costs = 6,000,000 / 0.88 - 6,000,000

= Rs.818 182

Adjusted NPV after adjusting for issue costs

= - 2,022,000 – 818,182
= - Rs.2,840,182

(c) The present value of interest tax shield is calculated below :

Year Debt outstanding at Interest Tax shield Present value of


the beginning tax shield
1 6,000,000 1,080,000 324,000 274,590
2 6,000,000 1,080,000 324,000 232,697
3 5,250,000 945,000 283,000 172,538
4 4,500,000 810,000 243,000 125,339
5 3,750,000 675,000 202,000 88,513
6 3,000,000 540,000 162,000 60,005
7 2,225,000 400,500 120,000 37,715
8 1,500,000 270,000 81,000 21,546
9 750,000 135,000 40,500 9,133

Present value of tax shield = Rs.1,022,076

5.
(a) Base case BPV

= - 8,000,000 + 2,000,000 x PVIFA (18,6)


= - Rs.1,004,000

(b) Adjusted NPV after adjustment for issue cost of external equity

= Base case NPV – Issue cost


= - 1,004,000 – [ 3,000,000 / 0.9 – 3,000,000]
= - Rs.1,337,333
(c) The present value of interest tax shield is calculated below :

Year Debt outstanding at Interest Tax shield Present value of


the beginning tax shield
1 5,000,000 750,000 300,000 260,880
2 5,000,000 750,000 300,000 226,830
3 4,000,000 600,000 240,000 157,800
4 3,000,000 450,000 180,000 102,924
5 2,000,000 300,000 120,000 59,664
6 1,000,000 150,000 60,000 25,938

Present value of tax shield = Rs.834,036


Chapter 18
RAISING LONG TERM FINANCE

1 Underwriting Shares Excess/ Credit Net


commitment procured shortfall shortfall

A 70,000 50,000 (20,000) 4919 (15081)

B 50,000 30,000 (20,000) 3514 (16486)

C 40,000 30,000 (10,000) 2811 (7189)

D 25,000 12,000 (13,000) 1757 (11243)

E 15,000 28,000 13,000

2. Po = Rs.180 N=5
a. The theoretical value of a right if the subscription price is Rs.150
Po – S 180 – 150
= = Rs.5
N+1 5+1

b. The ex-rights value per share if the subscription price is Rs.160


NPo + S 5 x 180 + 160
= = Rs.176.7
N+1 5+1

c. The theoretical value per share, ex-rights, if the subscription price is


Rs.180? 100?
5 x 180 + 180
= Rs.180
5+1

5 x 180 + 100
= Rs.166.7
5+1
Chapter 19
CAPITAL STRUCTURE AND FIRM VALUE

1. Net operating income (O) : Rs.30 million


Interest on debt (I) : Rs.10 million
Equity earnings (P) : Rs.20 million
Cost of equity (rE) : 15%

Cost of debt (rD) : 10%


Market value of equity (E) : Rs.20 million/0.15 =Rs.13 million
Market value of debt (D) : Rs.10 million/0.10 =Rs.100 million
Market value of the firm (V) : Rs.233 million

2. Box Cox

Market value of equity 2,000,000/0.15 2,000,000/0.15


= Rs.13.33 million = Rs.13.33 million
Market value of debt 0 1,000,000/0.10
=Rs.10 million
Market value of the firm Rs.13.33million =23.33 million

(a) Average cost of capital for Box Corporation


13.33. 0
x 15% + x 10% = 15%
13.33 13.33

Average cost of capital for Cox Corporation


13.33 10.00
x 15% + x 10% = 12.86%
23.33 23.33

(b) If Box Corporation employs Rs.30 million of debt to finance a project that yields
Rs.4 million net operating income, its financials will be as follows.

Net operating income Rs.6,000,000


Interest on debt Rs.3,000,000
Equity earnings Rs.3,000,000
Cost of equity 15%
Cost of debt 10%
Market value of equity Rs.20 million
Market value of debt Rs.30 million
Market value of the firm Rs.50 million
Average cost of capital
20 30
15% x + 10% = 12%
50 50

(c) If Cox Corporation sells Rs.10 million of additional equity to retire


Rs.10 million of debt , it will become an all-equity company. So its
average cost of capital will simply be equal to its cost of equity,
which is 15%.

3. rE = rA + (rA-rD)D/E
20 = 12 + (12-8) D/E
So D/E = 2

4. E D E D
rE rD rA = rE + rD
D+E D+E (%) (%) D+E D+E

1.00 0.00 11.0 6.0 11.00


0.90 0.10 11.0 6.5 10.55
0.80 0.20 11.5 7.0 10.60
0.70 0.30 12.5 7.5 11.00
0.60 0.40 13.0 8.5 11.20
0.50 0.50 14.0 9.5 11.75
0.40 0.60 15.0 11.0 12.60
0.30 0.70 16.0 12.0 13.20
0.20 0.80 18.0 13.0 14.00
0.10 0.90 20.0 14.0 14.20

The optimal debt ratio is 0.10 as it minimises the weighted average


cost of capital.

5. (a) If you own Rs.10,000 worth of Bharat Company, the levered company
which is valued more, you would sell shares of Bharat Company, resort
to personal leverage, and buy the shares of Charat Company.
(b) The arbitrage will cease when Charat Company and Bharat Company
are valued alike

6. The value of Ashwini Limited according to Modigliani and Miller


hypothesis is
Expected operating income 15
= = Rs.125 million
Discount rate applicable to the 0.12
risk class to which Aswini belongs
7. The tax advantage of one rupee of debt is :
1-(1-tc) (1-tpe) (1-0.55) (1-0.05)
= 1 -
(1-tpd) (1-0.25)

= 0.43 rupee

Chapter 20
CAPITAL STRUCTURE DECISION

1.(a) Currently
No. of shares = 1,500,000
EBIT = Rs 7.2 million
Interest = 0
Preference dividend = Rs.12 x 50,000 = Rs.0.6 million
EPS = Rs.2

(EBIT – Interest) (1-t) – Preference dividend


EPS =
No. of shares

(7,200,000 – 0 ) (1-t) – 600,000


Rs.2 =
1,500,000

Hence t = 0.5 or 50 per cent

The EPS under the two financing plans is :

Financing Plan A : Issue of 1,000,000 shares

(EBIT - 0 ) ( 1 – 0.5) - 600,000


EPSA =
2,500,000

Financing Plan B : Issue of Rs.10 million debentures carrying 15 per cent


interest

(EBIT – 1,500,000) (1-0.5) – 600,000


EPSB =
1,500,000

The EPS – EBIT indifference point can be obtained by equating EPSA and EPSB
(EBIT – 0 ) (1 – 0.5) – 600,000 (EBIT – 1,500,000) (1 – 0.5) – 600,000
=
2,500,000 1,500,000

Solving the above we get EBIT = Rs.4,950,000 and at that EBIT, EPS is Rs.0.75
under both the plans

(b) As long as EBIT is less than Rs.4,950,000 equity financing maximixes EPS.
When EBIT exceeds Rs.4,950,000 debt financing maximises EPS.

2.
(a) EPS – EBIT equation for alternative A
EBIT ( 1 – 0.5)
EPSA =
2,000,000
(b) EPS – EBIT equation for alternative B
EBIT ( 1 – 0.5 ) – 440,000
EPSB =
1,600,000

(c) EPS – EBIT equation for alternative C


(EBIT – 1,200,000) (1- 0.5)
EPSC =
1,200,000

(d) The three alternative plans of financing ranked in terms of EPS over varying
Levels of EBIT are given the following table

Ranking of Alternatives

EBIT EPSA EPSB EPSC


(Rs.) (Rs.) (Rs.) (Rs.)

2,000,000 0.50(I) 0.35(II) 0.33(III)


2,160,000 0.54(I) 0.40(II) 0.40(II)
3,000,000 0.75(I) 0.66(II) 0.75(I)
4,000,000 1.00(II) 0.98(III) 1.17(I)
4,400,000 1.10(II) 1.10(II) 1.33(I)
More than 4,400,000 (III) (II) (I)

3. Plan A : Issue 0.8 million equity shares at Rs. 12.5 per share.
Plan B : Issue Rs.10 million of debt carrying interest rate of 15 per cent.
(EBIT – 0 ) (1 – 0.6)
EPSA =
1,800,000
(EBIT – 1,500,000) (1 – 0.6)
EPSB =
1,000,000

Equating EPSA and EPSB , we get


(EBIT – 0 ) (1 – 0.6) (EBIT – 1,500,000) (1 – 0.6)
=
1,800,000 1,000,000

Solving this we get EBIT = 3,375,000 or 3.375 million

Thus the debt alternative is better than the equity alternative when
EBIT > 3.375 million

EBIT – EBIT 3.375 – 7.000


Prob(EBIT>3,375,000) = Prob >
σ EBIT 3.000

= Prob [z > - 1.21]


= 0.8869

4. ROE = [ ROI + ( ROI – r ) D/E ] (1 – t )


15 = [ 14 + ( 14 – 8 ) D/E ] ( 1- 0.5 )
D/E = 2.67

5. ROE = [12 + (12 – 9 ) 0.6 ] (1 – 0.6)


= 5.52 per cent

6. 18 = [ ROI + ( ROI – 8 ) 0.7 ] ( 1 – 0.5)


ROI = 24.47 per cent
EBIT
7. a. Interest coverage ratio =
Interest on debt

15
=
4
= 3.75
EBIT + Depreciation
b. Cash flow coverage ratio =
Loan repayment instalment
Int.on debt +
(1 – Tax rate)
= 15 + 3

4+5

= 2
8. The debt service coverage ratio for Pioneer Automobiles Limited is given by :
5
∑ ( PAT i + Depi + Inti)
i=1
DSCR = 5
∑ ( Inti + LRIi)
i=1

= 133.00 + 49.14 +95.80

95.80 + 72.00

= 277.94
167.80
= 1.66

9. (a) If the entire outlay of Rs. 300 million is raised by way of debt carrying 15 per cent
interest, the interest burden will be Rs. 45 million.
Considering the interest burden the net cash flows of the firm during
a recessionary year will have an expected value of Rs. 35 million (Rs.80 million - Rs. 45
million ) and a standard deviation of Rs. 40 million .
Since the net cash flow (X) is distributed normally
X – 35

40
has a standard normal deviation
Cash flow inadequacy means that X is less than 0.
0.35
Prob(X<0) = Prob (z< ) = Prob (z<- 0.875)
40
= 0.1909
(b) Since µ = Rs.80 million, σ = Rs.40 million , and the Z value corresponding to the risk
tolerance limit of 5 per cent is – 1.645, the cash available from the operations to service the
debt is equal to X which is defined as :
X – 80
= - 1.645
40
X = Rs.14.2 million
Given 15 per cent interest rate, the debt than be serviced is
14.2
= Rs. 94.67 million
0.15

MINICASE

(a) If the firm chooses the equity option, it will have to issue 2 crore shares and its interest
burden will remain at the current level of Rs.20 crore. If the firm chooses the debt option,
the interest burden will go upto Rs.36 crore, but the number of equity shares will remain
unchanged at 14 crore. So, the EPS – PBIT indifference point is the value of PBIT in the
following equation.

(PBIT -20)( 1- 0.3) (PBIT – 36)( 1-0.3)


=
16 14

PBIT = Rs. 148 crore

(b) The projected EPS under the two financing options is given below

Projected
Current Equity option Debt option
Revenues 800 1040 1040
Variable costs 480 624 624
Contribution margin 320 416 416
Fixed operating costs 180 230 230
PBIT 140 186 186
Interest 20 20 36
PBT 120 166 150
Tax 36 55.33 50
PAT 84 110.67 100
No.of equity shares 14 16 14
EPS 6 6.92 7.14
(c)

Contribution margin
The degree of total leverage (DTL) is defined as :
PBIT
So, the DTL will be as follows:

DTL

Current = 320/140 = 2.67


Equity option = 416/166 = 2.51
Debt option = 416/150 = 2.77
Chapter 21
DIVIDEND POLICY AND FIRM VALUE

1.(a) Payout ratio Price per share

3(0.5)+3(0.5) 0.15
0.5
0.12
= Rs. 28.13
0.12

3(0.7 5)+3(0.25) 0.15


0.12
0.75 = Rs. 26.56
0.12

3(1.00)
1.00 = Rs. 25.00
0.12
(b)

Dividend Price as per Gordon model P0


payout ratio =E1(1-b)/(k-br)
25% = 3 x 0.75/(0.12 - 0.75x 0.15) =Rs. 300
=Rs.
50% = 3 x 0.50/(0.12 - 0.50x 0.15) 33.33
=Rs.
75% = 3 x 0.25/(0.12 - 0.25x 0.15) 9.09

2.
P Q
• Next year’s price 80 74
• Dividend 0 6
• Current price P Q
• Capital appreciation (80-P) (74-Q)
• Post-tax capital appreciation 0.9(80-P) 0.9 (74-Q)
• Post-tax dividend income 0 0.8 x 6
• Total return 0.9 (80-P) 0.9 (74-Q) + 4.8
P Q
= 14% =14%
• Current price (obtained by solving P = Rs.69.23 Q = Rs.68.65
the preceding equation)

Chapter 22
DIVIDEND DECISION

1. a. Under a pure residual dividend policy, the dividend per share over the 4 year
period will be as follows:

DPS Under Pure Residual Dividend Policy


( in Rs.)

Year 1 2 3 4

Earnings 10,000 12,000 9,000 15,000


Capital expenditure 8,000 7,000 10,000 8,000
Equity investment 4,000 3,500 5,000 4,000
Pure residual
dividends 6,000 8,500 4,000 11,000
Dividends per share 1.20 1.70 0.80 2.20

b. The external financing required over the 4 year period (under the assumption that the
company plans to raise dividends by 10 percents every two years) is given below :
Required Level of External Financing
(in Rs.)

Year 1 2 3 4

A. Net income 10,000 12,000 9,000 15,000

B. Targeted DPS 1.00 1.10 1.10 1.21

C. Total dividends 5,000 5,500 5,500 6,050

D. Retained earnings(A-C) 5,000 6,500 3,500 8,950

E. Capital expenditure 8,000 7,000 10,000 8,000

F. External financing
requirement 3,000 500 6,500 Nil
(E-D)if E > D or 0 otherwise

c. Given that the company follows a constant 60 per cent payout ratio, the dividend per share
and external financing requirement over the 4 year period are given below
Dividend Per Share and External Financing Requirement
(in Rs.)

Year 1 2 3 4

A. Net income 10,000 12,000 9,000 15,00

B. Dividends 6,000 7,200 5,400 9,000

C. Retained earnings 4,000 4,800 3,600 6,000

D. Capital expenditure 8,000 7,000 10,000 8,000

E. External financing
(D-C)if D>C, or 0 4,000 2,200 6,400 2,000
otherwise

F. Dividends per share 1.20 1.44 1.08 1.80

2. Given the constraints imposed by the management, the dividend per share has to
be between Rs.1.00 (the dividend for the previous year) and Rs.1.60 (80 per
cent of earnings per share)
Since share holders have a preference for dividend, the dividend should be
raised over the previous dividend of Rs.1.00 . However, the firm has substantial
investment requirements and it would be reluctant to issue additional equity
because of high issue costs ( in the form of underpricing and floatation costs)
Considering the conflicting requirements, it seems to make sense to pay
Rs.1.20 per share by way of dividend. Put differently the pay out ratio may be
set at 60 per cent.

MINICASE

(a) Plausible Reasons for Paying Dividends

(i) Investor preference for dividends


(ii) Information signaling
(iii) Clientele effect
(iv) Agency costs
Dubious Reasons for Paying Dividends

(i) Bird-in-hand fallacy


(ii) Temporary excess cash
(b)
(i) Funds requirement
(ii) Liquidity
(iii) Access to external sources of financing
(iv) Shareholder preference
(v) Difference in the cost of external equity and retained earnings
(vi) Control
(vi) Taxes
(vii) Stability

(c)
Rs.in million
1 2 3 4 5 Total
Earnings 96 108 84 115 147 550
Net investments 104 94 90 108 192 588
Equity investment 69.33 62.67 60.00 72.00 128.00 392
Pure residual
dividends 26.67 45.33 24.00 43.00 19.00 158
Dividends under
fixed dividend
payout ratio 28.8 32.4 25.2 34.5 44.1 165
Dividends under
smoothed residual
dividend policy 30 30 30 34 34 158

(d)
DPS for the current year : Dt = cr EPSt + (1-c) Dt-1
= 0.6 x 0.3 x 9 + (1-0.6) x 2 = Rs.2.42

(e)
Bonus Issue Stock Split
• The par value of the share is • The par value of the share is
unchanged reduced.
• A part of reserves is capitalised • There is no capitalisation of
reserves
• The shareholders' proportional • The shareholders' proportional
ownership remains unchanged ownership remains unchanged
• The book value per share, the • The book value per share, the
earnings per share, and the market price earnings per share, and the market price
per share decline per share decline
• The market price per share is • The market price per share is
brought within a popular trading range. brought within a more popular trading
range.

Chapter 23
Debt Analysis and Management

1. (i) Initial Outlay


(a) Cost of calling the old bonds
Face value of the old bonds 250,000,000
Call premium 15,000,000
265,000,000
(b) Net proceeds of the new bonds
Gross proceeds 250,000,000
Issue costs 10,000,000

240,000,000
(c) Tax savings on tax-deductible expenses
Tax rate[Call premium+Unamortised issue cost on
the old bonds] 9,200,000
0.4 [ 15,000,000 + 8,000,000]
Initial outlay i(a) – i(b) – i(c) 15,800,000

(ii) Annual Net Cash Savings


(a) Annual net cash outflow on old bonds
Interest expense 42,500,000
- Tax savings on interest expense and amortisation of
issue expenses 17,400,000
0.4 [42,500,000 + 8,000,000/10]
25,100,000
(b) Annual net cash outflow on new bonds
Interest expense 37,500,000
- Tax savings on interest expense and amortisation of
issue cost 15,500,000
0.4 [ 37,500,000 – 10,000,000/8]
22,000,000
Annual net cash savings : ii(a) – ii(b) 3,100,000

(iii) Present Value of the Annual Cash Savings


Present value of an 8-year annuity of 3,100,000 at a
discount rate of 9 per cent which is the post –tax cost
of new bonds 3,100,000 x 5.535 17,158,500

(iv) Net Present Value of Refunding the Bonds


(a) Present value of annual cash savings 17,158,500
(b) Net initial outlay 15,800,000
(c) Net present value of refunding the bonds :
iv(a) – iv(b). 1,358,500
2. (i) Initial Outlay
(a) Cost of calling the old bonds
Face value of the old bonds 120,000,000
Call premium 4,800,000

124,800,000
(b) Net proceeds of the new issue
Gross proceeds 120,000,000
Issue costs 2,400,000

117,600,000
(c) Tax savings on tax-deductible expenses 3,120,000
Tax rate[Call premium+Unamortised issue costs on
the old bond issue]
0.4 [ 4,800,000 + 3,000,000]
Initial outlay i(a) – i(b) – i(c) 4,080,000

(ii) Annual Net Cash Savings


(a) Annual net cash out flow on old bonds
Interest expense 19,200,000
- Tax savings on interest expense and amortisation of
issue costs 7,920,000
0.4[19,200,000 + 3,000,000/5]
11,280,000

(b) Annual net cash outflow on new bonds


Interest expense 18,000,000
- Tax savings on interest expense and amortistion of issue
costs 7,392,000
0.4[18,000,000 + 2,400,000/5]
10,608,000
Annual net cash savings : ii(a) – ii(b) 672,000

(iii) Present Value of the Annual Net Cash Savings


Present value of a 5 year annuity of 672,000 at
as discount rate of 9 per cent, which is the post-tax 2,614,080
cost of new bonds

(iv) Net Present Value of Refunding the Bonds


(a) Present value of annual net cash savings 2,614,080
(b) Initial outlay 4,080,000
(c) Net present value of refunding the bonds : - 1,466,000
iv(a) – iv(b)

3. Yield to maturity of bond P


8 160 1000
918.50 =∑ +
t=1 (1+r)t (1+r)8

r or yield to maturity is 18 percent

Yield to maturity of bond Q


5 120 1000
761 = ∑ +
t=1 (1+r)t (1+r)5

r or yield to maturity is 20 per cent

Duration of bond P is calculated below

Year Cash flow Present Value Proportion of Proportion of bond’s


at 18% bond’s value Value x Time

1 160 135.5 0.148 0.148


2 160 114.9 0.125 0.250
3 160 97.4 0.106 0.318
4 160 82.6 0.090 0.360
5 160 69.9 0.076 0.380
6 160 59.2 0.064 0.384
7 160 50.2 0.055 0.385
8 160 308.6 0.336 2.688

4.913

Duration of bond Q is calculated below

Year Cash flow Present Value Proportion of Proportion of bond’s


at 20% bond’s value Value x Time

1 120 100.0 0.131 0.131


2 120 83.2 0.109 0.218
3 120 69.5 0.091 0.273
4 120 57.8 0.076 0.304
5 1120 450.2 0.592 2.960
3.886

Volatility of bond P Volatility of bond Q


4.913 3.886
= 4.16 = 3.24
1.18 1.20

4. The YTM for bonds of various maturities is

Maturity YTM(%)

1 12.36

2 13.10

3 13.21

4 13.48

5 13.72

Graphing these YTMs against the maturities will give the yield curve

The one year treasury bill rate , r1, is

1,00,000
- 1 = 12.36 %
89,000

To get the forward rate for year 2, r2, the following equation may be set up :

12500 112500
99000 = +
(1.1236) (1.1236)(1+r2)

Solving this for r2 we get r2 = 13.94%

To get the forward rate for year 3, r3, the following equation may be set up :

13,000 13,000 113,000


99,500 = + +
(1.1236) (1.1236)(1.1394) (1.1236)(1.1394)(1+r3)
Solving this for r3 we get r3 = 13.49%

To get the forward rate for year 4, r4 , the following equation may be set up :

13,500 13,500 13,500


100,050 = + +
(1.1236) (1.1236)(1.1394) (1.1236)(1.1394)(1.1349)

113,500
+
(1.1236)(1.1394)(1.1349)(1+r4)

Solving this for r4 we get r4 = 14.54%

To get the forward rate for year 5, r5 , the following equation may be set up :

13,750 13,750 13,750


100,100 = + +
(1.1236) (1.1236)(1.1394) (1.1236)(1.1394)(1.1349)

13,750
+
(1.1236)(1.1394)(1.1349)(1.1454)

113,750
+
(1.1236)(1.1394)(1.1349)(1.1454)(1+r5)

Solving this for r5 we get r5 = 15.08%


Chapter 24
LEASING, HIRE PURCHASE, AND PROJECT FINANCE

1
Year
0 1 2 3 4 5
Cost of the
1 asset 1,500,000
2 Depreciation 499,950.00 333,316.67 222,222.22 148,155.55 98,775.31
Loss of
depreciation - -
3 tax shield 166,633.34 111,094.44 -74,066.67 -49,380.25 -32,921.81
Lease
4 payment -420,000 -420,000 -420,000 -420,000 -420,000
Tax shield
on lease
5 payment 139,986.00 139,986.00 139,986.00 139,986.00 139,986.00
Loss of
salvage -
6 value 300,000.00
Cash flow of - - - - -
7 lease 1,080,000. 446,647.34 391,108.44 354,080.67 329,394.25 192,935.81
NAL of lease
446,647.34 391,108.44 354,080.67 329,394.25 192,935.81
= 1,080,000 - - - - -
1.08 ( 1.08)2 ( 1.08)3 ( 1.08)4 ( 1.08)5

= 1,080,000 – 413,562.35 – 335,312.45 – 281,080.65 – 242,114.61 – 131,308.87


= - 323,378.93

.
2. Under the hire purchase proposal the total interest payment is
2,000,000 x 0.12 x 3 = Rs. 720,000
The interest payment of Rs. 720,000 is allocated over the 3 years period using
the sum of the years digits method as follows:
Year Interest allocation

366
1 x Rs.720,000 = Rs.395,676
666

222
2 x Rs.720,000 = Rs.240,000
666

78
3 x Rs.720,000 = Rs.84,324
666

The annual hire purchase instalments will be :

Rs.2,000,000 + Rs.720,000
= Rs.906,667
3

The annual hire purchase instalments would be split as follows

Year Hire purchase instalment Interest Principal repayment


1 Rs.906,667 Rs.395,676 Rs. 510,991
2 Rs.906,667 Rs.240,000 Rs. 666,667
3 Rs.906,667 Rs. 84,324 Rs. 822,343

The lease rental will be as follows :


Rs. 560,000 per year for the first 5 years
Rs. 20,000 per year for the next 5 years
The cash flows of the leasing and hire purchse options are shown below

Year Leasing Hire Purchase -It(1-tc)-PRt+


- LRt (1-tc) -It(1-tc) -PRt Dt(tc) NSVt Dt(tc)+NSVt

1 -560,000(1-.4)=-336,000 -395,676(1-.4) -510,991 500,000(0.4) -548,397


2 -560,000(1-.4)=-336,000 -240,000(1-.4) -666,667 375,000(0.4) -660,667
3 -560,000(1-.4)=-336,000 - 84,324(1-.4) -822,343 281,250(0.4) -760,437
4 -560,000(1-.4)=-336,000 210,938(0.4) 84,375
5 -560,000(1-.4)=-336,000 158,203(0.4) 63,281
6 - 20,000(1-.4)= - 12,000 118,652(0.4) 47,461
7 - 20,000(1-.4)= - 12,000 88,989(0.4) 35,596
8 - 20,000(1-.4)= - 12,000 66,742(0.4) 26,697
9 - 20,000(1-.4)= - 12,000 50,056(0.4) 20,023
10 - 20,000(1-.4)= - 12,000 37,542(0.4) 200,000 215,017

Present value of the leasing option

5 336,000 10 12,000
= -∑ − ∑ = - 1,302,207
t=1 (1.10)t t=6 (1.10)t

Present value of the hire purchase option

548,397 660,667 760,437 84,375


=- - - -
(1.10) (1.10)2 (1.10)3 (1.10)4

63,281 47,461 35,596 26,697


+ + +
(1.10)5 (1.10)6 (1.10)7 (1.10)8

20,023 215,017
+
(1.10.9 (1.10)10

= - 1,369,383

Since the leasing option costs less than the hire purchase option , Apex should choose the
leasing option.
MINICASE

(a)
Year 1 2 3 4 5 6 7 8 9 10
Principal repayment -6 -6 -6 -6 -6
Interest payment -3.6 -2.88 -2.16 -1.44 -0.72
Depreciation 12 7.20 4.32 2.59 1.56 0.93 0.56 0.34 0.20 0.12
Tax shield on depn. 4.00 2.40 1.44 0.86 0.52 0.31 0.19 0.11 0.07 0.04
Post tax interest
payment -2.4 -1.92 -1.44 -0.96 -0.48
Net salvage value 6
Net cash flow -4.40 -5.52 -6.00 -6.10 -5.96 0.31 0.19 0.11 0.07 6.04
Present value of the cash ‘ borrowing cum buying option’ is
4.40 5.52 6.00 6.10 5.96 0.31 0.19 0.11 0.07 6.04
= - ----- - ------ - ------ - ----- - ----- + ----- + ------ + ------ + ------- + ------
(1.08) (1.08)2 (1.08)3 (1.08)4 (1.08)5 (1.08)6 (1.08)7 (1.08)8 (1.08)9 (1.08)10
= - 4.07 – 4.73 – 4.76 – 4.48 – 4.06 + 0.20 + 0.11 + 0.06 + 0.04 + 2.80
= - 18.89 million

(b)
Present value of lease cash flows =-7(1-0.3333)PVIFA8%, 5years –0.5(1- 0.3333)PVIFA8%, 5years
PVIF8% , 5years

= -7 x 0.6667 x 3.993 – 0.5 x 0.6667 x 3.993 x 0.681 =


-19.54million

(c) Total interest =30,000,000 x 0.08 x 3 = Rs.720,000


Monthly HP instalment = (30,000,000 + 720,000) / 36 = Rs.853,333
Annual instalment = (30,000,000 + 720,000) / 3 = Rs.10,240,000
Proportions for interest allocation:
36 +35+-----------------+ 25 366
I year = =
36 +35+-----------------+ 1 666

24 +23 +-----------------+13 222


II year = =
36 +35+-----------------+ 1 666
12 +35+-----------------+ 1 78
III year = =
36 +35+-----------------+ 1 666
Interest allocations for the three years:

I year = 720,000 x 366/666 = Rs.395,676


II year = 720,000 x 222/666 = Rs.240,000
IIIyear = 720,000 x 78/666 = Rs .84,324
The cash flows under the HP option:
Year -It(1-Tc) -Pr Dt(Tc) NSV Total CF PVIF PV
1 -263,797 -9,844,324 3,999,600 -6,108,521 0.9259 -5,656,038
2 -160,008 -10,000,000 2,399,760 -7,760,248 0.8573 -6,653,162
3 -56,219 -10,155,676 1,439,856 -8,772,039 0.7938 -6,963,527
4 863,914 863,914 0.7350 635,002
5 518,348 518,348 0.6806 352,779
6 311,009 311,009 0.6302 195,988
7 186,605 186,605 0.5835 108,882
8 111,963 111,963 0.5403 60,490
9 67,178 67,178 0.5002 33,606
10 40,307 6,000,000 6,040,307 0.4632 2,797,831
---
15,088,148
Total=

Present value of the cash flows under the HP option = - Rs.15.09 million
Chapter 25
HYBRID FINANCING
1.
l (S/E) + (r + σ2 /2) t
d1 =
σ √ t

= ln (40 / 25) + [0.16 + (0.35)2/2]2


0.35(2)1/2

= 0.4700 + 0.4425
0.4950

= 1.8434

d2 = d1 - σ √ t
= 1.8434 – 0.35√2
= 1.3484

N(d1) = N (1.8434).
From the tables we have N(1.80) = 1- 0.0359 = 0.9641
and N(1.85)= 1- 0.0322= 0.9678
By linear extrapolation, we get
N(1.8434) = 0.9641 + (1.8434 – 1.8000)(0.9678-0.9641)/0.05
= 0.9641 + 0.003212 = 0.9673
N(d2) = N(1.3484)
From the tables we have N(1.30) = 1- 0.0968 = 0.9032
N(1.35) = 1- 0.0885 = 0.9115
By linear extrapolation, we get
N(1.3484) = 0.9032 + (1.3484 – 1.3000)(0.9115 – 0.9032)/0.05
= 0.9032 + 0.008034 = 0.9112
E/ert = 25/1.3771 = 18.1541
C = So N(d1) – E. e-rt. N(d2)
= 40 x 0.9673 – 18.1541 x 0.9112= 22.15
Value of the warrant is Rs.22.15.

2
l (S/E) + (r + σ2 /2) t
d1 =
σ √ t

= ln (50 / 30) + [0.12 + (0.4)2/2]2


0.4(2)1/2
= 0.5108 + 0.4
0.5657

= 1.6100

d2 = d1 - σ √ t
= 1.6100 – 0.40√2
= 1.0443

N(d1) = N (1.6100).
From the tables we have N(1.60) = 1- 0.0548 = 0.9452
and N(1.65)= 1- 0.0495= 0.9505
By linear extrapolation, we get
N(1.6100) = 0.9452 + (1.6100 – 1.6000)(0.9505-0.9452)/0.05
= 0.9452 + 0.00106 = 0.9463
N(d2) = N(1.0443)
From the tables we have N(1.00) = 1- 0.1587 = 0.8413
N(1.05) = 1- 0.1469 = 0.8531
By linear extrapolation, we get
N(1.0443) = 0.8413 + (1.0443 – 1.0000)(0.8531 – 0.8413)/0.05
= 0.8413 + 0.01045 = 0.8517
E/ert = 30/1.2712 = 23.60
C = So N(d1) – E. e-rt. N(d2)
= 50 x 0.9463 – 23.60 x 0.8517= 27.21
Value of the warrant = Rs.27.21

3.
(a) No.of shares after conversion in one year = 2
Value of the shares at the price of Rs.150 = 2 x 150 = Rs.300
PV of the convertible portion at the required rate of 15% = 300/1.15 = Rs.260.87
Payments that would be received from the debenture portion:
Year Payments PVIF10%,t PV
1 60 0.909 54.55
2 40 0.826 33.06 Value of the convertible debenture = 260.87 +
3 40 0.751 30.05 418.18 = Rs. 679.05
4 40 0.683 27.32
5 240 0.621 149.02
6 220 0.564 124.18
Total= 418.18
(b)
The cash flow for Shiva is worked out as under:
Year Cash flow
0 600
-
1 =-240-60*(1-0.3) 282
2 =-40*(1-0.3) -28
3 =-40*(1-0.3) -28
4 =-40*(1-0.3) -28
-
5 =-40*(1-0.3)-200 228
-
6 =-20*(1-0.3)-200 214

The post-tax cost of the convertible debenture to Shiva is the IRR of the above
cash flow stream.
Let us try a discount rate of 9%. The PV of the cash flow will then be
= 600 – 282/(1.09) -28/(1.09)2 - 28/(1.09)3 -28/(1.09)4-228/(1.09)5-214/(1.09)6
= 600 – 258.72 – 23.57 – 21.62 – 19.84 – 148.18 – 127.60 = 0.47 which is very near to zero.
So the post –tax cost of the convertible debenture to Shiva is 9%

Chapter 26
WORKING CAPITAL POLICY

Average inventory
1 Inventory period =
Annual cost of goods sold/365

(60+64)/2
= = 62.9 days
360/365

Average accounts receivable


Accounts receivable =
period Annual sales/365

(80+88)/2
= = 61.3 days
500/365

Average accounts payable


Accounts payable =
period Annual cost of goods sold/365
(40+46)/2
= = 43.43 days
360/365

Operating cycle = 62.9 + 61.3 = 124.2 days


Cash cycle = 124.2 – 43.43 = 80.77 days

(110+120)/2
2. Inventory period = = 56.0 days
750/365

(140+150)/2
Accounts receivable = = 52.9 days
period 1000/365

(60+66)/2
Accounts payable = = 30.7 days
period 750/365

Operating cycle = 56.0 + 52.9 = 108.9 days


Cash cycle = 108.9 – 30.7 = 78.2 days

3. This is a repetition of the solved problem 26.1 .


Chapter 27
CASH AND LIQUIDITY MANAGEMENT

1 The projected cash inflows and outflows for the quarter, January through March, is shown
below .

Month December January February March


(Rs.) (Rs.) (Rs.) (Rs.)

Inflows :
Sales collection 50,000 55,000 60,000

Outflows :
Purchases 22,000 20,000 22,000 25,000
Payment to sundry creditors 22,000 20,000 22,000
Rent 5,000 5,000 5,000
Drawings 5,000 5,000 5,000
Salaries & other expenses 15,000 18,000 20,000
Purchase of furniture - 25,000 -

Total outflows(2to6) 47,000 73,000 52,000

Given an opening cash balance of Rs.5000 and a target cash balance of Rs.8000, the
surplus/deficit in relation to the target cash balance is worked out below :

January February March


(Rs.) (Rs.) (Rs.)

1. Opening balance 5,000


2. Inflows 50,000 55,000 60,000
3. Outflows 47,000 73,000 52,000
4. Net cash flow (2 - 3) 3,000 (18,000) 8,000
5. Cumulative net cash flow 3,000 (15,000) (7,000)
6. Opening balance + Cumulative
net cash flow 8,000 (10,000) (2,000)
7. Minimum cash balance required 8,000 8,000 8,000
8. Surplus/(Deficit) - (18,000) (10,000)
2 The balances in the books of Datta co and the books of the bank are shown below:

(Rs.)
1 2 3 4 5 6 7 8 9 10
Books of
Datta
Co:

Op 30,00 46,00 62,00 78,000 94,00 1,10,00 1,26,00 1,42,00 1,58,00 1,74,000
ening 0 0 0 0 0 0 0 0
Balance
Add: 20,00 20,00 20,00 20,000 20,00 20,00 20,00 20,00 20,00 20,000
Cheque 0 0 0 0 0 0 0 0
received
Less: 4,00 4,00 4,00 4,000 4,00 4,00 4,00 4,00 4,00 4,000
Cheque 0 0 0 0 0 0 0 0
issued
Clo 46,00 62,00 78,00 94,000 1,10,00 1,26,00 1,42,00 1,58,00 1,74,00 1,90,000
sing 0 0 0 0 0 0 0 0
Balance

Books of
the
Bank:

Op 30,00 30,00 30,00 30,000 30,000 30,000 50,000 70,000 90,000 1,06,000
ening 0 0 0
Balance
Add: - - - - - 20,000 20,000 20,000 20,000 20,000
Cheques
realised
Less: - - - - - - - - 4,000 4,000
Cheques
debited
Clo 30,00 30,00 30,00 30,000 30,000 50,000 70,000 90,000 1,06,00 1,22,000
sing 0 0 0 0
Balance

From day 9 we find that the balance as per the bank’s books is less than the balance as per Datta
Company’s books by a constant sum of Rs.68,000. Hence in the steady situation Datta Company
has a negative net float of Rs.68,000.
3. Optimal conversion size is
2bT
C =
I
b = Rs.1200, T= Rs.2,500,000, I = 5% (10% dividend by two)

So,
2 x 1200 x 2,500,000
C = = Rs.346,410
0.05

4.
2
3 3 bσ
RP = + LL
4I

UL = 3 RP – 2 LL

I = 0.12/360 = .00033, b = Rs.1,500, σ = Rs.6,000, LL = Rs.100,000

3 3 x 1500 x 6,000 x 6,000


RP = + 100,000
4 x .00033

= 49,695 + 100,000 = Rs.149,695

UL = 3RP – 2LL = 3 x 149,695 – 2 x 100,000


= Rs.249,085

5
. Optimal conversion size is
2bT
C =
I
b = Rs.2800, T= Rs.35,000,000, I = 5% (10% dividend by two)
So,
2 x 2800 x 35,000,000
C = = Rs.1,979,899
0.05
6

2
3 3 bσ
RP = + LL
4I

UL = 3 RP – 2 LL

I = 0.12/360 = .00033, b = Rs.3,200, σ = Rs.22,000, LL = Rs.800,000

3 3 x 3200 x 22,000 x 22,000


RP = + 800,000
4 x .00033

= 152,118 + 800,000 = Rs.952,118

UL = 3RP – 2LL = 3 x 952,118 – 2 x 800,000


= Rs.1,256,354
Chapter 28
CREDIT MANAGEMENT

1. Δ RI = [ΔS(1-V)- ΔSbn](1-t)- k ΔI
ΔS
ΔI = x ACP x V
360
Δ S = Rs.10 million, V=0.85, bn =0.08, ACP= 60 days, k=0.15, t = 0.40

Hence, ΔRI = [ 10,000,000(1-0.85)- 10,000,000 x 0.08 ] (1-0.4)

-0.15 x 10,000,000 x 60 x 0.85

360
= Rs. 207,500

2. Δ RI = [ΔS(1-V)- ΔSbn] (1-t) – k Δ I

So ΔS
Δ I = (ACPN – ACPo) +V(ACPN)
360 360

ΔS=Rs.1.5 million, V=0.80, bn=0.05, t=0.45, k=0.15, ACPN=60, ACPo=45, So=Rs.15 million
Hence ΔRI = [1,500,000(1-0.8) – 1,500,000 x 0.05] (1-.45)

-0.15 (60-45) 15,000,000 + 0.8 x 60 x 1,500,000

360 360
= 123750 – 123750 = Rs. 0

3. Δ RI = [ΔS(1-V) –Δ DIS ] (1-t) + k Δ I


Δ DIS = pn(So+ΔS)dn – poSodo

So ΔS
ΔI = (ACPo-ACPN) - x ACPN x V
360 360

So =Rs.12 million, ACPo=24, V=0.80, t= 0.50, r=0.15, po=0.3, pn=0.7,


ACPN=16, ΔS=Rs.1.2 million, do=.01, dn= .02
Hence
ΔRI = [ 1,200,000(1-0.80)-{0.7(12,000,000+1,200,000).02-
0.3(12,000,000).01}](1-0.5)
12,000,000 1,200,000
+ 0.15 (24-16) - x 16 x 0.80
360 360

= Rs.79,200

4. Δ RI = [ΔS(1-V)- ΔBD](1-t) –kΔ I


ΔBD=bn(So+ΔS) –boSo

So ΔS
ΔI = (ACPN –ACPo) + x ACPN x V
360 360

So=Rs.50 million, ACPo=25, V=0.75, k=0.15, bo=0.04, ΔS=Rs.6 million,


ACPN=40 , bn= 0.06 , t = 0.3

ΔRI = [ Rs.6,000,000(1-.75) –{.06(Rs.56,000,000)-.04(Rs.50,000,000)](1-0.3)

Rs.50,000,000 Rs.6,000,000
- 0.15 (40-25) + x 40 x 0.75
360 360

= - Rs.289,495

5. 30% of sales will be collected on the 10th day


70% of sales will be collected on the 50th day
ACP = 0.3 x 10 + 0.7 x 50 = 38 days

Rs.40,000,000
Value of receivables = x 38
360

= Rs.4,222,222
Assuming that V is the proportion of variable costs to sales, the investment in
receivables is :
Rs.4,222,222 x V

6. 30% of sales are collected on the 5th day and 70% of sales are collected on the
25th day. So,
(a) ACP = 0.3 x 5 + 0.7 x 25 = 19 days

Rs.10,000,000
Value of receivables = x 19
360

= Rs.527,778
(b) Investment in receivables = 0.7 x 527,778
= Rs.395,833

7. Since the change in credit terms increases the investment in receivables,


ΔRI = [ΔS(1-V)- ΔDIS](1-t) – kΔI
So=Rs.50 million, ΔS=Rs.10 million, do=0.02, po=0.70, dn=0.03,pn=0.60,
ACPo=20 days, ACPN=24 days, V=0.85, k=0.12 , and t = 0.40
ΔDIS = 0.60 x 60 x 0.03 – 0.70 x 50 x 0.2
= Rs.0.38 million

50 10
ΔI= (24-20) + x 24 x 0.85
360 360

= Rs.1.2222 million
Δ RI = [ 10,000,000 (1-.85) – 380,000 ] (1-.4) – 0.12 x 1,222,222
= Rs.525,333

8. The decision tree for granting credit is as follows :

Customer pays(0.95)
Grant credit Profit 1500
Customer pays(0.85)
Grant credit Customer defaults(0.05)
Profit 1500 Refuse credit
Loss 8500
Customer defaults(0.15)
Loss 8500
Refuse credit

The expected profit from granting credit, ignoring the time value of money, is :

Expected profit on + Probability of payment x Expected profit on


Initial order and repeat order repeat order

{ 0.85(1500)-0.15(8500)} + 0.85 {0.95(1500)-.05(8500)}


= 0 + 850 = Rs.850
9. Profit when the customer pays = Rs.10,000 - Rs.8,000 = Rs.2000
Loss when the customer does not pay = Rs.8000
Expected profit = p1 x 2000 –(1-p1)8000
Setting expected profit equal to zero and solving for p1 gives :
p1 x 2000 – (1- p1)8000 = 0 p1 = 0.80
Hence the minimum probability that the customer must pay is 0.80

MINICASE
Solution:

Present Data
• Sales : Rs.800 million
• Credit period : 30 days to those deemed eligible
• Cash discount : 1/10, net 30
• Proportion of credit sales and cash sales are 0.7 and 0.3. 50 percent of the credit customers
avail of cash discount
• Contribution margin ratio : 0.20
• Tax rate : 30 percent
• Post-tax cost of capital : 12 percent
• ACP on credit sales : 20 days

Effect of Relaxing the Credit Standards on Residual Income

Incremental sales : Rs.50 million


Bad debt losses on incremental sales: 12 percent
ACP remains unchanged at 20 days

∆RI = [∆S(1 – V) - ∆Sbn] (1 – t) – R ∆ I

∆S
where ∆ I = x ACP x V
360

∆ RI = [50,000,000 (1-0.8) – 50,000,000 x 0.12] (1 – 0.3)

50,000,000
- 0.12 x x 20 x 0.8
360

= 2,800,000 – 266,667 = 2,533,333

Effect of Extending the Credit Period on Residual Income


∆ RI = [∆S(1 – V) - ∆Sbn] (1 – t) – R ∆ I

So ∆S
where ∆I = (ACPn – ACPo) + V (ACPn)
360 360

∆RI = [50,000,000 (1 – 0.8) – 50,000,000 x 0] (1 – 0.3)

800,000,000 50,000,000
- 0.12 (50 – 20) x + 0.8 x 50 x
360 360

= 7,000,000 – 8,666,667
= - Rs.1,666,667

Effect of Relaxing the Cash Discount Policy on Residual Income

∆RI = [∆S (1 – V) - ∆ DIS] (1 – t) + R ∆ I


where ∆ I = savings in receivables investment
So ∆S
= (ACPo – ACPn) – V x ACPn
360 360

800,000,000 20,000,000
= (20 – 16) – 0.8 x x 16
360 360

= 8,888,889 – 711,111 = 8,177,778

∆ DIS = increase in discount cost


= pn (So + ∆S) dn – po So do
= 0.7 (800,000,000 + 20,000,000) x 0.02 – 0.5 x 800,000,000 x 0.01
= 11,480,000 – 4,000,000 = 7,480,000

So, ∆RI = [20,000,000 (1 – 0.8) – 7,480,000] (1 – 0.3) + 0.12 x 8,177,778


= - 2,436,000 + 981,333
= - 1,454,667
Chapter 29
INVENTORY MANAGEMENT

1.
a. No. of Order Ordering Cost Carrying Cost Total Cost
Orders Per Quantity (U/Q x F) Q/2xPxC of Ordering
Year (Q) (where and Carrying
(U/Q) PxC=Rs.30)
Units Rs. Rs. Rs.

1 250 200 3,750 3,950


2 125 400 1,875 2,275
5 50 1,000 750 1,750
10 25 2,000 375 2,375

2 UF 2x250x200
b. Economic Order Quantity (EOQ) = =
PC 30
2UF = 58 units (approx)
2. a EOQ =
PC
U=10,000 , F=Rs.300, PC= Rs.25 x 0.25 =Rs.6.25

2 x 10,000 x 300
EOQ = = 980
6.25
10000
b. Number of orders that will be placed is = 10.20
980
Note that though fractional orders cannot be placed, the number of orders
relevant for the year will be 10.2 . In practice 11 orders will be placed during the year.
However, the 11th order will serve partly(to the extent of 20 percent) the present year and
partly(to the extent of 80 per cent) the following year. So only 20 per cent of the ordering cost
of the 11th order relates to the present year. Hence the ordering cost for the present year will be
10.2 x Rs.300

c. Total cost of carrying and ordering inventories


980
= [ 10.2 x 300 + x 6.25 ] = Rs.6122.5
2
3. U=6,000, F=Rs.400 , PC =Rs.100 x 0.2 =Rs.20

2 x 6,000 x 400
EOQ = = 490 units
20

U U Q’(P-D)C Q* PC
Δπ = UD + - F- -
Q* Q’ 2 2

6,000 6,000
= 6000 x .5 + - x 400
490 1,000

1,000 (95)0.2 490 x 100 x 0.2


- -
2 2

= 30,000 + 2498 – 4600 = Rs.27898

4. U=5000 , F= Rs.300 , PC= Rs.30 x 0.2 = Rs.6

2 x 5000 x 300
EOQ = = 707 units
6
If 1000 units are ordered the discount is : .05 x Rs.30 = Rs.1.5 Change in
profit when 1,000 units are ordered is :

5,000 5,000
Δπ = 5000 x 1.5 + - x 300
707 1,000

1000 x 28.5 x 0.2 707 x 30 x 0.2


- - = 7500 + 622-729 =Rs.7393
2 2

If 2000 units are ordered the discount is : .10 x Rs.30 = Rs.3 Change in profit
when 2,000 units are ordered is :
5000 5000 2000x27x0.2 707x30x0.2
Δπ = 5000 x 3.0 + - x 300- -
707 2000 2 2

= 15,000 +1372 – 3279 = Rs.13,093

5. The quantities required for different combinations of daily usage rate(DUR) and lead
times(LT) along with their probabilities are given in the following table

LT
(Days)
DUR 5(0.6) 10(0.2) 15(0.2)
(Units)

4(0.3) 20*(0.18) 40(0.06) 60(0.06)


6(0.5) 30 (0.30) 60(0.10) 90(0.10)
8(0.2) 40 (0.12) 80(0.04) 120(0.04)
*
Note that if the DUR is 4 units with a probability of 0.3 and the LT is 5 days with
a probability of 0.6, the requirement for the combination DUR = 4 units and LT =
5 days is 20 units with a probability of 0.3x0.6 = 0.18. We have assumed that the
probability distributions of DUR and LT are independent. All other entries in the
table are derived similarly.
The normal (expected) consumption during the lead time is :
20x0.18 + 30x0.30 + 40x0.12 + 40x0.06 + 60x0.10 + 80x0.04 + 60x0.06 + 90x0.10
+ 120x0.04 = 46.4 tonnes
a. Costs associated with various levels of safety stock are given below :

Safety Stock Stock out Probability Expected Carrying Total Cost


Stock* outs(in Cost Stock out Cost
tonnes)

1 2 3 4 5 6 7
[3x4] [(1)x1,000] [5+6]

Tonnes Rs. Rs. Rs.


73.6 0 0 0 0 73,600 73,600
43.6 30 120,000 0.04 4,800 43,600 48,400

33.6 10 40,000 0.10


40 160,000 0.04 10,400 33,600 44,000

13.6 20 80,000 0.04


30 120,000 0.10 24,800 13,600 38,400
60 240,000 0.04

0 13.6 54,400 0.16


33.6 134,400 0.04 43,296 0 43,296
43.6 174,400 0.10
73.6 294,400 0.04
*
Safety stock = Maximum consumption during lead time – Normal
consumption during lead time
So the optimal safety stock= 13.6 tonnes
Reorder level = Normal consumption during lead time + safety stock
K= 46.4 + 13.6 = 60 tonnes

b. Probability of stock out at the optimal level of safety stock = Probability


(consumption being 80 or 90 or 120 tonnes)

Probability (consumption = 80 tonnes) + Probability (consumption = 90 tonnes) +


Probability (consumption = 120 tonnes)
= 0.04 +0.10+0.04 = 0.18
6.

Item Annual Usage Price per Annual Ranking


(in Units) Unit Usage Value
Rs. Rs.

1 400 20.00 8,000 6


2 15 150.00 2,250 10
3 6,000 2.00 12,000 5
4 750 18.00 13,500 4
5 1,200 25.00 30,000 1
6 25 160.00 4,000 9
7 300 2.00 600 14
8 450 1.00 450 15
9 1,500 4.00 6,000 7
10 1,300 20.00 26,000 2
11 900 2.00 1,800 11
12 1,600 15.00 24,000 3
13 600 7.50 4,500 8
14 30 40.00 1,200 12
15 45 20.00 900 13

1,35,200
Cumulative Value of Items & Usage

Item Rank Annual Cumulative Cumulative Cumulative


No. UsageValue Annual Usage % of Usage % of Items
(Rs.) Value (Rs.) Value

5 1 30,000 30,000 22.2 6.7


10 2 26,000 56,000 41.4 13.3
12 3 24,000 80,000 59.2 20.0
4 4 13,500 93,500 69.2 26.7
3 5 12,000 105,500 78.0 33.3
1 6 8,000 113,500 83.9 40.0
9 7 6,000 119,500 88.4 46.7
13 8 4,500 124,000 91.7 53.3
6 9 4,000 128,000 94.7 60.0
2 10 2,250 130,250 96.3 66.7
11 11 1,800 132,050 97.7 73.3
14 12 1,200 133,250 98.6 80.0
15 13 900 134,150 99.2 86.7
7 14 600 134,750 99.7 93.3
8 15 450 135,200 100.0 100.0

Class No. of Items % to the Total Annual Usage % to Total Value


Value Rs.

A 4 26.7 93,500 69.2


B 3 20.0 26,000 19.2
C 18 53.3 15,700 11.6

15 135,200
7. The quantities required for different combinations of daily usage rate(DUR) and lead
times(LT) along with their probabilities are given in the following table

LT
(Days)
DUR 5(0.4) 8(0.4) 12(0.2)
(Units)

2(0.2) 10 (0.08) 16(0.08) 24(0.04)


3(0.6) 15 (0.24) 24(0.24) 36(0.12)
4(0.2) 20 (0.08) 32(0.08) 48(0.04)

The normal (expected) consumption during the lead time is :


10x0.08 + 15 x0.24 + 20x0.08 + 16x0.08 + 24x0.24 + 32 x0.08 + 24x0.04 + 36
x0.12 + 48 x0.04 = 22.8 tonnes

c. Costs associated with various levels of safety stock are given below :

Safety Stock Stock out Probability Expected Carrying Total Cost


Stock* outs(in Cost Stock out Cost
tonnes)

1 2 3 4 5 6 7
[3x4] [(1)x1,500] [5+6]

Tonnes Rs. Rs. Rs.


25.2 0 0 0 0 37,800 37,800
13.2 12 60,000 0.04 2,400 19,800 22,200

9.2 4 20,000 0.12


16 80,000 0.04 5,600 13,800 19,400

1.2 8 40,000 0.08


12 60,000 0.12 15,200 1,800 17,000
24 120,000 0.04

0 1.2 6,000 0.28


9.2 46,000 0.08 18,320 0 18,320
13.2 66,000 0.12
25.2 126,000 0.04
*
Safety stock = Maximum consumption during lead time – Normal
consumption during lead time
a) So the optimal safety stock= 1.2 tonnes
Reorder level = Normal consumption during lead time + safety stock
K= 22.8 + 1.2 = 24 tonnes

b) Probability of stock out at the optimal level of safety stock = Probability


(consumption being 32 or 36 or 48 tonnes)

= 0.08 +0.12+0.04 = 0.24


Chapter 30
WORKING CAPITAL FINANCING

1. Annual interest cost is given by ,


Discount % 360
x
1- Discount % Credit period – Discount period

Therefore, the annual per cent interest cost for the given credit terms will be as
follows:

a. 0.01 360
x = 0.182 = 18.2%
0.99 20

b. 0.02 360
x = 0.367 = 36.7%
0.98 20

c. 0.03 360
x = 0.318 = 31.8%
0.97 35

d. 0.01 360
x = 0.364 = 36.4%
0.99 10

2.
a.
0.01 360
x = 0.104 = 10.4%
0.99 35

b. 0.02 360
x = 0.21 = 21%
0.98 35

c. 0.03 360
x = 0.223 = 22.3%
0.97 50
d. 0.01 360
x = 0.145 = 14.5%
0.99 25
3. The maximum permissible bank finance under the three methods suggested by
The Tandon Committee are :

Method 1 : 0.75(CA-CL) = 0.75(36-12) = Rs.18 million


Method 2 : 0.75(CA)-CL = 0.75(36-12 = Rs.15 million
Method 3 : 0.75(CA-CCA)-CL = 0.75(36-18)-12 = Rs.1.5 million

4. Raw material and stores and spares consumed (RMC)= Opening stock of raw materials and
stores and spares + purchases – closing stock of raw materials and stores and spares
= 524 + 1821 – 540 = 1805
Cost of production = RMC + Other operating expenses(including depreciation) +
Opening stock of work-in-process – Closing stock of work-in-process
= 1805 + 674 + 218 – 226 = 2471
Cost of sales = Cost of production + Opening stock of finished goods – Closing stock of finished
goods
= 2471 + 485 – 588 = 2368
Holding level of raw material and stores and spares(months consumption)
=( 540 x 12) / 1805 = 3.59 months
Holding level of work-in-process ( months cost of production)
= ( 226 x 12) / 2471 = 1.10 months
Holding level of finished goods(months cost of sales)
= (588 x 12)/ 2368 = 2.98 months
Chapter 31
WORKING CAPITAL MANAGEMENT :EXTENSIONS

1.(a) The discriminant function is :

Zi = aXi + bYi
where Zi = discriminant score for the ith account
Xi = quick ratio for the ith account
Yi = EBDIT/Sales ratio for the ith account

The estimates of a and b are :


σ y2. dx - σ xy . dy
a =
σ x 2. σ y 2 - σ xy .σ xy

σ x
2
. dy − σ xy . dx
b =
σ x
2
. σ y
2
− σ xy . σ xy

The basic calculations for deriving the estimates of a and b are given
the accompanying table.

Drawing on the information in the accompanying table we find that

∑Xi = 19.81 ∑Yi= 391 ∑(Xi-X)2 ∑( Yi-Y)2 ∑( Xi-X)(Yi-Y)

X = 0.7924 Y = 15.64 = 0.8311 =1661.76 = 10.007

Account Xi Yi (Xi-X) (Yi-Y) (Xi-X)2 (Yi-Y)2 (Xi-X)(Yi-Y)


Number

1 0.90 15 0.1076 -0.64 0.0116 0.4096 -0.0689


2 0.75 20 -0.0424 4.36 0.0018 19.0096 -0.1849
3 1.05 10 -0.2576 -5.64 0.0664 31.8096 -1.4529
4 0.85 14 0.0576 -1.64 0.0033 2.6896 -0.0945
G 5 0.65 16 -0.1424 0.36 0.0203 0.1296 -0.513
R 6 1.20 20 0.4076 4.36 0.1661 19.0096 1.7771
O 7 0.90 24 0.1076 8.36 0.0116 69.8896 0.8995
U 8 0.84 26 0.0476 10.36 0.0023 107.3296 0.4931
P 9 0.93 11 0.1376 -4.64 0.0189 21.5296 -0.6385
10 0.78 18 -0.0124 2.36 0.0002 5.5696 -0.0293
I 11 0.96 12 0.1676 -3.64 0.0281 13.2496 -0.6101
12 1.02 25 0.2276 9.36 0.0518 87.6096 2.1303
13 0.81 26 0.0176 10.36 0.0003 107.3296 0.1823
14 0.76 30 -0.0324 14.36 0.0010 206.2096 -0.4653
15 1.02 28 0.2276 12.36 0.0518 152.7696 2.8131

16 0.76 10 -0.0324 -5.64 0.0010 31.8069 0.1827


17 0.68 12 -0.1124 -3.64 0.0126 13.2496 0.4091
G 18 0.56 4 -0.2324 -11.64 0.0540 135.4896 2.7051
R 19 0.62 18 -0.1724 2.36 0.0297 5.5696 -0.4069
O 20 0.92 -4 0.1276 -19.64 0.0163 385.7296 -2.5061
U 21 0.58 20 -0.2124 4.36 0.0451 19.0096 -0.9261
P 22 0.70 8 -0.0924 - 7.64 0.0085 58.3696 0.7059
23 0.52 15 –0.2724 -0.64 0.0742 0.4096 0.1743
II 24 0.45 6 –0.3424 -9.64 0.1172 92.9296 3.3007
25 0.60 7 –0.1924 -8.64 0.0370 74.6496 1.6623

19.81 391 0.8311 1661.76 9.539

Sum of Xi for group 1 13.42


X1 = = = 0.8947
15 15

Sum of Xi for group 2 6.39


X2 = = = 0.6390
10 10

Sum of Yi for group 1 295


Y1 = = = 19.67
15 15

Sum of Yi for group 2 96


Y2 = = = 9.60
10 10

1 0.8311
σ x
2
= ∑( Xi –X) =
2
= 0.0346
n-1 25-1

1 1661.76
σ y
2
= ∑( Yi – Y) =
2
= 69.24
n-1 25-1
1 10.0007
σ xy = ∑( Xi-X)(Yi-Y) = = 0.4167
n-1 25-1

dx = X1 - X2 = 0.8947 – 0.6390 = 0.2557

dy = Y1 – Y2 = 19.67 – 9.60 = 10.07

Substituting these values in the equations for a and b we get :

69.24 x 0.2557 – 0.4167 x 10.07


a = = 6.079
0.0346 x 69.24 – 0.4167 x 0.4167

0.0346 x 10.07 – 0.4167 x 0.2557


b= = 0.1089
0.0346 x 69.24 – 0.4167 x 0.4167

Hence , the discriminant function is :


Zi = 6.079 Xi + 0.1089 Yi

(b) Choice of the cutoff point


The Zi score for various accounts are shown below

Zi scores for various accounts

Account No. Zi Score

1 7.1046
2 6.7373
3 7.4720
4 6.6918
5 5.6938
6 9.4728
7 8.0847
8 7.9378
9 6.8514
10 6.7018
11 7.1426
12 8.9231
13 7.7554
14 7.8870
15 9.2498
16 5.7090
17 5.4405
18 3.8398
19 5.7292
20 5.1571
21 5.7038
22 5.1265
23 4.7946
24 3.3890
25 4.4097

The Zi scores arranged in an ascending order are shown below

Good(G)
Account Number Zi Score or
Bad (B)

24 3.3890 B
18 3.8398 B
25 4.4097 B
23 4.7946 B
22 5.1265 B
20 5.1571 B
17 5.4405 B
5 5.6938 G
21 5.7038 B
16 5.7090 B
19 5.7292 B
4 6.6918 G
10 6.7018 G
2 6.7373 G
9 6.8514 G
1 7.1046 G
11 7.1426 G
3 7.4720 G
13 7.7554 G
14 7.8870 G
8 7.9378 G
7 8.0847 G
12 8.9231 G
15 9.2498 G
6 9.4728 G
From the above table, it is evident that a Zi score which represents the mid-point between
the Zi scores of account numbers 19 and 4 results in the minimum number of misclassifications .
This Zi score is :

5.7292 + 6.6918
= 6.2105
2
Given this cut-off Zi score, there is just one misclassification (Account number 5)

CA
2 WCL =
(CA + NFA)– 0.2 CA

Dividing both the numerator and denominator by CA, we get

1
=1
1 +( NFA/CA) -0.2

0.8 +NFA/CA = 1 or NFA/CA =0.2

31.3
Account ROE(%)
Number )2 )2
Xi DER Y i (X i -X) (Y i -Y) (X i -X (Y i -Y (X i -X)(Y i -Y)
G 1 20 0.5 11.3125 -0.6250 127.9727 0.3906 -7.0703
O 2 18 0.6 9.3125 -0.5250 86.7227 0.2756 -4.8891
O 3 24 0.8 15.3125 -0.3250 234.4727 0.1056 -4.9766
D 4 15 0.9 6.3125 -0.2250 39.8477 0.0506 -1.4203
A 5 12 0.8 3.3125 -0.3250 10.9727 0.1056 -1.0766
C 6 9 0.5 0.3125 -0.6250 0.0977 0.3906 -0.1953
C 7 19 1 10.3125 -0.1250 106.3477 0.0156 -1.2891
T 8 16 1.2 7.3125 0.0750 53.4727 0.0056 0.5484
B 9 -6 2 -14.6875 0.8750 215.7227 0.7656 -12.8516
A 10 4 1.5 -4.6875 0.3750 21.9727 0.1406 -1.7578
D 11 2 0.9 -6.6875 -0.2250 44.7227 0.0506 1.5047
A 12 -5 1.8 -13.6875 0.6750 187.3477 0.4556 -9.2391
C 13 11 1.6 2.3125 0.4750 5.3477 0.2256 1.0984
C 14 7 0.8 -1.6875 -0.3250 2.8477 0.1056 0.5484
T 15 3 1.2 -5.6875 0.0750 32.3477 0.0056 -0.4266
16 -10 1.9 -18.6875 0.7750 349.2227 0.6006 -14.4828
From the above table we get the following
∑Xi = 139 ∑Yi= 18 ∑(Xi-X)2 ∑( Yi-Y)2 ∑( Xi-X)(Yi-Y)

X = 8.6875 Y = 1.125 = 1519.438 = 3.69 = -55.975


Sum of Xi for Good Accts. 133
X1 = = = 16.625
8 8

Sum of Xi for Bad Accts. 6


X2 = = = 0.75
8 8

Sum of Yi for Good Accts. 6.3


Y1 = = = 0.7875
8 8

Sum of Yi for Bad Accts. 11.7


Y2 = = = 1.4625
8 8
1 1519.438
σ x
2
= ∑( Xi –X)2 = = 101.30
n-1 16-1

1 3.69
σ y
2
= ∑( Yi – Y) =
2
= 0.246
n-1 16-1

1 -55.975
σ xy = ∑( Xi-X)(Yi-Y) = = - 3.73
n-1 16 -1

dx = X1 - X2 = 16.625 – 0.75 = 15.875

dy = Y1 – Y2 = 0.7875 – 1.4625 = - 0.675

Substituting these values in the equations for a and b we get :

0.246 x 15.875 - 3.73 x 0.675


a = = 0.1261
101.30 x 0.246 – 3.73 x 3.73

-101.30 x 0.675 + 3.73 x 15.875


b= = - 0.8325
101.30 x 0.246 – 3.73 x 3.73
Hence , the discriminant function is :
Zi = 0.1261 Xi - 0.8325 Yi

Chapter 32
CORPORATE VALUATION

1. (a) The calculations for Hitech Limited are shown below :


Year 2 Year3
EBIT
PBT 86 102
+ Interest expense 24 28
- Interest income (10) (15)
- Non-operating income (5) (10)
EBIT 95 105

Tax on EBIT
Tax provision on income statement 26 32
+ Tax shield on interest expense 9.6 11.2
- Tax on interest income (4) (6)
- Tax on non-operating income (2) (4)
Tax on EBIT 29.6 33.2

NOPLAT 65.4 71.8


Net investment (50) (50)
Non-operating cash flow (post-tax) 3 6
FCFF 18.4 27.8

(b) The financing flow for years 2 and 3 is as follows :


Year 2 Year 3
After-tax interest expense 14.4 16.8
Cash dividend 30 40
- Net borrowings (30) (30)
+ ∆ Excess marketable securities 30 10
- After-tax income on excess (6) (9)
marketable securities
- Share issue (20) -
18.4 27.8

(c) Year 2 Year 3


Invested capital (Beginning) 310 360
Invested capital (Ending) 360 410
NOPLAT 65.4 71.8
Turnover 400 460
Net investment 50 50

Post-tax operating margin 16.35% 15.61%


Capital turnover 1.29 1.28
ROIC 21.1% 19.9%
Growth rate 16.1% 13.9%
FCF 15.4 21.8

2. Televista Corporation

0 1 2 3 4 5
Base year

1. Revenues 1600 1920 2304 2765 3318 3650


2. EBIT 240 288 346 415 498 547
3. EBIT (1-t) 156 187 225 270 323 356
4. Cap. exp. 200 240 288 346 415 -
- Depreciation 120 144 173 207 249
5. Working capital 400 480 576 691 829 912
6. ∆ Working capital 80 96 115 138 83
7. FCFF 11 13 16 19 273
(3-4-6)

Discount factor 0.876 0.767 0.672 .589


Present value 9.64 9.97 10.76 11.19

Cost of capital for the high growth period

0.4 [12% + 1.25 x 7%] + 0.6 [15% (1 - .35)]


8.3% + 5.85%
= 14.15%

Cost of capital for the stable growth period


0.5 [12% + 1.00 x 6%] + 0.5 [14% (1 - .35)]
9% + 4.55%
= 13.55%

Present value of FCFF during the explicit forecast period


= 9.64 + 9.97 + 10.76 + 11.19 = 41.56
273 273
Horizon value = = = 7690
0.1355 – 0.10 0.0355
Present value of horizon value = 4529.5

Value of the firm = 41.56 + 4529.50 = Rs.4571.06 million

3. The WACC for different periods may be calculated :

WACC in the high growth period

Year kd(1-t) = 15% (1-t) ke = Rf + β x Market risk premium ka = wd kd (1-t)+ we ke


1 15 (0.94) = 14.1% 12 + 1.3 x 7 = 21.1% 0.5 x 14.1 + 0.5 x 21.1 = 17.6%
2 15 (0.88) = 13.2% 21.1% 0.5 x 13.2 + 0.5 x 21.1 = 17.2%
3 15 (0.82) = 12.3% 21.1% 0.5 x 12.3 + 0.5 x 21.1 = 16.7%
4 15 (0.76) = 11.4% 21.1% 0.5 x 11.4 + 0.5 x 21.1 = 16.3%
5 15 (0.70) = 10.5% 21.1% 0.5 x 10.5 + 0.5 x 21.1 = 15.8%

WACC in the transition period


kd(1-t) = 14 (1 – 0.3) = 9.8%
ke = 11 + 1.1 x 6 = 17.6%
ka = 0.44 x 9.8 + 0.56 x 17.6 = 14.2%

WACC for the stable growth period


kd(1-t) = 13 (1 – 0.3) = 9.1%
ke = 11 + 1.0 x 5 = 16%
ka = 1/3 x 9.1 + 2/3 x 16 = 13.7%

The FCFF for years 1 to 11 is calculated below. The present value of the
FCFF for the years 1 to 10 is also calculated below.
3
Multisoft Limited

Period Growth EBIT Tax EBIT Cap. Dep. ∆ WC FCFF D/E Beta WACC PV Present
rate (%) rate (1-t) exp. % Factor value
(%)
0 90 100 60
1 40 126 6 118 140 84 26 36 1:1 1.3 17.6 .850 30.6
2 40 176 12 155 196 118 39 38 1:1 1.3 17.2 .726 27.6
3 40 247 18 203 274 165 50 44 1:1 1.3 16.7 .622 27.4
4 40 346 24 263 384 230 70 39 1:1 1.3 16.3 .535 20.8
5 40 484 30 339 538 323 98 26 1:1 1.3 15.8 .462 12.0
6 34 649 30 454 721 432 132 33 0.8:1 1.1 14.2 .405 13.4
7 28 830 30 581 922 553 169 43 0.8:1 1.1 14.2 .354 15.4
8 22 1013 30 709 1125 675 206 53 0.8:1 1.1 14.2 .310 16.7
9 16 1175 30 822 1305 783 239 61 0.8:1 1.1 14.2 .272 16.9
10 10 1292 30 905 1436 862 263 68 0.8:1 1.1 14.2 .238 16.6
11 10 1421 30 995 1580 948 289 74 0.5: 1.1 13.7 476
1.0
673.4
The present value of continuing value is :

FCF11 74
x PV factor 10 years = x 0.238 = 476
k–g 0.137 – 0.100

This is shown in the present value cell against year 11.

The value of the firm is equal to :


Present value of FCFF during + Present value of continuing
The explicit forecast period of 10 years value

This adds up to Rs.685.4 million as shown below


MINI CASE

Solution:
1 2 3 4 5 6
1. Revenues 950 1,000 1,200 1,450 1,660 1,770
2. PBIT 140 115 130 222 245 287
3. NOPAT = PBIT 91 74.8 84.5 144.3 159.3 186.6
(1 – .35)
4. Depreciation 55 85 80 83 85 87
5. Gross cash flow 146 159.8 164.5 227.3 244.3 273.7
6. Gross investment 100 250 85 100 105 120
in fixed assets
7. Investment in net 10 15 70 70 70 54
current assets
8. Total investment 110 265 155 170 175 174
9. FCFF (5) – (8) 36 (105.2) 9.5 57.3 69.3 99.6

0.4 1.0
WACC = x 12 x (1 – 0.35) + {8 + 1.06 (8)}
1.4 1.4

= 14%

99.6 (1.10)
Continuing Value = = 2739.00
0.14 – 0.10

2739
Present value of continuing value = = 1249
6
(1.14)

PV of the FCFF during the explicit forecast period


3.6 105.2 9.5 57.3 69.3 99.6
= – + + + +
(1.14) (1.14)2 (1.14)3 (1.14)4 (1.14)5 (1.14)6

= 72.4
Firm value = 72.4 + 1249 = 1321.4

Value of equity = 1321.4 – 200 = 1121.4 million


Chapter 33
VALUE BASED MANAGEMENT

1. The value created by the new strategy is calculated below :

Current Income Statement Projection


Values
(Year 0) 1 2 3 4 5

• Sales 2000 2240 2509 2810 3147 3147


• Gross margin (20%) 400 448 502 562 629 629
• Selling and general 160 179 201 225 252 252
administration (8%)
• Profit before tax 240 269 301 337 378 378
• Tax 72 81 90 101 113 113
• Profit after tax 168 188 211 236 264 264

Balance Sheet Projections


• Fixed assets 600 672 753 843 944 944
• Current assets 600 672 753 843 944 944
• Total assets 1200 1344 1505 1696 1888 1888
• Equity 1200 1344 1505 1686 1888 1888

Cash Flow Projections


• Profit after tax 188 211 236 264 264
• Depreciation 60 67 75 84 94
• Capital expenditure 132 148 166 185 94
• Increase in current assets 72 81 90 101 -
• Operating cash flow 44 49 55 62 264

• Present value of the operating cash flow = 147


• Residual value = 264 / 0.15 = 1760
• Present value of residual value = 1760 / (1.15)4 = 1007
• Total shareholder value = 147 + 1007 = 1154
• Pre-strategy value = 168/0.15 = 1120
• Value of the strategy = 1154 – 1120 = 34

2. According to the Marakon approach


M r–g
=
B k–g

r - .10
2 =
k - .10
r - .10 = 2k - .20
r = 2k - .10
r/k = 2 - (.10/k)

Thus r/k is a function of k. Unless k is specified r/k cannot be determined.

NOPAT for 20X1


PBIT (1 – T) = 24 (0.65) = 15.6
Cost of equity
6% + 0.9 (6%) = 11.4%

Average cost of capital


0.5 x 8% (1 - .35) + 0.5 x 11.4% = 8.3%

EVA for 20X1


NOPAT - Average cost of capital x Capital employed
15.6 - .083 x 100 = 7.3

4.
I = Rs.200 million
r = 0.40
c* = 0.20
T = 5 years
200 (0.40 – 0.20) 5
Value of forward plan =
0.20 (1.20)

= Rs.833.3 million

5. Cost of capital = 0.5 x 0.10 + 0.5 x 0.18 = 0.14 or 14 per cent

1. Revenues 2,000 2,000 2,000 2,000 2,000


2. Costs 1,400 1,400 1,400 1,400 1,400
3. PBDIT 600 600 600 600 600
4. Depreciation 200 200 200 200 200
5. PBIT 400 400 400 400 400
6. NOPAT 240 240 240 240 240
7. Cash flow (4+6) 440 440 440 440 440
8. Capital at charge 1,000 800 600 400 200
9. Capital charge (8x0.14) 140 112 84 56 28
10. EVA (6-9) 100 128 156 184 212
5 440
NPV = ∑ - 1000 = 440 x 3.433 – 1000 = 510.5
t
t=1 (1.14)

EVAt
NPV = ∑ = 100 x 0.877 + 128 x 0.769 + 156 x 0.675 + 184 x 0.592 +
t
(1.14) 212 x 0.519
= 510.3

6. Equipment cost = 1,000,000


Economic life = 4 years
Salvage value = Rs.200,000
Cost of capital = 14 per cent

Present value of salvage value = 200,000 x 0.592


= 118,400

Present value of the annuity = 1,000,000 – 118,400


= 881,600

881,600 881,600
Annuity amount = =
PVIFA14%, 4yrs 2.914

= Rs.302,540

Depreciation charge under sinking fund method


1 2 3 4
Capital 1,000,000 837,460 652,164 440,927
Depreciation 162,540 185,296 212,237 240,810
Capital charge 140,000 117,244 91,303 61,730
Sum 302,540 302,540 302,540 302,540

7. Investment : Rs.2,000,000
Life : 10 years
Cost of capital : 15 per cent
Salvage value : 0
2,000,000
Economic depreciation =
FVIFA(10yrs, 15%)

2,000,000
= = 98,503
20.304

8. Investment : Rs.5,000,000
Life : 5 years
Cost of capital : 12 per cent
Salvage value : Nil

PVIFA(5yrs,12%) = 3.605 ; Annuity amount = 5,000,000 / 3.605 = 1,386,963

Depreciation charge under sinking fund method


1 2 3 4 5
Capital 5,000,000 4,213,037 3,331,638 2,344,472 1,238,846
Depreciation 786,963 881,399 987,166 1,105,626 1,238,301
Capital charge 600,000 505,564 399,797 281,336 148,662
Sum 1,386,963 1,386,963 1,386,963 1,386,963 1,386,963

5,000,000
Economic depreciation =
FVIFA(5yrs, 12%)

5,000,000
= = Rs.787,030
6.353

9. (a) Investment = Rs.100 million


Net working capital = Rs.20 million
Life = 8 yrs
Salvage value = Rs.20 million (Net working capital)
Annual cash flow = Rs.21.618 million
Cost of capital = 15%
Straight line depreciation = Rs.10 million per year

80 80
Economic depreciation = = = Rs.5.828 million
FVIFA(8, 15%) 13.727
Year 1 Year 4
• Profit after tax 11.618 11.618
• Depreciation 10.000 10.000
• Cash flow 21.618 21.618
• Book capital 100 70
(Beginning)
• ROCE 11.62% 16.59%
• ROGI 21.62% 21.62%
• CFROI 15.79% 15.79%

(b)
Year 1 Year 4

EVA 11.618 – 100 x 0.15 11.618 - 70 x 0.15


= - 3.382 million = - 8.882 million
CVA (11.618 + 10) – 5.828-(100x0.15) (11.618+10)-5.828- (100x0.15)
= 0.79 million = 0.79 million
MINICASE

1. Both HLL and Infosys have excluded extraordinary or exceptional items.

2. HLL calculated NOPAT as:


PAT (1- T) + INT (1-T)
Remember that NOPAT can be calculated as
PBIT (1-T) or
as PAT (1-T) + INT (1-T). The two are equivalent.

Infosys calculated NOPAT as:


OPERATING PROFIT LESS TAXES

Since Infosys is a zero debt company with nil interest, operating profit less taxes is
equivalent to PBIT (1-T).

3. For calculating the cost of equity both HLL and Infosys have used the Capital Asset Pricing
Model. However, they have used somewhat different inputs for the risk-free rate and the
market risk premium.

• HLL used a risk-free rate of 6.95% whereas Infosys used a risk-free


rate of 7.50%. To some extent this difference may be because HLL’s financial year
ended on 31/12/2005 and Infosys’s financial year ended on 31/3/2006.

• HLL assumed a market risk premium of 9% whereas Infosys used a


market risk premium of 7%. This difference is, however, substantial.

4. Both HLL and Infosys have used a beta variant without explaining how the same has been
calculated. HLL has used a beta variant of 0.95 for the year 2005. It seems reasonable for
an FMCG major like HLL. Infosys has used a beta variant of 0.78 for 2006. Interestingly,
the beta variant of Infosys was 1.41 in 2006 and declined steadily to 0.78 in 2006. This
reflects the diminished riskiness of Infosys.

5. On the whole, the procedures used by the two companies seem reasonable. However, one
would have liked to know the rationale of their assumptions and the exact method for the
calculation of beta variant.
Chapter 34
MERGERS, ACQUISITIONS AND RESTRUCTURING

1. The pre-amalgamation balance sheets of Cox Company and Box Company and the post-
amalgamation balance sheet of the combined entity, Cox and Box Company, under the ‘pooling’
method as well as the ‘purchase’ method are shown below :

Before Amalgamation After Amalgamation


Cox & Box Company
Cox Box Pooling method Purchase
method
Fixed assets 25 10 35 45
Current assets 20 7.5 27.5 30
Goodwill 2.5
Total assets 45 17.5 62.5 77.5

Share capital 20 5 25 20
(face value @ Rs.10)
Reserves & surplus 10 10 20 10
Debt 15 2.5 17.5 17.5
45 17.5 42.5 77.5

2. Post-merger EPS of International Corporation will be

2 x 100,000 + 2 x100,000

100,000 + ER x 100,000

Setting this equal to Rs.2.5 and solving for ER gives


ER = 0.6

3. PVA = Rs.25 million, PVB = Rs.10 million


Benefit = Rs.4 million, Cash compensation = Rs.11 million
Cost = Cash compensation – PVB = Rs.1 million
NPV to Alpha = Benefit – Cost = Rs.3 million
NPV to Beta = Cash Compensation – PVB = Rs.1 million

4. Let A stand for Ajeet and J for Jeet


PVA = Rs.60 x 300,000 = Rs.18 million
PVJ = Rs.25 x 200,000 = Rs.5 million
Benefit = Rs.4 million
PVAJ = 18 + 5 + 4 = Rs.23 million
Exchange ratio = 0.5
The share of Jeet in the combined entity will be :
100,000
α = = 0.25
300,000 + 100,000

a) True cost to Ajeet Company for acquiring Jeet Company


Cost = α PVAB - PVB
= 0.25 x 27 - 5 = Rs.1.75 million

b) NPV to Ajeet
= Benefit - Cost
= 4 - 1.75 = Rs.2.25 million

c) NPV to Jeet = Cost = Rs.1.75 million

5. a) PVB = Rs.12 x 2,000,000 = Rs.24 million


The required return on the equity of Unibex Company is the value of k in the
equation.

Rs.1.20 (1.05)
Rs.12 =
k - .05

k = 0.155 or 15.5 per cent.

If the growth rate of Unibex rises to 7 per cent as a sequel to merger, the intrinsic
value per share would become :

1.20 (1.07)
= Rs.15.11
0.155 - .07

Thus the value per share increases by Rs.3.11 Hence the benefit of the
acquisition is
2 million x Rs.3.11 = Rs.6.22 million

(b) (i) If Multibex pays Rs.15 per share cash compensation, the cost of the
merger is 2 million x (Rs.15 – Rs.12) = Rs.6 million.
(ii) If Multibex offers 1 share for every 3 shares it has to issue 2/3 million
shares to shareholders of Unibex.

So shareholders of Unibex will end up with

0.667
α = = 0.1177 or 11.77 per cent
5+0.667

shareholding of the combined entity,


The present value of the combined entity will be
PVAB = PVA + PVB + Benefit
= Rs.225 million + Rs.24 million + Rs.6.2 million
= Rs.255.2 million

So the cost of the merger is :


Cost = α PVAB - PVB
= .1177 x 255.2 - 24 = Rs.6.04 million

6. The expected profile of the combined entity A&B after the merger is shown in the last column
below.

A B A&B
Number of shares 5000 2000 6333
Aggregate earnings Rs.45000 Rs.4000 Rs.49000
Market value Rs.90000 Rs.24000 Rs.114000
P/E 2 6 2.33

7. Value of Alpha Limited’s equity as a stand-alone company.

50 55 60 64 70 70 (1.06) 1
+ + + + + x
(1.12) (1.12)2 (1.12)3 (1.12)4 (1.12)5 0.12 – 0.06 (1.12)5

= Rs. 912.79 million

Value of the equity of the combined company.

80 90 105 120 135 135 (1.05) 1


+ + + + + x
(1.12) (1.12)2 (1.12)3 (1.12)4 (1.12)5 0.12 – 0.05 (1.12)5

= Rs. 1518.98 million


Let a be the maximum exchange ratio acceptable to the shareholders of Alpha Limited. Since the
management of Alpha Limited wants to ensure that the net present value of equity-related cash
flows increases by at least 5 percent, the value of a is obtained as follows.

10
x 1518.98 = 1.05 x 912.79
10 + a 8

Solving this for a we get

a = 0.7311

Note that the number of outstanding shares of Alpha Limited and Beta Limited are 10 million and
8 million respectively.

8. (a) The maximum exchange ratio acceptable to shareholders of Vijay Limited is :

S1 (E1+E2) PE12
ER1 = - +
S2 P1S2

12 (36+12) 8
= - + = 0.1
8 30 x 8

(b) The minimum exchange ratio acceptable to shareholders of Ajay Limited is :


P2 S1
ER2 =
(PE12) (E1+E2) - P2 S2

9 x 12
= = 0.3
9 (36+12) - 9 x 8

(c) 12 (48) PE12


ER1 = - +
8 240
9 x 12
ER2 =
PE12 (48) - 72
Equating ER1 and ER2 and solving for PE12 gives, PE12 = 9
When PE12 = 9
ER1 = ER2 = 0.3
Thus ER1 and ER2 intersect at 0.3

9. The present value of FCF for first seven years is


16.00 14.30 9.7 0
PV(FCF) = - - - +
(1.15) (1.15)2 (1.15)3 (1.15)4

0 10.2 16.7
+ + +
(1.15)5 (1.15)6 (1.15)7

= - Rs.20.4 million
The horizon value at the end of seven years, applying the constant growth model is

FCF8 18
V4 = = = Rs.257.1 million
0.15-0.08 0.15 – 0.08

1
PV (VH) = 257.1 x = Rs.96.7 million
7
(1.15)

The value of the division is :


- 20.4 + 96.7 = Rs.76.3 million
MINICASE

Solution:
(a)
Modern Pharma Magnum Drugs Exchange
Ratio
Book value per share 2300 650 65
= Rs.115 = Rs.65
20 10 115
Earnings per share 450 95 9.5
= Rs.22.5 = Rs.9.5
20 10 22.5
Market price per share Rs.320 Rs.102 102

320

Exchange ratio that gives equal weightage to book value per share, earnings per share, and market
price per share

65 9.5 102
+ +
115 22.5 320 0.57 + 0.42 + 0.32
= = 0.44
3 3

(b) An exchange ratio based on earnings per share fails to take into account the
following:

(i) The difference in the growth rate of earnings of the two companies.
(ii) The gains in earnings arising out of merger.
(iii) The differential risk associated with the earnings of the two companies.

(c) Current EPS of Modern Pharma


450
= = Rs.22.5
20

If there is a synergy gain of 5 percent, the post-merger EPS of Modern Pharma is

(450 + 95) (1.05)

20 + ER X 10
Equating this with Rs.22.5, we get
(450 + 95) (1.05)
= 22.5
20 + 10ER
This gives ER = 0.54

Thus the maximum exchange ratio Modern Pharma should accept to avoid initial dilution of EPS
is 0.54

(d) Post-merger EPS of Modern Pharma if the exchange ratio is 1:4, assuming no
synergy gain:

450 + 95
= Rs.24.2
20 + 0.25 x 10

(e) The maximum exchange ratio acceptable to the shareholders of Modern Pharma if
the P/E ratio of the combined entity is 13 and there is no synergy gain

-S1 (E1 + E2) P/E12


ER1 = +
S2 P1 S2

- 20 (450 + 95) 13
= + = 0.21
10 320 x 10

(f) The minimum exchange ratio acceptable to the shareholders of Magnum Drugs if
the P/E ratio of the combined entity is 12 and the synergy benefit is 2 percent
P2S1
ER2 =
(P/E12) (E1 + E2) (1 + S) – P2S2

102 x 20
=
12 (450 + 95) (1.02) – 102 X 10
= 0.36

(g) The level of P/E ratio where the lines ER1 and ER2 intersect.

To get this, solve the following for P/E12


- S1 (E1 + E2) P/E12 P2S1
+ =
S2 P1S2 P/E12 (E1 + E2) – P2S2

- 20 (450 +95) P/E12 102 x 20


+ =
10 320 x 10 P/E12 (450 +95) – 1020

- 6400 + 545 P/E12 2040


=
3200 545 P/E12 – 1020

(545 P/E12 – 1020) (545 P/E12 – 6400) = 2040 x 3200

297025 P/E212 – 3488000 P/E12 – 555900 P/E12


+6528000 = 6528000
2
297025 P/E 12 = 4043900 P/E
297025 P/E12 = 4043900
P/E12 = 13.61
Chapter 37
INTERNATIONAL FINANCIAL MANAGEMENT

1. The annualised premium is :

Forward rate – Spot rate 12


x
Spot rate Forward contract length in months

46.50 – 46.00 12
= x = 4.3%
46.00 3

2. 100
100 (1.06) = x 1.07 x F
1.553

106 x 1.553
F = = 1.538
107
A forward exchange rate of 1.538 dollars per sterling pound will mean indifference between
investing in the U.S and in the U.K.

3. (a) The annual percentage premium of the dollar on the yen may be calculated with
reference to 30-days futures
105.5 – 105 12
x = 5.7%
105 1

(b) The most likely spot rate 6 months hence will be : 107 yen / dollar

(c) Forward rate 1 + domestic interest rate


=
Spot rate 1 + foreign interest rate

107 1 + domestic interest rate in Japan


=
105 1.03

Domestic interest rate in Japan = .0496 = 4.96 per cent

4. S0 = Rs.46 , rh = 11 per cent , rf = 6 per cent


Hence the forecasted spot rates are :
Year Forecasted spot exchange rate
1 Rs.46 (1.11 / 1.06)1 = Rs.48.17
2 Rs.46 (1.11 / 1.06)2 = Rs.50.44
3 Rs.46 (1.11 / 1.06)3 = Rs.52.82
4 Rs.46 (1.11 / 1.06)4 = Rs.55.31
5 Rs.46 (1.11 / 1.06)5 = Rs.57.92

The expected rupee cash flows for the project

Year Cash flow in dollars Expected exchange Cash flow in rupees


(million) rate (million)
0 -200 46 -9200
1 50 48.17 2408.5
2 70 50.44 3530.8
3 90 52.82 4753.8
4 105 55.31 5807.6
5 80 57.92 4633.6

Given a rupee discount rate of 20 per cent, the NPV in rupees is :

2408.5 3530.8 4753.8


NPV = -9200 + + +
(1.18) (1.18)2 (1.18)3

5807.6 4633.6
+ +
(1.18)4 (1.18)5

= Rs.3291.06 million

The dollar NPV is :


3291.06 / 46 = 71.54 million dollars

5. Forward rate 1 + domestic interest rate


=
Spot rate 1 + foreign interest rate

F 1 + .015
=
1.60 1 + .020
F = $ 1.592 / £
6. Expected spot rate a year from now 1 + expected inflation in home country
=
Current spot rate 1 + expected inflation in foreign country

Expected spot rate a year from now 1.06


=
Rs.70 1.03

So, the expected spot rate a year from now is : 72 x (1.06 / 1.03) = Rs.72.04

7. (a) The spot exchange rate of one US dollar should be :


12000
= Rs.48
250
(b) One year forward rate of one US dollar should be :
13000
= Rs.50
260

8. (1 + expected inflation in Japan)2


Expected spot rate = Current spot rate x
2 years from now (1 + expected inflation in UK)2

(1.01)2
= 170 x = 163.46 yen / £
2
(1.03)

9. (i) Determine the present value of the foreign currency liability (£100,000) by using
90-day money market lending rate applicable to the foreign country. This works
out to :
£100,000
= £ 98522
(1.015)
(ii) Obtain £98522 on today’s spot market
(iii) Invest £98522 in the UK money market. This investment will grow to
£100,000 after 90 days

10. (i) Determine the present value of the foreign currency asset (£100,000) by using
the 90-day money market borrowing rate of 2 per cent.
100,000
= £98039
(1.02)
(ii) Borrow £98039 in the UK money market and convert them to dollars in the spot
market.

(iii) Repay the borrowing of £98039 which will compound to £100000 after 90 days
with the collection of the receivable

11. A lower interest rate in the Swiss market will be offset by the depreciation of the US
dollar vis-à-vis the Swiss franc. So Mr.Sehgal’s argument is not tenable.

12
INR/CHF = (INR/USD) x (USD/CHF) = 0.0248 x 1.2056 = 0.0299
13
As the forward bid in points is more than the offer rate in points the forward rate is at a
discount. So we have to subtract the points from the respective spot rate. The outright one
month forward quotation for USD/INR is therefore: 41.3524 / 41.3534
( Note that one swap point = 0.0001)
14
USD/INR Spot midrate = (41.3424 + 41.3435)/2 = 41.34295
USD/INR 1 month forward midrate = ( 41.2050 + 41.2060)/2 = 41.2055
As the forward rate indicates lesser rupee for a dollar, the rupee is at a premium.
The annual percentage of premium = [(41.34295 – 41.2055)/ 41.34295] x 12 = 0.0399

MINICASE

Outright rates Spot 90 days 180 days


JPY per USD 117.43/117.45 115.83/116.03 113.55/114.73
INR per USD 44.86/44.87 45.14/45.17 45.32/45.35
Rupee receivable per JPY 44.87/117.43 45.17/115.83 45.35/113.55
=0.3821 0.3900 0.3994
Annualised Premium 8.27 % 9.06 % (A)

Required annualised return 12% 12% (B)


Interest rate per annum to be quoted
( B - A) 3.73 % 2.94 %
Price to be quoted in Yen
for immediate payment 5,000/0.3821
=13,085.58

Chapter 40
CORPORATE RISK MANAGEMENT
1. (a) The investor must short sell Rs.1.43 million (Rs.1 million / 0.70) of B
(b) His hedge ratio is 0.70
(c) To create a zero value hedge he must deposit Rs.0.43 million

2. Futures price Spot price x Dividend yield


= Spot price -
(1+Risk-free rate)0.5 (1+Risk-free rate)0.5

4200 4000 x Dividend yield


= 4000 -
(1.145) 0.5 (1.145) 0.5

The dividend yield on a six months basis is 2 per cent. On an annual basis it is approximately
4 per cent.

3. Futures price
= Spot price + Present value of – Present value
(1+Risk-free rate)1 storage costs of convenience yield

5400
= 5000 + 250 – Present value of convenience yield
(1.15)1

Hence the present value of convenience yield is Rs.554.3 per ton.

4
LIBOR -25BP SWAP LIBOR - 25BP
BANK

5.25% 5%

EXCEL APPLE
EXCEL CORPN. LTD.

LIBOR+ 50BP 5%

You might also like